Jump to content

Wikipedia:Reference desk/Science: Difference between revisions

From Wikipedia, the free encyclopedia
Content deleted Content added
Line 1,042: Line 1,042:
You can also try [[Wikipedia:WikiProject Resource Exchange]], either posting to the requests page or asking one of the members. You can count me out though, as I don't have a subscription to that series. [[User:Someguy1221|Someguy1221]] ([[User talk:Someguy1221|talk]]) 22:15, 2 June 2008 (UTC)
You can also try [[Wikipedia:WikiProject Resource Exchange]], either posting to the requests page or asking one of the members. You can count me out though, as I don't have a subscription to that series. [[User:Someguy1221|Someguy1221]] ([[User talk:Someguy1221|talk]]) 22:15, 2 June 2008 (UTC)


:It is not illegal to distribute single copies of articles for academic purposes—it easily falls under [[fair use]] (even if might be a breach of user policies for the journals, but even then, most journals are pretty lenient when you are talking about very small-scale, not-for-profit, one-off stuff). My university doesn't carry e-copies of that one either (which surprises me), otherwise I'd be happy to send you a copy. --[[User:Captain Ref Desk|Captain Ref Desk]] ([[User talk:Captain Ref Desk|talk]]) 22:15, 2 June 2008 (UTC)
:It is not illegal to distribute single copies of articles for academic purposes—it easily falls under [[fair use]] (even if might be a breach of user policies for the journals, but even then, most journals are pretty lenient when you are talking about very small-scale, not-for-profit, one-off stuff). My university doesn't carry e-copies of that one either (which surprises me), otherwise I'd be happy to send you a copy. Your best bet might be e-mailing one of the authors, if you want it quickly. It seems like a pretty rare journal. --[[User:Captain Ref Desk|Captain Ref Desk]] ([[User talk:Captain Ref Desk|talk]]) 22:15, 2 June 2008 (UTC)


== Liquid state of water ==
== Liquid state of water ==

Revision as of 22:17, 2 June 2008

Welcome to the science section
of the Wikipedia reference desk.
Select a section:
Want a faster answer?

Main page: Help searching Wikipedia

   

How can I get my question answered?

  • Select the section of the desk that best fits the general topic of your question (see the navigation column to the right).
  • Post your question to only one section, providing a short header that gives the topic of your question.
  • Type '~~~~' (that is, four tilde characters) at the end – this signs and dates your contribution so we know who wrote what and when.
  • Don't post personal contact information – it will be removed. Any answers will be provided here.
  • Please be as specific as possible, and include all relevant context – the usefulness of answers may depend on the context.
  • Note:
    • We don't answer (and may remove) questions that require medical diagnosis or legal advice.
    • We don't answer requests for opinions, predictions or debate.
    • We don't do your homework for you, though we'll help you past the stuck point.
    • We don't conduct original research or provide a free source of ideas, but we'll help you find information you need.



How do I answer a question?

Main page: Wikipedia:Reference desk/Guidelines

  • The best answers address the question directly, and back up facts with wikilinks and links to sources. Do not edit others' comments and do not give any medical or legal advice.
See also:


May 27

Mars rovers

If I understand correctly, the purpose of the latest rover is to ascertain whether life was once possible (or present) on Mars. My question is, what is the purpose of knowing whether it was or wasn't? If it was, does that make Mars any more of a candidate for terraforming? Or is it really just an expensive curiousity for scientists? Thanks. Vranak (talk) 00:02, 27 May 2008 (UTC)[reply]

If there was life in the past on Mars, it will tell us a lot about the chances of there being life elsewhere in the Universe. The question of "are we alone?" is actually pretty important for many people. Also, finding traces of past life would help us to better understand how life evolved on our own planet. Finding actual existing living things (maybe deep down in the rocks), of course, would be one of the greatest discoveries ever made by mankind. However, it's very unlikely Mars will ever be "terraformed", since it's lost its atmosphere and is really, really cold. Franamax (talk) 01:32, 27 May 2008 (UTC)[reply]
(edit conflict)The recent lander is not a "rover" although it is a digger and analyzer. It is indeed an "expensive curiousity" for those who are interested in learning about what is "out there" beyond their familiar environment, besides the scientists who build the space probes and operate them. That is one reason they are covered extensively in newspapers and tv news channels, for whom scientists are a vanishingly small fraction of the viewership. This one's chemistry set will not test for life, but it will do some chemical analysis to see if there are suitable conditions for life, or if there were in the past. Determining this will provide insight into whether our planet is the only place suitable for life to evolve. Edison (talk) 01:36, 27 May 2008 (UTC)[reply]
Thank you. Vranak (talk) 02:09, 27 May 2008 (UTC)[reply]

Someone once said something like: "To a scientist, there are only three interesting numbers: zero, one, and many, and 'one' is usually an aberration." That is, scientists are often involved with trying to decide whether a thing is impossible (so zero instances of it exist) or possible (so many instances of it exist). This includes the question "How many planets in the Universe have life?". To prove a second planet has/had life would shift the answer from "one" to "many, and this would require a very significant change in all of humanity's worldwview "universeview".

Atlant (talk) 14:24, 27 May 2008 (UTC)[reply]

I had the opportunity to attend a lecture from the former director of Jet Propulsion Laboratory. (Can't remember his name, unfortunately). According to him, the motivation for finding life was one of funding! (This should come as no surprise to anyone who works in any scientific field). Surprising (to me at least), the federal government is much more willing to fund Mars missions if the notion of "finding life" or "finding possibility of life" or some such stuff is mentioned in the mission proposal. This has become such a high priority that it defies scientific data... most of the Mars scientists I've ever met do not believe in life on Mars now or ever. (There's a few outliers, of course, and as the adage goes, the absence of evidence is not the evidence of absence, and so forth). All the NASA releases are careful to word their releases to discuss the possibility of sustaining life, because this is ambiguous enough to allow for many types of missions (after all, what does it really mean to be "possible" to sustain life?) Nimur (talk) 15:27, 27 May 2008 (UTC)[reply]
Huh? I thought the recent lander was a Canadian weather station set up at 70 degrees latitude to see if water is possible? Or am I confusing it for another lander? Or is it the same lander but different details? Thanks. ~AH1(TCU) 01:28, 28 May 2008 (UTC)[reply]
Phoenix (spacecraft) is "a partnership of universities in the United States, Canada, Switzerland, the Philippines, Denmark, Germany and the United Kingdom, NASA, the Canadian Space Agency, the Finnish Meteorological Institute, Lockheed Martin Space Systems, and other aerospace companies." Nimur (talk) 05:01, 28 May 2008 (UTC)[reply]
From a letter (reproduced in his book) from Lazlo Toth to NASA on the occasion of the Viking_program: "But this [finding organic chemicals in the analysis of the sample] wouldn't mean that there is life on Mars; it would mean that there was life on Mars, but you killed it!" Gzuckier (talk) 15:18, 29 May 2008 (UTC)[reply]

Pathogens in body

How would you be able to tell if you body detected a pathogen? I was wondering what would be the tell tale signs or even symptoms —Preceding unsigned comment added by 220.235.59.49 (talk) 00:03, 27 May 2008 (UTC)[reply]

Fever--Lenticel (talk) 00:18, 27 May 2008 (UTC)[reply]
Immune system and infection may hold some answers. Apart from fever there's increased white blood cell count. If you have a wound, pus will show. And there's the signal system reporting pain to rally the troops to the site where an infection has caused tissue damage. (Don't we really enjoy that part.:-) In case of your body detecting cold viruses you'll notice the increased histamine levels by your stuffy head. If your body detects harmful pathogens in the digestive system it will effect a purge. (either up or down). Your body is also likely to produce sleep inducing chemicals. These help the immune response and free up resources for healing. --71.236.23.111 (talk) 03:01, 27 May 2008 (UTC)[reply]
Revise your understanding. Your body encounters and deals with pathogens all the time. A pathogen is simply an organism that can sometimes cause noticeable harm to some people in some circumstances. Some pathogens are so virulent that the likelihood of noticeable harm is high, others are usually so benign that only people with severely compromised defenses are harmed. 159.14.240.230 (talk) 12:00, 27 May 2008 (UTC)[reply]

Drunk animals and Marula trees

There is a video on youtube of African animals supposedly getting drunk off the fruit of the Marula tree (www.youtube.com/watch?v=vCu5s80uAMQ) --and apparently the video is a hoax: http://news.nationalgeographic.com/news/2005/12/1219_051219_drunk_elephant.html But that article doesn't say anything about what's actually going on in the video--does anyone know how it was made? Were the animals just given alcohol and filmed stumbling around or was something else going on? Thanks! —Preceding unsigned comment added by 204.73.103.253 (talk) 00:52, 27 May 2008 (UTC)[reply]

Marula (see the article under its scientific name Sclerocarya birrea) is the source of berries that makes the delicious Amarula liquor. There was a film that showed animals getting drunk from the fermented fruit but was likely to be staged scenes. Also, elephants would need to eat a large amount to get drunk. -- Alan Liefting (talk) - 06:56, 27 May 2008 (UTC)[reply]
Sure, that was all in the national geographic article--I'm wondering if anyone knows what was REALLY going on. Who made it, how and why? —Preceding unsigned comment added by 204.73.103.253 (talk) 16:07, 27 May 2008 (UTC)[reply]
The same phenomenon features in the 70's South African documentary by Jamie Uys (which I recall seeing as a kid) called Animals Are Beautiful People. I can't quite imagine how they would have staged it. --Anonymaus (talk) 08:17, 1 June 2008 (UTC)[reply]

Tasting a meadow in butter

I normally eat margarine, but I bought some organic butter recently. I thought I could taste the grass and flowers of the meadow where the cows ate. Was this just my imagination or not? 80.0.110.30 (talk) 01:22, 27 May 2008 (UTC)[reply]

Flavor shows that what we call "taste" is a 2 part process involving olfaction and taste. Since the smell component are substances that can be "carried" in fat, it's not that far out. However it might just be a clever ploy by a flavorist selecting natural ingredients that have that effect. organic doesn't mean they are not permitted to use additives, as long as those are organic, too. There was an odd experiment they used to do in biology classes when I was a kid. I can't recall what went into the recipe, apart from peas. The end result tasted like strawberries, although there were none in there. You just had to blindfold people or the color would give it away. Although some flowers are edible, most meadow flowers and the grass would probably add up to a taste that would be rather not like the "idea" of meadow taste. --71.236.23.111 (talk) 03:28, 27 May 2008 (UTC)[reply]
If a cow manages to eat some wild garlic, the flavor shows up in, and spoils the milk. If a lactating female human eats strongly flavored food, the milk produced may be upsetting to a nursing infant. From these observations, it seems plausible that a "meadow" flavor might present in cow's milk and hence in the butter. Or it could be the power of suggestion. Edison (talk) 05:55, 27 May 2008 (UTC)[reply]
Milk flavour varies a lot according to season etc., a friend rang the milk company to ask about the different flavour. Polypipe Wrangler (talk) 11:08, 27 May 2008 (UTC)[reply]
Well, have you ever tasted grass and flowers from a meadow? If no, then how can you be sure that this is what the butter tasted like? If yes... why? Nimur (talk) 15:29, 27 May 2008 (UTC)[reply]
That's where olfaction comes into it. We are all familiar with the smell of grass and flowers. Scientists should read "I could detect the flavour..."--Shantavira|feed me 06:09, 28 May 2008 (UTC)[reply]
The organic butter that I buy is cultured, whereas most butter sold in the U.S. is not cultured. The cultured butter has more of a sour "tang" like yogurt, and a more complex flavor in general. If your butter is cultured, that would be a major factor in its taste. -- Coneslayer (talk) 11:32, 28 May 2008 (UTC)[reply]
See also terroir. arkuat (talk) 06:32, 2 June 2008 (UTC)[reply]

Blowing one's nose

Does blowing ones nose have any real known and scientifically measured effect on reducing the chances of getting a cold (or other infection), compared with people who only sniff the phlegm (?) up and do not blow? 80.0.110.30 (talk) 01:27, 27 May 2008 (UTC)[reply]

Well, I don't know about measured, but it makes sense. The mucous collects all sorts of things from the air, and it's probably better off to expel them rather than to take them into the system. --98.217.8.46 (talk) 02:04, 27 May 2008 (UTC)[reply]
I believe it's been shown that rinsing the sinuses with salt water does reduce the chances of catching a cold. It would make sense that blowing the nose would have a similar effect - removing microorganisms from the body, rather than moving them to the throat or elsewhere. -- Beland (talk) 16:14, 27 May 2008 (UTC)[reply]
Clearing your nose is something your going to do pretty automatically. It's like scratching an itch. There wouldn't really be any way of testing this. The type of experiment you're asking about would require that people not blow their noses, but if your nose is full of snot it's going to come out somehow: either you blow it, snort it into your nasopharynx (spit or swallow is your choice), or it's just going to dribble out. I think more than anything, once the mucus is there, it's just uncomfortable not to blow your nose. --Shaggorama (talk) 18:56, 27 May 2008 (UTC)[reply]
The problem with blowing one's nose is that sometimes it blows things into the sinuses. That's one reason I remember being stated for the results coming out ambiguously. (The article I remember predates the internet archives.:-( Salt water has been proven and we have a page Nasal irrigation. --Lisa4edit (talk) 19:11, 27 May 2008 (UTC)[reply]

You probably get this question all the time...

... but what would be the Latin name of a species of Woolly Mammoth found only in cathedrals?

Ta

Adambrowne666 (talk) 01:49, 27 May 2008 (UTC)[reply]

I assume you mean this cathedral not this one Cathedral. Sounds like an odd thing to happen with a Woolly mammoth find in any case. "Mammuthus primigenius" would be all of them.--71.236.23.111 (talk) 02:31, 27 May 2008 (UTC)[reply]
No, I mean the latter - and not just cathedrals, also large chapels and basilicas - I don't deny it's an odd thing to happen. Adambrowne666 (talk) 02:45, 27 May 2008 (UTC)[reply]
Given that the last Woolly mammoth died around 1700 BC, and cathedrals are decidedly AD, that's more than odd. Can you give a bit of context as to how the two would meet? And, for that matter, why the ones found in cathedrals would be a different species to those found elsewhere? Confusing Manifestation(Say hi!) 03:41, 27 May 2008 (UTC)[reply]
It's for a science fiction novel I'm working on in which some time periods have been mixed up - as for how ones found in cathedrals would be a different species to those found elsewhere, that's a good question, but would take too long to answer here - suffice it that it's a very weird science fiction novel. Adambrowne666 (talk) 03:55, 27 May 2008 (UTC)[reply]
It makes perfect sense to me. If, suddenly, woolly mammoths started turning up in cathedrals, we'd need to call them something. Due to my basic Latin knowledge, I can only suggest Mammuthus basilica or Mammuthus ecclesia. To be specifically a woolly mammoth, then perhaps Mammuthus primigenius ecclesia would be appropriate. The language desk might come up with something clever. Gwinva (talk) 04:10, 27 May 2008 (UTC)[reply]

I like it - Mammuthus primigenius ecclesia or basilica or basilicensis or something like that - thanks, Gwinva.

If it's a species the name should only have two parts, genus and species. When a third word is added, this indicates a subspecies. The basic rule is that if it interbreeds with other mammoths and produces fertile offspring then it's the same species as them, and if not, it isn't; but this is a simplification. --Anonymous, 23:58 UTC, May 27, 2008.
Yes, I assumed a sub-species of woolly mammoth when I offered the three name version (Adam said they were only found in cathedrals and churches, so they cannot be the same as a normal woolly mammoth. In any case, they can't mate and produce fertile offspring; they are separated by several millenia, after all. Gwinva (talk) 09:05, 28 May 2008 (UTC)[reply]
Primates are more often found in cathedrals than are mammoths. Edison (talk) 05:51, 27 May 2008 (UTC)[reply]
Had to think about it, then chortled, thanks, Edison. Adambrowne666 (talk) 12:09, 27 May 2008 (UTC)[reply]
Sounds like a cool book, can you tell me when it's done? Ziggy Sawdust 15:36, 27 May 2008 (UTC)[reply]

Thanks for saying so, Ziggy - I like your name btw - I'll let you know - will be a coupla years, though - I'll make a big announcement in the Ref Desk Talk Page. Adambrowne666 (talk) 10:53, 28 May 2008 (UTC)[reply]

Question

Can something that travels faster than light escape a black hole? Interactive Fiction Expert/Talk to me 05:59, 27 May 2008 (UTC)[reply]

If you can answer the question, "can something travel faster than light?", you may have the start of an answer. 63.224.79.202 (talk) 06:25, 27 May 2008 (UTC)[reply]
Particles cán escape from a black hole however: as the article on Hawking Radiation quotes: "Because Hawking radiation allows black holes to lose mass, black holes which lose more matter than they gain through other means are expected to evaporate, shrink, and ultimately vanish." —Preceding unsigned comment added by 87.67.39.188 (talk) 11:07, 27 May 2008 (UTC)[reply]
The particles in Hawking radiation are created just outside the event horizon - they're never actually in the black hole, so don't escape it. --Tango (talk) 19:52, 27 May 2008 (UTC)[reply]
I have no idea how tachyons would interact with black holes. It's an interesting question, though. I suspect they wouldn't be able to leave, either, but I'm not sure why not. --Tango (talk) 19:52, 27 May 2008 (UTC)[reply]
Since tachyons seem to experience time backwards, I'd say (based on the point of view) that it would moonwalk out of the black hole. 63.224.79.202 (talk) 04:29, 29 May 2008 (UTC)[reply]
Tachyons (by definition) travel on spacelike paths. Inside a (for the sake of simplicity Schwarzschild) black hole the time coordinate becomes spacelike and the radial coordinate becomes timelike, so inside the black hole tachyons would travel somehow like normal particles outside the black hole. Icek (talk) 14:55, 30 May 2008 (UTC)[reply]

Battery sizes

After reading the articles, I am a bit confused about what might happen if I were to use a C battery to power (for example) my mp3 player which normally uses a AAA battery. Since the C is rated for higher current, would that damage the mp3 player? Or would the set 1.5 voltage mean it would give me a longer play time? In other words, does the electric current depend on my device, or the size of the 1.5-volt battery I'm using? 63.224.79.202 (talk) 06:11, 27 May 2008 (UTC)[reply]

You can use any battery on your MP3 player as long as it is the correct voltage. The larger batteries can supply a higher current and for a longer time but the MP3 player will only draw the current that it needs. -- Alan Liefting (talk) - 06:59, 27 May 2008 (UTC)[reply]
To avoid any confusion: the "can supply a higher current" part is correct but irrelevant. The larger battery would be safe and would last longer. Of course, it would also be less convenient. --Anon, 00:00 UTC, May 28, 2008.
You can damage the mp3 player with two batteries placed in series (lined up one end to another). If you can somehow get your hands on a lower-voltage battery, the damage will be slower and less predictable. Imagine Reason (talk) 00:33, 28 May 2008 (UTC)[reply]
Minor nitpicking here but two batteries line in a series where the total voltage in the series is higher then the voltage expected by the device. If you get your hand on a significantly lower-voltage battery i.e. 0.75V or lower and place two of these in series, you're not going to damage your MP3 player since the voltage supplied will still be 1.5V or lower Nil Einne (talk) 15:28, 28 May 2008 (UTC)[reply]
If that mp3 player uses one AAA battery and you are going to use one C battery instead, the only problem i could imagine is that C battery will not physically fit in a place designed for AAA battery. If you connect it with wires (and duct tape on outside of device) it should work as good as AAA battery (+ last longer than AAA battery). -Yyy (talk) 07:28, 29 May 2008 (UTC)[reply]

Questions about acceleration due to gravity

I have two question about physics

(1)where will be the value of "g" high,at equator/poles/surface of earth.
(2)what are the number of molecules in kilometer of a gas.

thanks —Preceding unsigned comment added by 202.125.143.74 (talk) 07:00, 27 May 2008 (UTC)[reply]

(1) g will be the same anywhere on the surface of the earth (9.81ms-2. It will differ slightly because of difference in elevation and the fact the earth isn't a perfect sphere, but negligibly.
(2) A cubic kilometer? I can tell you that the number of molecules of 1 mole of a gas is 6.02E23, and 1 mole of a gas occupies 22.4 litres. Try working it out from there. 61.69.132.119 (talk) 07:51, 27 May 2008 (UTC)[reply]
(1) According to our earth's gravity article, variations can indeed be significant depending on the precision required. I'd think that differences on the order of 0.4% would be quite significant for say, missile launching. --hydnjo talk 08:07, 27 May 2008 (UTC)[reply]
I beg to differ. If you could get a missile engine large enough to fly in an orbital or sub-orbital path with thrust that could be reliably measured to with 0.4%, you'd be way better than any current system. I think there are many other variations (wind, engine efficiency, target location / trajectory), which introduce errors significantly larger than graviational perturbation. Of course the issue is moot because a modern launcher will use feedback control and some type of tracking (RADAR or GPS) to update its current position and correct for any error from desired trajectory, regardless of the cause... Nimur (talk) 15:36, 27 May 2008 (UTC)[reply]
(1) Largest effect on the apparent value of g seems to be the effect due to latitude, with a difference of about 0.5% between poles and equator. Altitude has a smaller effect (about 0.3% difference between sea-level and the summit of Mt. Everest); local topography, atmospheric density and gravitational influence of Sun and Moon have still smaller effects.
(2) Depends on the temperature of the gas, the pressure of the gas and its equation of state. If you can assume that the gas obeys the ideal gas law then this will simplify your calculation considerably. Gandalf61 (talk) 09:23, 27 May 2008 (UTC)[reply]

Car economy accuracy

How accurate are the fuel economy figures that cars report? My car has a little digital display that says I do around 50 miles to the gallon, and I can see this figure go down if I drive faster and up if I keep my speed around 65 miles per hour, but is it reliable? Of course the reliability could vary from one manufacturer to another, but I'd expect the major manufacturers will have been independently checked. — PhilHibbs | talk 10:33, 27 May 2008 (UTC)[reply]

I have not looked into this in depth but given the improving technology and accuracy of fluid quantity and distance measurement devices I would say there is a good accuracy. (if tyres are correctly inflated of course). -- Alan Liefting (talk) - 10:41, 27 May 2008 (UTC)[reply]
The average MPG display in both my Audi A8 and Dodge Caravan seem to correlate pretty well with the values I compute at the gas station (miles driven on this tank of gas versus gallons refilled). The instantaneous MPG figures are wildly varying, of course and only help to train you to be a more-efficient driver.
Atlant (talk) 14:09, 27 May 2008 (UTC)[reply]
Yeah- most cars will have some way of viewing "average" mileage instead of "instantaneous" or "average over the last 5 seconds" mileage. I'd generally expect this to be quite accurate on newer vehicles. On a related question.. I know the EPA has been trying to make their figures more accurate. Does anyone know a source for real-world results compared to EPA ratings? I bought a car a year ago that consistently outperforms EPA rating on the highway (even with a few passengers and AC on). The identical car has a lower highway rating for 08 than it did in 07. It sounds to me like (on this particular car, anyway) the EPA ratings are now unrealistically low instead of unrealistically high. Is this how it goes now, or does it vary from car to car? Friday (talk) 15:33, 27 May 2008 (UTC)[reply]
The EPA has just reformulated the way they require the manufacturers to test for and report the MPG ratings. I always found I could meet or exceed the old EPA ratings, at least for the particular American and European cars that I've bought so I am confident that the new rating system is "too pessimistic" for my driving habits. I was down at the Audi dealer looking at the stickers and even the little two-ish litre four-bangers are now stickered as getting lower highway mileage than I routinely achieve in my Audi A8.
Atlant (talk) 16:01, 27 May 2008 (UTC)[reply]
Most modern cars use electronic fuel injection, and as such have pretty acurate measurement as to the volume of fuel being used at any given second. (The car's computer needs this information as it adjusts the amount of fuel injected to match operating conditions.) The car also has rather accurate distance measurements through the (electronic) odometer. So as long as your EFI is operating correctly (and if it isn't, you should see a mechanic), and your odometer is correctly calibrated (e.g. you haven't changed your tire size), the miles per gallon should be relatively accurate. -- 128.104.112.147 (talk) 01:37, 28 May 2008 (UTC)[reply]

Chemistry project Help

I am a 10th grader. My Science teacher asked us students to make a working model on any topic of Physics and Chemistry but I'm confused on which topic to take. Can you please suggest some topics for me to make the projects. —Preceding unsigned comment added by 220.224.98.54 (talk) 14:30, 27 May 2008 (UTC)[reply]

In before "we don't do your homework". I can't help you on the project itself, however I can give you some ideas. You could try a Foucault pendulum or some such thing if you had enough time, or even just demonstrating that a pendulum's period remains constant. Another idea would be to mix baking soda and vinegar, demonstrating the reaction therein, or Mentos + Diet Coke, or any similar reaction. Ziggy Sawdust 15:15, 27 May 2008 (UTC)[reply]

(edit conflict) The problem with the Foucault pendulum experiment, is it takes quite some time to show a visible effect and most pendulums that are practical for a high school student to set up in school, will have decayed long before the effect can be seen. However, if you are still interested in the experiment, there are plenty of them set up in various places around the world. There are other experiments you can do with pendulums, such as showing the period is dependant on length and not amplitude, or the effect of drag on the decay of the pendulum's motion. Astronaut (talk) 15:51, 27 May 2008 (UTC)[reply]
How about a steam generator rigged up for electric generation? You can buy a small motor at your local electronics or hobby store, hook it up to a fan, and boil water underneath the fan to spin it. Nimur (talk) 15:39, 27 May 2008 (UTC)[reply]
I've seen students make models of trebuchets. Not a very peaceful idea, but probably fun if you're careful. --Allen (talk) 15:54, 27 May 2008 (UTC)[reply]
Cutting sheet glass under water with a pair of ordinary household scissors usually gets a couple of oohs and aahs. Make very sure to wear proper protective gear (gloves) and don't make uncontrolled movements. Those edges are sharp!!! Somewhat safer and easier to set up: Continuing from Nimur's steam idea, you could also put a couple of mirrors and plates above a steaming pot. Use different materials, temperatures and chemicals like e.g. oil, saltwater etc. on the surfaces and see what happens to condensation. (For a trick show and some laughs you can color the water and show that "the red water will condensate, while the blue doesn't. Just make sure to reveal the solution at the end, to catch the right grade from your teacher.)Lisa4edit (talk) 16:43, 27 May 2008 (UTC)[reply]
Cutting glass with a pair of scissors - are you serious? Do you have a link to a description or a video? I gotta check it out, sounds amazing. Franamax (talk) 18:10, 27 May 2008 (UTC)[reply]
It works. I could never get those glass cutters to work. You don't get quite as predictable a cut with scissors. I came across a demonstration once (must have been at some science fair, can't remember) and then tried it at home. You can also use a hand drill or a hand saw under water. But watch those edges. (3 guesses why I'm stressing that!) You tube has this [1] and this [2]. Lisa4edit (talk) 18:37, 27 May 2008 (UTC)[reply]
zOMG! Thanks :) My first guess about the edges would be that when the water is red, it's hard to see the demonstration, right? Kevlar gloves are in order. I found another one, how to cut a bottle with a string [3], also quite cool. If only they'd had youtube when I was a kid, I'd be long since dead :) Franamax (talk) 18:50, 27 May 2008 (UTC)[reply]
Even worse for the OP any sight of blood might make the teacher forget to assign a good grade. Back to ideas: I've only seen this done once, but if you're willing to give it a try it can be amazing. It is very difficult to pull off though. You'll need an aquarium, an old pair of headphones (Make sure you get sound out of one side. I don't think earbuds would work)and a balloon. Put the working "speaker" in a balloon (easier said than done!). Inflate the balloon and tie off the end and tie the string around a rock at bottom of the aquarium. Fill with water, add a layer of colored lampoil on top. Play music through the headphone and watch the movement in the water. @Franamax surviving childhood is entirely coincidental when even shoe-polish can become a hazardous material (Without you-tube, and no, I'll not elaborate the details, my backside still remembers 4 decades later, though.) As a relative once commented we should give up sending soldiers anywhere and just send our kids to go wreak havoc :-). Lisa4edit (talk) 19:55, 27 May 2008 (UTC)[reply]

tire/wheel diameter

Why wheel diameter of car, truck and tractor are different?

Why truck can't have small diameter wheels?

Is there any answer in context of radius of gyration? Neel shah556 (talk) 17:06, 27 May 2008 (UTC)[reply]

Bigger wheels are heavier and "waste" more energy. So a given vehicle will tend to have wheels large enough to do the job but not (much) larger. Also, when it comes to things like tractors, the muchlarger outside diameter of the wheel improves traction and ability to get over bumps in the surface or other obstacles. Friday (talk) 17:09, 27 May 2008 (UTC)[reply]
Tractors attempt to distribute their weight (and tractive effort!) over a large contact patch so that they don't either tear up or compress the soil over which they are rolling.
Trucks, meanwhile, use large tires no only for their larger load-carrying capacity but because, all other things being equal, a tire with a larger circumference will go farther before it wears out. To a first approximation, the rubber of the tire only wears when it is in contact with the road so a larger circumference leads to fewer "touchdowns" per mile which leads to more miles per tire. And then there's recapping/retreading...
Atlant (talk) 17:51, 27 May 2008 (UTC)[reply]

Thank you Atlant for your reply, i appreciate that inceased contact patch distribute the tractive force, but still it's not very clear to me that...

How larger diameter wheels possess more load carrying capacity? Do greater radius of gyration play any role in that?

Rather I doubt that largter dia wheel would be more prone to lateral buckling and cambering.

I mean why truck wheel can't be with smaller diameter and wider tread for the sake of increasing load carryin capacity and contact patch? Neel shah556 (talk) 09:50, 28 May 2008 (UTC)[reply]

I think that the principal reason tires get larger as the load-carrying capacity goes up is that the "contact patch" for a practical tire can only exert a certain force per unit area of the contact patch. Let's take the weight of a truck as 90,000 pounds (that's slightly high, but makes the math easy.) That weight is divided among 18 wheels so each wheel is bearing (about) 5,000 pounds. That 5,000 pounds must now be spread over a large enough area of the road surface so that it doesn't damage the road surface. On a hot summer day, macadam paving is pretty soft, so you want the weight spread over a pretty large area.
Meanwhile, we have a question of inflation pressure. What actually determines the area of the contact patch? To a first approximation, it's the inflation pressure of the tire. To continue our example, let's assume 50 PSI inflation pressure. That means that we need 100 square inches of "50 PSI" contact patch to support the 5,000 pound load on each tire. Now we don't want the tire flexing much to create the flat contact patch because flexure creates heat through frictional losses and heat shortens the life of the tire. This means that the tire must have a pretty large diameter so that 100 square inch "flat patch" doesn't represent much bending of the tire. So heavily loaded tires tend to be large tires.
You could use a much smaller tire with a much higher inflation pressure (much like some cars have miniature spare tires), but it gets harder and harder to build a higher and higher-pressure tire. And eventually, the PSI load on the surface of the road gets to be too high anyway and the tire sinks into the hot pavement.
Atlant (talk) 12:45, 28 May 2008 (UTC)[reply]

Unidentified concept car

Hello,

Does anyone have an idea of what this car might be?

Thank you very much in advance ! Rama (talk) 18:03, 27 May 2008 (UTC)[reply]

Based on the nearly inscrutable logo behind the car in the pic on the left, it seems to be a Fioravanti Hidra. (see also Fioravanti) --LarryMac | Talk 20:03, 27 May 2008 (UTC)[reply]
Brilliant, thank you so much! Rama (talk) 15:37, 28 May 2008 (UTC)[reply]

Plasma TV surface hot after use?

Is it normal for a Plasma TV to have be extremely warm after watching for a few hours? I'm talking about the actual screen; I can feel it radiate heat. When I hold my hand over the air vents on the top and back of the TV, it's relatively cooler than the screen is. --70.167.58.6 (talk) 21:49, 27 May 2008 (UTC)[reply]

Plasma screens are noted for their generation of heat, take a peek here. Fribbler (talk) 22:53, 27 May 2008 (UTC)[reply]
IIRC, they used to get *really* hot. I remember seeing some TV footage from the early 90s where a presenter decided to (for some reason) put his lips against a plasma screen live on air and ended up with his skin stuck to it... --Kurt Shaped Box (talk) 23:21, 27 May 2008 (UTC)[reply]
See plasma physics. Nimur (talk) 00:26, 28 May 2008 (UTC)[reply]


Early plasma panels back in the 1970's did not generate appreciable heat. Edison (talk) 06:58, 28 May 2008 (UTC)[reply]

Follow-up on Geographic feature

Thank you for your feedback on the meaning and scope of "geographic feature".

The article has been expanded to include your suggestions.

Please take a look to see if it is accurate and complete.

Have all types of geographical feature been included? Is anything missing?!

Are there any errors???

I look forward to your observations and suggestions.

The Transhumanist 00:05, 28 May 2008 (UTC)[reply]

Strange parrot behaviour - what's really going on?

Just found this YouTube video of someone's pet Green-cheeked Conure skulking around under a sofa and behaving rather strangely. A few people have commented on the video with suggestions as to what the parrot is doing - but does anyone here know for sure (there's someone here that owns Conures, right)? I've certainly never observed any of the psittacines I've personally owned over the years doing anything that even remotely resembles that. A pattern of behaviour limited to this particular species, perhaps? --Kurt Shaped Box (talk) 23:58, 27 May 2008 (UTC)[reply]

This is undoubtedly sexual behavior: the bird is masturbating. Why under the couch? Almost all parrots nest in cavities, and seek out similar locations when in breeding condition--quite often under the couch. Check out these sites: [4], [5], [6], [7]. Good thing that Wikipedia is not censored!--Eriastrum (talk) 18:24, 28 May 2008 (UTC)[reply]

Interesting. I'm far from being a bird novice - but it literally never occurred to me that this Conure might just be humping the floor and getting herself worked up... :) --Kurt Shaped Box (talk) 21:06, 28 May 2008 (UTC)[reply]

Eriastrum: you sure? the folks who posted the video seem pretty certain their bird is female. Would a female exhibit humping behavior like that? --Shaggorama (talk) 03:44, 29 May 2008 (UTC)[reply]
Yes, females do masturbate. Did you look at some of the links I provided? Especially take a look at [8].--Eriastrum (talk) 16:34, 29 May 2008 (UTC)[reply]

From my own experience with keeping birds, the females can be every bit as filthy, wanton and sinful as the males. :) --Kurt Shaped Box (talk) 22:57, 29 May 2008 (UTC)[reply]

Which just goes to show that we are even more like parrots than we thought!--Eriastrum (talk) 23:47, 29 May 2008 (UTC)[reply]


May 28

Can you make black holes move?

If you throw something at it... Like the sun, and throw it hard. Like 99.99% the speed of light, what will it do to the black hole? Will it affect the black hole's velocity? 65.41.92.123 (talk) 00:25, 28 May 2008 (UTC)[reply]

Yes, it would work the same way as any other object, assuming the other object would be able to absorb a star. In fact, absorbing any object would make it move, just more slowly. See conservation of momentum. *Max* (talk) 00:45, 28 May 2008 (UTC)[reply]
A gravitational tractor would likely be the most reliable way to move a black hole. (Although given the likely masses involved, it would take a long time and a lot of energy.) -- 128.104.112.147 (talk) 01:25, 28 May 2008 (UTC)[reply]

Magnetic Moment

In dipole, two equasions for the magnetic field are given; one describes it as a scalar, the other as a vector. They are

and

Setting them equal,

Dividing by common stuff and since the Dirac delta will gives zero,

Now, and , so . But,

These are not equal. Am I doing something wrong or is one formula wrong? Thanks *Max* (talk) 00:41, 28 May 2008 (UTC).[reply]

Isn't a function of r. For your example r , or 45°. Graeme Bartlett (talk) 02:47, 28 May 2008 (UTC)[reply]

Second equation is the one I remember since undergrad, it is correct. The first one, as you say quite correctly, gives B as a scalar, that is, gives an absolute value of B. Therefore, m should not be boldfaced in that equation. Now, as you know, absolute value of a vector is a square root of sum of squares of its orthogonal components. Let us take z-axis along m and x-axis in (m, r) plane. B has no y-component then. The second equation gives, for and r > 0, the following value of B components:

,
.

Root of sum of squares of the two is

.

Thus, the first equation is also correct, provided you replace the boldface m by m in it. Hope this helps. --Dr Dima (talk) 03:50, 28 May 2008 (UTC)[reply]

And this is why I shall not be studying physics at anything beyond 'A' Level :| Regards, CycloneNimrodTalk? 09:11, 28 May 2008 (UTC)[reply]
I'll unboldface m in the article. I calculated λ based on r. The opposite side is and the adjcent is , so shouldn't λ be ? Does the value of B mean that a point dipole fixed here by other forces here will orient itself perpendicular to the first dipole? I was not expecting that. *Max* (talk) 17:24, 28 May 2008 (UTC)[reply]
In the example you give, m = (1,0,0) and r = (1,1,1) / 31/2. Therefore, one gets . As you know, , thus, for you get , which means . Please check. As for your second question, I did not quite understand what you mean. If you ask whether there are points in space where B vector is perpendicular to m vector, then the answer is certainly yes, for any r > 0. --Dr Dima (talk) 02:32, 30 May 2008 (UTC)[reply]

Zero point energy

Is there any way to magnify the scalar forces? So that it works on large objects? Like make a car levitate or anything? 65.41.92.123 (talk) 02:13, 28 May 2008 (UTC)[reply]

I don't know much about this subject, but Zero_point_energy#Levitation_and_inertia looks relevant. --Allen (talk) 05:35, 28 May 2008 (UTC)[reply]
There's no such thing as a zero-point-energy force. The Casimir force is just an ordinary electromagnetic attraction between closely spaced neutral conducting plates. It's unusual in that you can compute its magnitude approximately from an argument involving zero-point fluctuations, but you can also compute its magnitude (more accurately) in the same way as any other electromagnetic force. See this paper. That said, you can levitate macroscopic objects electromagnetically—see magnetic levitation—so in effect the answer to your question is yes. -- BenRG (talk) 13:03, 28 May 2008 (UTC)[reply]
I find it interesting that the paper above contradicts the implications in the articles Zero_point_energy and Casimir_force that the Casimir force is strong evidence for the existence of zero point energy. Is it possible those two articles need to be rewritten to de-emphasize Casmir forces as being linked to zero point energy?
Also Ben commented above that there is no such thing as a zero point energy force. My understanding is that you can't extract work out of zero point energy, since it is the lowest possible energy state, but that zero point energy itself does exist as a strictly positive amount of minimal energy. So while you presumably can't extract work from zero point energy, the random quantum fluctuations of zero point energy will prevent any system from reaching an absolute zero energy state. Am I understanding all this correctly? 63.95.36.13 (talk) 20:44, 28 May 2008 (UTC)[reply]
Yes you are correct, zero-point energy is beleived to be real, I have read many books published by physicists that zero-point energy is the cause of the casimir force, along with multiple webpages including Wikipedia's, the article above is the first I have read that says the opposite. I would think that this one article is wrong instead of multiple on Wikipedia, others on the web, and multiple books.

Can BritaTM Containters Leach BPA?

Can BritaTM Containters Leach Bisphenol A? Are they made of polycarbonate? Are they made of a plastic that can leach BPA?68.148.164.166 (talk) 21:40, 21 May 2008 (UTC)[reply]

Can you find the plastic number marking on the container? 2 and 3 are safe, I think. Imagine Reason (talk) 00:43, 28 May 2008 (UTC)[reply]
I could not find the plastic number marking on the container.68.148.164.166 (talk) 06:55, 28 May 2008 (UTC)[reply]
Ask the manufacturer. Coincidentally, my brother forwarded this email from P&G: "PUR dispenser bodies are manufactured from an acrylic-based polymer classified as recycling code #7. PUR dispenser lids are manufactured from polystyrene, code #6. PUR dispenser filters are made from polypropylene, code #5, and also contain no BPA." I still won't use it, however, because it no longer filters flouride or even chlorine. Imagine Reason (talk) 02:38, 28 May 2008 (UTC)[reply]

height and alcohol

sir i want to know about can alcohol drink related with height increases. if yes how it work and if no so how some body heavy drinke and they become very tall and stong. —Preceding unsigned comment added by Dss sakti (talkcontribs) 09:40, 28 May 2008 (UTC) for any sugesion u mail me plz {e-mail removed)[reply]

I'm not aware of any study or claim saying drinking alcohol increases height. As to the second part of your question, if they drink beer they get a lot of calories. If they get enough exercise to burn those calories they'll get strong. If they don't burn the calories they get obese and may develop health problems.
We don't respond by e-mail, please don't include your e-mail on this desk. (You never know who might use it.) --71.236.23.111 (talk) 10:14, 28 May 2008 (UTC)[reply]
Are you referring to children (who haven't stopped growing) being heavy drinkers? I would expect it to reduce their final height, although I have no references for that. Also, drinking while pregnant can reduce the height of the baby - see Fetal alcohol syndrome. Drinking as an adult shouldn't have any effect on height - you've stopped growing by then. --Tango (talk) 10:17, 28 May 2008 (UTC)[reply]

Glow Sticks

I have two questions about Glow Sticks. 1. On the article, it says that the ester oxidizes. But, I don't quite understand how it oxidizes on a molecular level. Could you please show me the equation. So far, I only know the LEO and GER. 2. How do glow sticks manage to stay bright for long periods of time? In videos/ demonstrations, they only stay bright for a few seconds.

Thanks. 121energy (talk) 09:46, 28 May 2008 (UTC)[reply]

Glow stick and Cyalume give the chemical equations going from oxalylic ester to carbondioxide. --Stone (talk) 15:20, 28 May 2008 (UTC)[reply]

Re: Wheel diameter

(Moved up to join the original question.)

Subtance that increases sex drive?

I'm doing some research for a script. I wonder if there is any drug that makes people... well... more horny? Preferably without any elaborate side effects. It should be used as a kind of "love potion", meaning it should help one character persuade another to have sex with him. I realize there's probably no substance that makes you go crazy for sex without any other side effects, but anything even remotely close to that would be enough, it's not a scientific script so I can make things up a bit to make it fit. It would be nice to just have the name of a substance that is remotely close, to make it a touch more realistic. Thanks! 81.236.199.5 (talk) 11:12, 28 May 2008 (UTC)[reply]

The word you're looking for is aphrodisiac. There's a good list of alleged aphrodisiacs in the article. -- Coneslayer (talk) 11:27, 28 May 2008 (UTC)[reply]
For fiction you might also find the human portion of Pheromones interesting. Or you could spin an idea off this [9]--71.236.23.111 (talk) 14:39, 28 May 2008 (UTC)[reply]
Some dopaminergic drugs have alot of anecdotal evidence supporting 'horniness'. MDPV, methamphetamine#Sexual_behaviour, yohimbine and GHB are the most common ones I can think from the top of my head. --Mark PEA (talk) 19:36, 28 May 2008 (UTC)[reply]
Testosterone is well-known to increase libido. --Sean 15:34, 29 May 2008 (UTC)[reply]

Cleaning a dog skull part two

Here is the dog skull that I cleaned using the advice you gave here. What article can we enhance with this image?--Lenticel (talk) 12:08, 28 May 2008 (UTC)[reply]

How about Dog anatomy or Skull? Only thing is there are many pictures there already. Yours is a good pic, though. Fribbler (talk) 12:26, 28 May 2008 (UTC)[reply]
Good picture - I'd add it to dog anatomy, there are lots of pictures there, but none of a dog's skull! --Tango (talk) 12:55, 28 May 2008 (UTC)[reply]

Survivable?

I went to see this film a couple of days ago. In it, the hero hides in a metal box which is thrown through the air for several miles by an explosion, before it comes crashing to the ground, tumbles end-over-end, and eventually coming to rest. The door bursts open and our hero rolls out of the box apparently unscathed by his high-speed journey. Assuming the box is strong enough to retain it's shape, would a man actually be able to such a violent journey? Astronaut (talk) 12:13, 28 May 2008 (UTC)[reply]

I doubt it. I would expect him to be crushed by the g-forces on both take off and landing. To propel a person several miles would require massive acceleration. According to projectile motion, the maximum range is , assuming he travelled 5km, that gives a minimal initial velocity of about 224m/s (higher if he wasn't launched at exactly 45 degrees), assuming the explosion provides all of that speed in 1 second, that corresponds to an acceleration of about 22g. He'd probably lose conciousness, but may survive that. However, that same amount of speed needs to be lost when it lands, and that will be done in a fraction of a section (if the box retains it's shape, it means it didn't absorb the impact and would have stopped dead), resulting in much greater accelerations, almost certainly killing to occupant. (Imagine a car crash at 500mph with none of a car's usual safety features - not likely to walk away from that!) NB: I'm ignoring air resistance - for a heavy, compact box, it's negligable. --Tango (talk) 12:49, 28 May 2008 (UTC)[reply]
In the film, the box bounced across the ground several times before finally stopping; and I suppose there could have been some minor deformation of the outside of the box (ie. it didn't change the inside shape of the box, but could have been quite dented on the outside). Could the deceleration have been such that each bounce removed some of the velocity so our hero didn't get crushed on landing? Astronaut (talk) 13:34, 28 May 2008 (UTC)[reply]
Reality would have made for a really short movie. --LarryMac | Talk 13:26, 28 May 2008 (UTC)[reply]
Heh. Anybody see "Air America" with Mel Gibson, where the C130 (I think?) crashes in the jungle, and begins a long skid for about a mile over a couple of minutes before stopping, with (to me) pretty funny reaction from those on board? (It was a comedy, btw, I'm not being sociopathic). Gzuckier (talk) 15:11, 28 May 2008 (UTC)[reply]
Bouncing would require some temporary deformation - it works by deforming the object and storing some of the kinetic energy as elastic potential energy and then that elastic energy turns back into kinetic energy and the deformation rebounds. I'm not sure what that would do to the person inside - the total change in velocity is actually greater, since you go from +224m/s to -10m/s, say, which is a change of 234, rather than the change of 224 if you stop dead. That change probably takes place over more time, though, so the acceleration (which is what's important) would be less. --Tango (talk) 14:13, 28 May 2008 (UTC)[reply]
Stranger things have happened. WilyD 13:35, 28 May 2008 (UTC)[reply]
That's a lot less strange. He would have been falling about four times slower, plus his fall was cushioned by a handy glass roof, rather than being inside an unyielding metal object. Algebraist 13:42, 28 May 2008 (UTC)[reply]
A metal box is better then? Or sitting comfortably in a chair? WilyD 13:57, 28 May 2008 (UTC)[reply]
Actually, you make an interesting point - I neglected air resistance, since it's usually negligible for a box, but at those kind of velocities, it probably isn't any more. Terminal velocity for the box would probably be less than 500 mph, would would improve his chances, but only to the chance of surviving a fall from a plane without a parachute and not landing on anything soft, which is still pretty much zero. Falling while still in a plane (which seems to be the case with most of these miracle stories) would give you a much lower terminal velocity - there's far more drag on a large plane (even after breaking up) than on a person. --Tango (talk) 14:13, 28 May 2008 (UTC)[reply]
The plane weighs a lot more, though. Your terminal velocity is just a function of your density and cross-sectional area. At high speeds, drag is going to go like r2v2 and gravity will go like ρr3. If you're roughly spherical, your terminal velocity scales like vt ~ ρ½r½ ... density's probably irrelevent (you're 1, a plane with lots of airspace is probably ~1. So person strapped to a chair = person, person in aircraft depends on chunk size, but they're falling faster. WilyD 14:26, 28 May 2008 (UTC)[reply]
My very rough calculations say that the density of a fully laden 747 is about 200 50 Note to self: Diameter and radius aren't the same thing! times less than that of a person (based on Boeing 747#Specifications). They are designed to be a light as possible, otherwise they wouldn't be able to fly. Even broken up into a few pieces, it's going to be significantly less than a person, and terminal velocity will be significantly reduced. (Unless you're unlucky enough to end up in a nose dive, perhaps.) --Tango (talk) 15:01, 28 May 2008 (UTC)[reply]
Yeah, deceleration by a long series of bumps, rolls, disintegration, etc. looks spectacular, but will keep peak G forces down to survivable levels. Look for the various Youtube videos of the Michael Mcdowell Texas Nascar Crash from a couple of months back. Since the OP's guy in a box is not strapped in, he has to avoid impact with the inside wall of the box, but if it's spinning, centrifugal force could keep him "strapped down" figuratively and prevent him from fracturing his skull. Kids: don't try this at home. Gzuckier (talk) 15:08, 28 May 2008 (UTC)[reply]
I doubt it would spin reliably enough while bouncing. If it's small enough, he could wedge himself inside so he doesn't get shaken around. Even with all the bouncing, landing at terminal velocity in a metal box doesn't sound survivable to me without something to absorb the impact. --Tango (talk) 15:12, 28 May 2008 (UTC)[reply]
But if you are Indiana Jones, cinematic rules override kinematic rules! --Stephan Schulz (talk) 15:16, 28 May 2008 (UTC)[reply]
It's also probably worth noting that the amount of lead in a lead refrigerator would not have protected the hero in question from the thing in question that tossed him. Even if he had, improbably, survived he'd have probably gotten quick sick. --98.217.8.46 (talk) 19:34, 28 May 2008 (UTC)[reply]

I think the original poster is wrong to assume the metal box was thrown "several miles". As I understood the scene, it didn't move particularly far; it's just that the same thing that put it in motion also damaged the surrounding area, so it may have looked different. You don't see it flying through the air or anything. --Anonymous, somewhere in the real world, 00:01 UTC, May 29, 2008.

Actually, it was shown flying through the air. In fact, from what I remember, its first bounce was just in front of the army car which was driving away from the explosion. The car didn't make it, but the fridge carried on bouncing. Jdrewitt (talk) 08:50, 29 May 2008 (UTC)[reply]
Oh. Perhaps I remembered seeing what I expected to see, then. --Anon, 23:49 UTC, May 29.
you all seem to be forgetting that each time the box experiences a collision, its passenger is going to experience an internal collision. Our hero would've been broken and bloody after that trip, even if the box stayed closed and intact. Which it wouldn't have. --Shaggorama (talk) 03:41, 29 May 2008 (UTC)[reply]
I think the most ridiculous thing that has been lost in all of this (having just returned from a showing of said movie) is that he stands in the immediate aftermath of (what one can only assume is) a fairly dirty uranium-fission explosion, what looks like less than a mile from ground zero, with no long- or short-term effects on his health. The physics of that whole movie was (were?) shall we say, less than intelligent. -RunningOnBrains 05:38, 29 May 2008 (UTC)[reply]
I bet his hair was perfectly styled when he stood up too. That always happens to me when I get caught in nuclear explosions :) And I always get clean clothes in the next scene. Franamax (talk) 08:18, 29 May 2008 (UTC)[reply]
He was rinsed off afterwards though to remove any radioactive particles on the outside of his body. As long as he didn't ingest any of these particles then he would be ok. The largest fraction of gamma radiation is emitted in the very first milliseconds of a nuclear explosion, during which time he was in the lead lined fridge (why was the fridge lead lined?) which would have reduced the dose. Jdrewitt (talk) 08:31, 29 May 2008 (UTC)[reply]
Hmm.. it is only beta particles that are stopped by lead, and it would need to be a concrete fridge to at least slightly stop an A bomb. 86.153.37.241 (talk) 00:14, 2 June 2008 (UTC)[reply]
Gamma radiation will get through lead, however it will still be attenuated and so the dose will be reduced, whether this would be enough, probably not but it depends on the thickness of the lead and the exposure time. Jdrewitt (talk) 10:14, 2 June 2008 (UTC)[reply]

Homing distance of snails

Sorry - I didn't check responses to my question: How far can snails travel back to their usual habitat if you remove them from this? within the 4 days. Please would anyone who knows the answer repeat it for me? Thank you. Ruth 555Ruth555 (talk) 12:38, 28 May 2008 (UTC)[reply]

The previous question is archived here. No-one knew a distance, they suggested marking the shells before releasing them so you can tell if they come back. --Tango (talk) 12:52, 28 May 2008 (UTC)[reply]

Original Research is frowned upon here, but it is allowed if no one can come up with a reference. Why not find out and tell us? On day one, pull each snail off, mark it with a red X and gently relocate it 100m away. On day 2, if any snails have a red X you know they can go 100m, so use a green marker and try 200m. If none have made it back, try 50m. Anyway, you have to use different colors and symbols so that on day 3 you know whether the returnees were from day 1 or day 2, etc. Once you have run out of colors use a circle, etc. Ensure that the marks are non-toxic because the experiment ends when you get tired of it and eat them. -SandyJax (talk) 16:01, 28 May 2008 (UTC)[reply]

Original research is frowned upon only when describing it in articles. Experimentation is highly encouraged, however it needs to be described in a reliable source before it can be included in an article. It would make a great school science project, though. ~Amatulić (talk) 18:38, 30 May 2008 (UTC)[reply]

What is the instrument with the guy with his fingers in those beer glasses?68.148.164.166 (talk) 12:39, 28 May 2008 (UTC)[reply]

It's known as a Glass harp. --Tango (talk) 12:54, 28 May 2008 (UTC)[reply]
It's not clear why it is shown under the heading Plasmaphone though. Pfly (talk) 21:03, 28 May 2008 (UTC)[reply]
(edit conflict) That is NOT a glass harp. A glass harp is played by running your finger around the rim of the glass. In that picture, the musician clearly has his fingers in the water. Moreover, the water level is the same in each glass, so there would be no difference in pitch. My guess is the cords connected to the glasses aren't just lighting them up: I'm betting those fluids are charged. I can only speculate how the instrument works, but maybe by sticking his fingers in the liquid, he changes its conductance properties, or cause a current to pass through his body changing the conductance of the system....I can only guess. In any event, it's probably some sort of fancy electronic instrument or user interface -- Shaggorama (talk) 03:26, 29 May 2008 (UTC)[reply]
Good point. I'd assumed his fingers were inside the glasses simply because he was getting them wet, and he wasn't actually playing at that instant. I hadn't noticed the water levels were all the same - there must be something more going on (presumably to do with the wires, as you say). --Tango (talk) 13:38, 29 May 2008 (UTC)[reply]

Current, volts, Ohm's law...

Ok, I think after all this time I still do not really have a good grasp of what current is (and I've read the articles).

We normally rate batteries by their voltage. So a battery might have 1.5 Volts, say. If we connect a resistor across the battery's terminals, a current goes through it. That current can be calculated by Ohm's Law: I = V/R. So if we have a 10 ohm resistor, we have 0.15 Amps coming out of the battery, right?

If we have a resistor which is 0.1 ohms, we have 150 amps going through the resistor. So I guess we can keep going up to nearly infinity if we keep making the resistance smaller, is that right? (Except eventually the battery will run out.)

However, when I look at solar cells, they give both a voltage and a current (or sometimes they just give a current). So I might have a cell that says it provides 1 amp. So if I connect a resistor across it, is the voltage of the cell going to change? The current? I assume that Ohm's law has to be maintained?

Why are solar cells and batteries different? Can batteries produce any current they want, and cells can only produce a fixed current? Or is it a maximum current? What happens if I have a 5V solar cell connected across an 0.001 Ohm resistor? What is the current?

Thanks! — Sam 14:48, 28 May 2008 (UTC)

Ah; the catch is (there's always a catch) that batteries and solar cells are not ideal voltage sources, nothing being ideal, the way they are often simplified as. In the real world, of course the battery/solar cell has its own internal resistance; everything does. So, if your 1.5 volt battery has a 1 ohm internal resistance, then even if it is short circuited, it's not going to deliver any more than 1.5 amps. Same for the solar cell. Same for transformers, anything in the real world; there's always an internal resistance which needs to be factored in in some cases. Of course, also there's internal capacitance and inductance which may need to be accounted for under some conditions; that's why electronic engineers who understand real things not just ideal things do a better job. Gzuckier (talk) 15:01, 28 May 2008 (UTC)[reply]
(See also internal resistance). TenOfAllTrades(talk) 15:23, 28 May 2008 (UTC)[reply]
If you'll allow me to expand on Gzuckier's point, we normally design electronics so that our "voltage sources" are operating pretty close to the regime of being an "ideal" voltage source. That is, while a battery might deliver (say) 5 amps into a dead short, it's commonly used in equipment that's drawing far less current, say 0.25 Amps. There are three reasons for this:
  1. It makes the circuitry a lot easier to design if you can mostly ignore the internal resistance of the voltage source (battery).
  2. It would be wasteful if the battery were internally dissipating a large share of the total power that was being consumed from it. To a first approximation, if you tried to operate our hypothetical "5 amp" battery with a 2.5 Amp load, roughly half the power being produced from the battery's chemical reaction is being dissipated in the load and half the power within the battery itself. The power dissipated within the battery does no useful work.
  3. The internal resistance of a battery rises as it ages. If you design your circuit to consume a lot less current than a new battery is capable of, an old, decrepit battery may still be capable of powering the circuit.
Hydraulic analogies are often useful in this case. A 1.5 volt battery might be compared to a water tank 1.5 metres off the ground. That tiny AAA cell represents a tank of 1 litre capacity. That big D cell represents a tank of 20 litres capacity. The bigger tank also has a proportionally-larger shut-off valve on the tank's discharge connection. Now, connect a drainage pipe to the tank. For a very skinny pipe, both tanks will provide adequate water pressure but the AAA-sized tank will run out of water twenty times as fast as the D-size tank. Now connect a bigger discharge tube to the tanks. For the tiny tank, the tank's discharge valve will limit water flow into the big pipe and there'll be very little pressure in the discharge pipe. But for the D-sized tank, its larger discharge valve can still provide full water flow into the discharge pipe and full water pressure. Batteries (and solar cells and most other voltage sources) work just like this.
Atlant (talk) 17:28, 28 May 2008 (UTC)[reply]
Ok, that makes a little sense. However, I still don't quite understand why solar panels are rated just by their amperage. Is a "one amp" panel describing the current that would be delivered into a short? — Sam 20:49, 28 May 2008 (UTC) —Preceding unsigned comment added by 63.138.152.238 (talk)
It's more likely a roughly defined point at which the internal resistance can be considered negligible, and thus at which the cell is suitable for powering a given application. As Atlant notes in the 5-amp (short current) battery example, it wouldn't make sense to use it in a 2.5 amp application, either. — Lomn 21:57, 28 May 2008 (UTC)[reply]
I don't know about the case of solar cells, but normally when both a voltage and an amperage are quoted the voltage is an estimate while the amperage is a maximum. E.g. a 240V 13A wall socket will maintain a potential difference of about 240 volts and you shouldn't try to draw more than 13 amps from it or something bad will happen (in this case a tripped circuit breaker). -- BenRG (talk) 23:27, 28 May 2008 (UTC)[reply]
A battery or solar cell could be characterized by the open circuit (no load voltage), which will be the highest possible output voltage, higher than seen when it is supplying current, and the short circuit current, which is quite high (several amps) even for an AA alkaline cell. But such a high current would limit the useful life to a very short period for a battery, and the voltage would drop dramatically. Another rating system would be so many amps (or milliamps) at a certain output voltage for normal or optimal operation. For example, a 200 Watt Kyocera solar panel [10] specifies that for max power output, the current should be 7.61 amps and the voltage should be 26.3 volts, the product of current times voltage equalling 200.61 Watts,which implies a total external circuit resistance including the load of 3.46 Ohms. but says the short circuit current is 8.21 amps and the open circuit voltage is 32.9 volts. The power under short circuit conditions would be 8.21 amps times zero volts, or zero Watts. The power for open circuit conditions would be 32.9 volts times zero amps, or zero Watts. For conditions close to short circuit or open circuit, the output power would be nonzero but far smaller than the 200 Watt maximum. Edison (talk) 02:41, 29 May 2008 (UTC)[reply]

solar power

i want get the details of solar power process and its panel types with photo graph & the details of batterys used to store power with maximum capacity and its configration details

if provide.any source or call center to discuss about said details —Preceding unsigned comment added by 124.125.42.50 (talk) 15:11, 28 May 2008 (UTC)[reply]

I would think that solar energy would be a good place to start and then, when you have specific questions, ask again. -- kainaw 19:08, 28 May 2008 (UTC)[reply]

Homing Instinct of snails.

Thank you for responding to my latest enquiry. The idea of nail varnish is a good one. Please be patient with my lack of expertise on this site. I'm aware that this is a response, rather than another question, but am not sure how to slot this comment into the correct place! As a newcomer, I'm enjoying the site immensely! What a variety of different subjects - I've spent hours reading through the topics instead of getting on with my own work - I'm in process of writing a novel. Ruth555Ruth555 (talk) 15:21, 28 May 2008 (UTC)[reply]

I answered the question above. -SandyJax (talk) 16:13, 28 May 2008 (UTC)[reply]
Hello Ruth - we're glad you're enjoying yourself. When you want to add to an existing section, just click on the "edit" link on the right side of the line that contains the section title. --LarryMac | Talk 16:19, 28 May 2008 (UTC)[reply]

Chemistry: What exactly is produced by immersing steel in salt water?

I know that rust is produced, but what gas is created?

I've noticed bubbles continually forming. Is it chlorine? —Preceding unsigned comment added by 24.81.198.165 (talk) 20:11, 28 May 2008 (UTC)[reply]

It's got to be hydrogen. Rust is a mix of iron oxides. Since there isn't enough dissolved oxygen in water to rust things very fast, the water must be dissociating into hydrogen and oxygen. The oxygen combines with the iron in steel to form rust, and the leftover hydrogen atoms combine to form hydrogen gas. --Carnildo (talk) 20:45, 28 May 2008 (UTC)[reply]

Ethyl Alcohol

Hello. Is C2H5OH a hydrocarbon? When burning, is the chemical equation: C2H5OH + 3O2 → 2CO2 + 3H2O? Can this reaction possibly be double displacement? Thanks in advance. --Mayfare (talk) 21:21, 28 May 2008 (UTC)[reply]

Have a look at Ethanol, Alcohol and Metathesis reaction (double displacement). Jkasd 21:41, 28 May 2008 (UTC)[reply]
You should also take a look at combustion. bibliomaniac15 04:02, 29 May 2008 (UTC)[reply]
Hydrocarbons contain hydrogen and carbon only. Ethanol contains oxygen so is not a hydrocarbon. 86.153.37.241 (talk) 00:16, 2 June 2008 (UTC)[reply]

Terminal Velocity

This French guy has just recently failed in an attempt to float his helium balloon at 40,000m and then jump out, free-falling for around 15 minutes before opening his parachute. In every article I have read about this, it says at 35,000m he would have been expected to break the sound barrier. Is this possible? I know at that height the air is relatively thinner and there would therefore be less friction, but surely his terminal velocity would not rise from c. 150mph (nearer the ground) to over 750mph? Also, would there be a sonic boom and what effect would this have on him, considering he wouldn't be in the safety of a jet-plane?--ChokinBako (talk) 21:26, 28 May 2008 (UTC)[reply]

I can't comment on the specifics, but one thing people sometimes forget is: the only reason a falling object has a terminal velocity at all is air resistance. So, if the air resistance almost goes away, the resulting speeds could be very high. Friday (talk) 21:30, 28 May 2008 (UTC)[reply]
Also, the speed of sound varies with temperature. Lower temperatures at high altitude decrease the speed of sound. See sound speed gradient for a discussion of this phenomenon. — Lomn 21:47, 28 May 2008 (UTC)[reply]
Indeed 150mph is not an accurate terminal velocity near the ground. According to Terminal velocity an experienced skydiver can achieve about 200 mph by optimising body position (of course an experienced skydiver wouldn't want to be at that speed to near the ground, but hopefully people get what I mean). According to [11] the record is 321MPH without any special equipment and Joseph W. Kittinger achieved 619mph Nil Einne (talk) 23:08, 28 May 2008 (UTC)[reply]

TECHIE COOLIE

Who is a TECHIE COOLIE & Why are they so called?117.197.240.153 (talk) 21:44, 28 May 2008 (UTC)[reply]

It could be a racial slur about someone who is a "techie". See techie and coolie. Friday (talk) 21:50, 28 May 2008 (UTC)[reply]
Alternatively, it may derive from the use of "coolie" to indicate a brute-force laborer. That would still leave it a derogatory remark, though perhaps not an outright slur. In any event, Google shows very little use of the phrase, so there's not likely a definitive answer. — Lomn 21:54, 28 May 2008 (UTC)[reply]
It could be a derogatory remark about the techie's employer, saying that they treat their workers like coolies. That seems the most likely interpretation to me, but of course I'm guessing. --Anonymous, 00:05 UTC, May 29, 2008.
Perhaps he works next door to the Code monkey. Edison (talk) 02:22, 29 May 2008 (UTC)[reply]

Water + grease fire: Why so much energy released?

RE: http://www.youtube.com/v/kS9inNW4KS8 -- Why is so much energy released when water is added to a grease fire? I'm wondering whether the combustion of the fuel in an internal combustion engine could be similarly augmented, by injecting some water into the cylinder, either with the fuel or separately, before or immediately after ignition. --Nonlocal (talk) 21:58, 28 May 2008 (UTC)Nonlocal[reply]

All that's happening is that the water is (very quickly) brought to a boil by the hot oil. So, it bubbles and expands and the hot grease goes everywhere, and the fire spreads. There is such a thing as water injection in automotive technology (see Water injection (engines), but it's about cooling, not getting energy out of the water. Friday (talk) 22:24, 28 May 2008 (UTC)[reply]
Man, what is it with the British and their very disturbing PSAs! Yikes! --98.217.8.46 (talk) 01:23, 29 May 2008 (UTC)[reply]
Disturbing = unlikely to be forgotten Theresa Knott | The otter sank 06:35, 29 May 2008 (UTC)[reply]
I think my main confusion is that the British seem to think their PSAs should actually be useful rather than just hollow ad campaigns used to write-off a tax deduction or something along those lines. ;-) American PSAs are almost always totally toothless, laughable, poorly designed and poorly executed. --98.217.8.46 (talk) 15:27, 29 May 2008 (UTC)[reply]
That's a pretty awesome demonstration in that video. I think the fire grows so rapidly because the explosive boiling of the water conitnually throws up grease because it's contained in the pot (and there's also probably an unusual amount of grease in the pot). Normally, grease fires don't result from pots of grease like that; usually, the flaming grease is in a pan and there is less of it. In that scenario, adding water spreads the fire around instead of making it grow like that. But in any event, don't put water on a grease fire! --Shaggorama (talk) 03:13, 29 May 2008 (UTC)[reply]
A kilogram of TNT releases less enery than a candle of 1kg, but it looks different, becaus the candle takes hours and the TNT far less than a second for the reaction.--Stone (talk) 05:25, 29 May 2008 (UTC)[reply]
The paper, On the interaction of a liquid droplet with a pool of hot cooking oil, Short communications, by S.L. Manzello, J.C. Yang and T.G. Cleary (Fire Safety J 38 7 (2003), pp. 651–659), notes that such occurrences of rapid boiling of water "have been termed vapor explosions, explosive boiling, or rapid vapor explosions." --Nonlocal (talk) 16:39, 29 May 2008 (UTC)[reply]

Conservation laws and space-time symmetry

As I understand the modern conception of physics, conservation laws arise because of symmetries in the laws of physics (Noether's theorem, I think). Conservation of momentum is the result of the universe being symmetric under spatial translation, conservation of angular momentum is due to spatial rotational symmetry, and the conservation of energy is due to time translational symmetry. However, we also have the concept of space-time, where there is no distinction between space and time (after accounting for a few -1's, i's, and c's). So if we have conservation of angular momentum due to rotational symmetry in the x-y plane (a rotation that occurs between two spatial axises), is there a conservation law for rotation in the x-t plane? (That is, a rotation that occurs between a spatial axis and the time axis.) - How about for rotations and translations involving the other 6+ dimensions proposed by string theory? -- 128.104.112.147 (talk) 22:32, 28 May 2008 (UTC)[reply]

The best way to think about it is to consider the metric. It will be a function of your coordinates, (t,x,y,z), say. If it doesn't depend on one of them, you have a conserved quantity (momentum in that direction, roughly speaking - see Killing field for more details). If you use different co-ordinates, for example, spherical coordinates, you may find that the metric doesn't depend on one of the angles, that gives you conservation of momentum in the direction of that angle - angular momentum, in other words. If you choose a coordinate system in which one of the coordinates is a rotation in the x-t plane, I imagine you would find that the metric does depend on that coordinate (those -1's, i's and c's do need to be accounted for!), so you don't get a conserved quantity. I'd have to find pen and paper to work it out for sure. The extra dimensions of string theory would definitely allow for extra conserved quantities, but if you ever actually have Killing fields in those directions, I don't know. --Tango (talk) 23:01, 28 May 2008 (UTC)[reply]
The conserved quantity associated with rotation around the origin in the xy plane is x py − y px, i.e. the angular momentum around the origin in the xy plane. This holds for any pair of perpendicular axes, even if one of them is timelike. So the conserved quantity associated with rotation in the xt plane is x E − t px. I'll let you work out what that represents. Note that it depends explicitly on t, unlike the conserved quantities people normally talk about, but that's not a deep difference—you just plug in different values of t at different times.
But the laws of physics are not symmetric with respect to ordinary rotation in the xt plane; you need a Lorentz transformation instead. I'll let someone else figure out what the corresponding conserved quantity is, though. —Ilmari Karonen (talk) 22:16, 29 May 2008 (UTC)[reply]
The Lorentz symmetries give rise to conservation of four-momentum, surely? Algebraist 22:26, 29 May 2008 (UTC)[reply]
I don't know whether the Calabi-Yau manifolds that describe the extra six dimensions in string theory have any symmetries that lead to conserved quantities. In Kaluza-Klein theory I think that the conserved quantity associated with translation in the fifth dimension is the electric charge. -- BenRG (talk) 00:19, 29 May 2008 (UTC)[reply]


May 29

electrons and perpetual motion

To start off, I am a 17 year old high school student that has just completed his first year of high school-level physics. I'm pretty smart (at least I like to think so), but if this question can't be answered with vocabulary and ideas that someone of my edjucation level will be able to understand, that's ok. The answer to this question is probably in Wikipedia somewhere, but in my reading I've only seen equations and explanations that I don't think I'll ever understand until sometime through college.

Question: If there is no such thing as perpetual motion, how can electrons keep moving around the nucleus of an atom? My guess is that at the level we're talking about, there's not really anything like friction in the common sense, so there's nothing to act against the electrons after they are moving, but then where does the energy come from to move them in the first place?Hypershadow647 (talk) 00:03, 29 May 2008 (UTC)[reply]

The motion of electrons in an atom is perpetual motion in the ordinary English sense but not in the technical sense used in physics. A "perpetual motion machine" in physics is a system that violates the first law of thermodynamics (a "free energy machine" or "perpetual motion machine of the first kind") or the second law of thermodynamics (a "perpetual motion machine of the second kind"). The electrons in an atom don't gain or lose energy or entropy (at least not without interacting with the outside world), so they don't violate either of those laws. -- BenRG (talk) 00:39, 29 May 2008 (UTC)[reply]
You'll be happy to know that it's not a dumb question at all—in fact, it was a major objection to the original Rutherford atomic model, that the electrons seemed to whip around without losing energy, and if you imagine them behaving like any other gravitating object (like a planet) you'd expect that not to work out. Bohr's great insight, the insight which set off quantum mechanics (as distinct from the original quantum theory by Planck and Einstein), was that if you take for granted that the electrons can do that in specific orbitals, that they are not in "motion" in the same way that we understand it to be on a macroscopic level, then it all works out perfectly well mathematically and according to experiment, even if it doesn't make as much conceptual sense (this is the kernel of what would become the Copenhagen interpretation of Bohr and Heisenberg). Einstein thought this approach to physics was repulsive—to come up with answers that didn't make any intuitive sense, as he saw it—but Bohr's method won out in the end, and much of quantum mechanics is about taking for granted certain properties of the quantum world which are not shared by the macroscopic world. Why don't electrons behave like billiard balls, is essentially your question, and the answer is simply because the laws that govern billiard balls don't apply on that scale. --98.217.8.46 (talk) 01:10, 29 May 2008 (UTC)[reply]
Whoa, whoa, whoa. Bohr's atomic model did help inspire quantum theory, but it was completely wrong. Real electrons are nothing like the electrons in Bohr's model. They don't even have the angular momenta that Bohr thought they did. Bohr thought the angular momenta were 1, 2, 3, ..., but in fact they're 0, , , .... I think it was little more than dumb luck that his model gave the right values for the hydrogen spectrum (which could be measured to confirm it) despite having the wrong values for the angular momenta (which couldn't be measured). And quantum mechanics does explain the quantization of the orbits, it doesn't simply assume it the way Bohr's model did. Schroedinger's equation doesn't have any quantization built in; you have to solve it before you discover that the electron orbits are quantized.
There are two things that keep the electrons from dropping into a state of zero energy: the uncertainty principle and the exclusion principle. They're both hard to explain. The electron can't just fall into the nucleus because the nucleus is very small; that would give the electron a well defined position, which would mean that it would have to have a very uncertain momentum, which would quickly propel it out of the nucleus again. The "ground state" is a compromise between position uncertainty and momentum uncertainty. The exclusion principle comes into play when there's more than one electron in the atom. Without the exclusion principle they would all fall into the ground state, since systems "like" to have the lowest energy possible. With the exclusion principle you can only have one electron in each state, so some of them have to be in higher-energy states with larger angular momenta (meaning they're actually orbiting the atom, unlike the ground state electrons which have an angular momentum of zero and could be said to not be moving at all). -- BenRG (talk) 04:00, 29 May 2008 (UTC)[reply]
Bohr's approach was but one little way down the path of quantum mechanics and marked a distinct break with how things were done previously. That's the entire point I was making—not that the Bohr model ended up being exactly correct, but that the question is tried to answer was exactly the same one, and the methodological approaches used started QM down its rabbit hole. You don't get Copenhagen without the Bohr atom. You will note that I spoke specifically of Bohr's method. --98.217.8.46 (talk) 15:46, 4 June 2008 (UTC)[reply]
If the electron were a ball bearing circling inside a hemispherical bowl, or a weight tied to a string circling around the point of suspension, friction and air resistance would cause it to drop gradually to lower and lower levels. The reason electrons do not show such a gradual decay of energy level, as it was explained to me back when, is that even taking it as a sort of standing wave, it might be expected to bleed off energy like an electromagnetic wave circling on a loop of conductor. The quantum effect is the theory or observation that it cannot drop gradually to imperceptibly lower energy energy levels as the circling weights could in the mechanical analogies. Only a quantum of energy can be absorbed or emitted, changing it to a higher or lower energy level, thus no slight gradual decay of energy level is possible.Edison (talk) 02:20, 29 May 2008 (UTC)[reply]
It's true that only a quantum of energy (a photon) can be absorbed or emitted, but the energy of a photon can be arbitrarily small, so in effect the energy that can be emitted isn't quantized at all. So this can't explain why the electron orbit doesn't decay. -- BenRG (talk) 04:00, 29 May 2008 (UTC)[reply]
However, the stable energy states around a nucleus are quantized as well, and this imposes the appropriate restriction on the possible photons that can be emitted. Confusing Manifestation(Say hi!) 04:12, 29 May 2008 (UTC)[reply]
I don't know about our 17 year old OP, but reading through this thread I'd feel confused (and not just in manifestation). So can we explain it in understandable terms without sacrificing enough to run into my favorite Feynman quote?--71.236.23.111 (talk) 16:57, 29 May 2008 (UTC)[reply]

It may be helpful to the OP that a "perpetual motion machine" is essentially something that can perform work indefinitely without an external source of power. Electrons moving around a nucleus, or moons orbiting a planet orbiting a sun, are perpetually in motion, although there is no energy expenditure to maintain that motion. ~Amatulić (talk) 18:34, 30 May 2008 (UTC)[reply]

antioxidants to delay ageing and damage of cells

what combinations work and do they¿ i know you cant give medical advice, but what does science say so far about the effectivity of them¨¿thnx guys —Preceding unsigned comment added by 75.69.26.8 (talk) 03:20, 29 May 2008 (UTC)[reply]

The empirical evidence to date has been quite disappointing; the benefits are largely theoretical rather than demonstrated. - Nunh-huh 05:07, 29 May 2008 (UTC)[reply]
While antioxidants are great at scavenging damaging free radicals generated via metabolism, which can denature proteins and injury other macromolecules such as nucleic acids and lipids, there is no conclusive evidence that they prevent general cellular senescence. See telomeres. Wisdom89 (T / C) 06:45, 29 May 2008 (UTC)[reply]

Career

What jobs and career options are there in relation with animation? —Preceding unsigned comment added by 117.194.225.94 (talk) 03:55, 29 May 2008 (UTC)[reply]

If I had to surmise, I'd say a viable option would be 3D video game development. Wisdom89 (T / C) 06:42, 29 May 2008 (UTC)[reply]
There are tons of video animation companies for everything from bad car commercials to popular children's movies. -- kainaw 12:57, 29 May 2008 (UTC)[reply]
There are quite a few unique ones, too. If you like kids you can run animation workshops at museums or fair events, for one. What job you can find and end up in might not be the same as the "standard" job description. And even that can differ from one place to another. --71.236.23.111 (talk) 16:35, 29 May 2008 (UTC)[reply]

Thermo Pond Epoxy

Hi. I saw a '3rd Rock From the Sun' episode where mary albright puts this chemical on dick solomon's desk, because earlier they had been playing impractical jokes on one another. Dick comes in and sits at his desk. as soon as he puts his hands on the desk, they are instantly stuck. Then he uses too much force to get his hands off and he accidently puts his head on the desk and then is stuck to this position. My question is: Can this chemical really do that sort of thing, paste objects together that quickly?Jwking (talk) 05:32, 29 May 2008 (UTC)[reply]

Yes, some types of superglue can. There are loads of documented cases of people putting it on toilet seats, glasses etc. Not a very sociable think to do. If you put it on a non-porous surface skin will bond almost instantly. There have also been eyeball to eyelid and similar in accidents. I'd stick to clingfilm for jokes... --BozMo talk 06:22, 29 May 2008 (UTC)[reply]
Since detaching the stuck human can require medical assistance this way exceeds funny. The doctors and lawyers will laugh all the way to the bank, though. (Neither legal nor medical advice)--71.236.23.111 (talk) 16:21, 29 May 2008 (UTC)[reply]

Brain + Sex

What part of the brain becomes more active during sexual arousal? Or at least, what part controls that. --68.90.143.120 (talk) 06:30, 29 May 2008 (UTC)[reply]

The Raphe nucleus and serotonin are thought to be involved. Wisdom89 (T / C) 06:35, 29 May 2008 (UTC)[reply]
Well, speaking as a guy, it's pretty clear your entire brain goes on vacation. Gzuckier (talk) 20:47, 29 May 2008 (UTC)[reply]

Under a burning night sky

I'm curious why the sky is orange during nighttime. First of all it was 10 pm so I discount the sun. It is relatively cloudy and rainy during the night and the clouds are actually the ones that is a dull orange. I live in Manila so the air pollution is strong but I can't account fo r the orange hue as we use fluorescent lamps here rather than oil or fires.--Lenticel (talk) 06:58, 29 May 2008 (UTC)[reply]

Sodium street lamps? (See the section on light pollution) 81.174.226.229 (talk) 08:41, 29 May 2008 (UTC)[reply]
Also the article on Light pollution Jdrewitt (talk) 12:00, 29 May 2008 (UTC)[reply]
hmm... I think you're right. It is Skyglow. No wonder this never happens when I'm in the provinces. --Lenticel (talk) 12:38, 29 May 2008 (UTC)[reply]
Hi. This happens here too, where I live in Southern Ontario. The streetlights cause the clouds to turn orange, and this can cause it to reflect into my bedroom window, where there are no nearby streetlights, so much so that I can see penumbral shadows of objects and even read printed text in the dark (but just barely and only with night vision adaptation caused by pupil expansion). When there are no clouds, or when the clouds are high, the light isn't strong enough to be reflected. However, if there is say a snowstorm, the light will illuminate the nearby snowflakes the way the sun illuminates dandelion seeds flying on a cool, windy, clear spring day. I've also noticed that the far away cloud bases sometimes appear darker than the nearby clouds, perhaps escaping the light pollution. Hope this helps. Thanks. ~AH1(TCU) 17:16, 31 May 2008 (UTC)[reply]

Power Generators

I was wondering how efficient generators are at converting mechanical energy from the rotating magnet to electrical energy? 61.69.132.119 (talk) 07:32, 29 May 2008 (UTC)[reply]

I think it's about 80-85%, which is from actual, but half-remembered, research. If someone doesn't give the definitive answer, I'll try and get back to you. It's definitely quite high, because the overall efficiency is about 40%, with most of the losses at an early stage, I think in the production of steam. 130.95.106.128 (talk) 11:25, 29 May 2008 (UTC)[reply]
I second the 80-85%, but can't quote a good source either. I know the figure from adding up losses for overall systems in which case it would be stated as a 15-20% loss. --71.236.23.111 (talk) 17:05, 29 May 2008 (UTC)[reply]
Surely it is higher than 80 to 85%, given that in 1879 Thomas Edison's "Long legged Mary Ann" generator achieved 82% mechanical to electrical efficiency, a sizeable increase from the 40% of earlier generators, with his reasearch team over the next decade achieving 90% efficiency [12] . If present day 1000 megawatt utility baseload generator were 15% to 80 20 % inefficient, it would get heated up by 150 to 200 megawatts, which seems unlikely. [13] cites 93% to 97% efficiency for small hydro generators. [14] says a wind generator has achieved 98% efficiency. [15] cites an 800 megawatt generator's efficiency being increased from 98.93% to 99.01% efficiency via stator rewinding and other improvements. I wonder if the lower efficiency deducts the auxiliary equipment such as coal handling, plant lighting, pumps etc? Edison (talk) 19:12, 29 May 2008 (UTC)[reply]
I guess not all your losses are heat. You lose quite a bit to vibration. I've seen the 80-85% ballpark used in wind turbine and cogeneration setups. (And I've usually seen those come out on the low end rather than overshoot the target.) It should be straightforward enough Joules in Watt out, but there's a lot of leeway in who measures what where as usual. If you look at steam temperature/wind energy to power output you get a different number than if you look at torque to output. 71.236.23.111 (talk) 22:43, 29 May 2008 (UTC)[reply]
I (130.95) will probably have to take back what I wrote. My notes, based on the rather old (1970s) book Man, Energy, Society by Earl Cook, tell me that the steam generator (in combination, called a furnace, boiler and superheater) is about 85% efficient, not at all what I thought, and the generator is 46% efficient. The figure of 46% refers to the steam energy going in (187 G Cal per hour) and the electrical energy coming out (86 G Cal per hour), so 86/187 is about 0.46. This means I know virtually nothing about the efficiency of the generator magnet itself, which is a small part of the whole contraption, so the other guesses are more reliable, and more relevant. In short, the answer I gave, "quite high," seems to stand up well :). 203.221.127.63 (talk) 17:09, 30 May 2008 (UTC)[reply]

Mortality rates expressed as hazard rates

Does anyone know where I could find mortality rates calculated as hazard rates? I'm trying to answer questions like, if a Canadian man makes it to the age of 93, what are his odds of surviving to 94? If an American baby reaches the age of 6 months, what are the odds of her surviving the remainder of her childhood? Thanks. moink (talk) 12:36, 29 May 2008 (UTC)[reply]

I once did a similar analysis on South African mortality rates and I used Statistics South Africa for the data so I would presume that the United States Census Bureau would be able to help you for US data. Do you know how to calculate the probabilities you need from actuarial tables? Also, see force of mortality which is the hazard rate in survival analysis with respect to mortality. Zain Ebrahim (talk) 13:20, 29 May 2008 (UTC)[reply]
Thank you! I think I just needed the term to search for. moink (talk) 16:45, 29 May 2008 (UTC)[reply]
You're welcome. All the best with your work. Zain Ebrahim (talk) 18:08, 29 May 2008 (UTC)[reply]

Horrible plastic smell!

I bought some plimsolls online from River Island, but they stink of plastic like they're from some crappy discount shoe warehouse.. I've put them outside to air but they still smell awful. I thought I'd ask you clever lot for some advice. Whack on the Febreze? Air tight container with some Bicarbonate of soda? Thanks in advance! 79.78.46.84 (talk) 12:57, 29 May 2008 (UTC)[reply]

You bought some "lines painted on the hull of a ship to help determine displacement and draft"? Where are you planning on using them? There has got to be another definition for that word.... -SandyJax (talk) 15:01, 29 May 2008 (UTC)[reply]
Um yes? plimsolls Nil Einne (talk) 15:05, 29 May 2008 (UTC)[reply]
(after edit conflict) Plimsoll shoes have a very pungent smell of rubber, I've always found, but not plastic which doesn't really smell much. Theres not much you can do about it. It fades with with time, I find. Fribbler (talk) 15:07, 29 May 2008 (UTC)[reply]
Also see wiktionary:plimsoll Nil Einne (talk) 15:08, 29 May 2008 (UTC)[reply]
My guess would be that this is the culprit Thiourea. Vulcanization makes rubber usable, but sulfur is known to combine into truly stinky substances. The thing is that you want to avoid the rubber drying out (which would ruin it.) So the baking soda will eat the smell, but in an airtight container will also desiccate the rubber which will make it brittle. Fragrance oil might cover the smell, and you could use some activated charcoal insoles. 71.236.23.111 (talk) 16:10, 29 May 2008 (UTC)[reply]
Maybe your posh expensive plimsolls from River island were made in a sweatshop just like the ones from a "crappy discount shoe warehouse"? Astronaut (talk) 16:20, 30 May 2008 (UTC)[reply]

A doubt

How does one cite matter from a chapter in a book that has contributors that are different from the editors mentioned on the cover and elsewhere? Which all authors are to be included in the citation?

Kindly help me with this doubt.

Thanks in advance.

Regards.

PS: Have posted the same query on the talk page of citations, but am not expecting a quick reply. Thanks.


—KetanPanchaltaLK 13:30, 29 May 2008 (UTC)[reply]

I would err on the side of more information, listing both the chapter's editors as well as the editors of the entire book. Roughly working from the MLA guidelines, "Works with Multiple Authors" should list the individual author first, followed by the group:
Heller, Steven, ed. The Education of an E-Designer.
Heller, Steven and Karen Pomeroy. Design Literacy: Understanding Graphic Design.
Hopefully this example helps. Nimur (talk) 14:23, 29 May 2008 (UTC)[reply]
Incidentally, that's not how you cite chapter authors. Chapters are cited separately and then in the same citation you cite the main work and note the editors. E.g. Paul, Diane B. (2003), "Darwin, social Darwinism and eugenics", in Hodge, Jonathan and Radick, Gregory, eds., The Cambridge Companion to Darwin, Cambridge University Press, 214–239--98.217.8.46 (talk) 15:23, 29 May 2008 (UTC)[reply]
Thanks Nimur, for your reply. But, my doubt was more about the technical aspect since I'd never seen the {{cite book}} in its expanded form, had only seen its abbreviated form at WP:Citation templates. Surprisingly, I got my doubt cleared at the talk page of citations itself! Bye. Take care. —KetanPanchaltaLK 14:54, 29 May 2008 (UTC)[reply]

Optimal nap/sleep time

I've read somewhere that if you're not having a full night's sleep, or if you're just taking a nap, certain sleep durations are better than others because you'll not be waking up during the "wrong" part of the sleep cycle. I think two recommended durations were given in the article I read, one for short naps and the other for longer sleep.

I can't find the article and don't remember what the recommended durations are. Can someone point me to some good articles/resources on the subject? —Preceding unsigned comment added by 71.162.242.15 (talk) 16:53, 29 May 2008 (UTC)[reply]

Power nap doesn't state a duration, but you might get that in the original NIMH study cited. --71.236.23.111 (talk) 17:11, 29 May 2008 (UTC)[reply]

Shindo earthquake scale

When measuring earthquakes there are different sclaes, like Mercalli intensity scale and Richter magnitude scale. Why did the Japanese invent their own scale "shindo" (Japan Meteorological Agency seismic intensity scale) (In Japanese called 震度)? Moberg (talk) 16:57, 29 May 2008 (UTC)[reply]

As the article's lede notes, the Japanese scale serves a function distinct from that of the Richter scale (and, for that matter, it predates the Richter). The Mercalli scale serves a similar purpose, but again postdates the Japanese scale (1902 vs 1884). — Lomn 17:21, 29 May 2008 (UTC)[reply]
("lede", what does it mean?) Ah okey, predates, thanks! But why is the Japanese the ones who made their own scale and not China or Australia or some other country? Moberg (talk) 18:55, 29 May 2008 (UTC)[reply]
(lede is an alternative spelling of lead and refers to the beginning of an article) --Tango (talk) 19:03, 29 May 2008 (UTC)[reply]
(ok! :)) Follow-up-question: I think that Taiwan uses this scale or a similar one. Is there any other other countries that uses the shindo- (or a from the shindo-scale derived) scale? And if so, why? Moberg (talk) 19:24, 29 May 2008 (UTC)[reply]

Original rotation of solar system.

Under the Nebular Hypothesis, the rotation of the solar system is caused by the contraction of the original cloud and, as it condensed, the rotation went from perhaps once every million years to that which it is today. The principle of the conservation of angular momentum is applied. The question is, however, what caused the original rotation? Any generally accepted reasons for the cause under this hypothesis, or are we still guessing?Boblaw1 (talk) 17:52, 29 May 2008 (UTC)[reply]

I don't think you need anything to cause it - chances are, when you add together the angular momenta of all the particles in the cloud, you're going to get something non-zero, the chance of all the random movements cancelling out exactly is vanishingly small. That tiny, but non-zero, value is then increased dramatically when the cloud collapses. --Tango (talk) 18:16, 29 May 2008 (UTC)[reply]
It's in the nature of gravitational capture for things to orbit around each other. Even if the whole cloud were to magically stop spinning, the next thing to wander close enough to start orbiting it would make the system be rotating again. --Sean 13:40, 30 May 2008 (UTC)[reply]
I'd say a slight rotation was imparted on the solar system by the rotation of the galaxy. So, the next question will be where that came from. There must have been some initial rotation right after the Big Bang (or perhaps even before), just like the dispersal of matter couldn't be exactly equal or we never would have gotten galaxies and solar system forming in the first place. StuRat (talk) 00:17, 1 June 2008 (UTC)[reply]

Nuclear reactors and fissile materials

What distinguishes the following technologies?

  • Uranium enrichment
  • Plutonium reprocessing
  • Graphite reactor
  • Light water reactor
  • Heavy water reactor

--141.161.98.180 (talk) 18:12, 29 May 2008 (UTC)[reply]

Might they all be distinguished by being the subject of homework questions? Edison (talk) 19:13, 29 May 2008 (UTC)[reply]
We have Wikipedia articles on each of those. Uranium enrichment, nuclear reprocessing, graphite moderated reactor, light water reactor, heavy water reactor. The different reactor types use different neutron moderators (which has different implications for what type of fuel you put in them and what types of products you get out in the end), uranium enrichment is about increasing the percentage of U-235 in a given sample of uranium, reprocessing is about taking spent fuel and getting out certain materials created by the fission reactions (such as plutonium). For more details, read the articles. --Captain Ref Desk (talk) 19:34, 29 May 2008 (UTC)[reply]

Diodes

what exactly will happen when we short two terminals of a common type of diode ,does there be any charge movement and a resulting effect on depletion region202.125.143.75 (talk) 19:25, 29 May 2008 (UTC)[reply]

Presuming that the diode was not connected to a circuit then probably nothing since unlike say a capacitor, I'm pretty sure a diode doesn't keep a charge. If the diode is connected to a circuit, then I believe shorting it will basically remove any potential affect the diode may have Nil Einne (talk) 19:51, 29 May 2008 (UTC)[reply]
A reverse-biased diode does effectively act like a small capacitor, storing a tiny amount of charge. I don't suppose most diodes have enough capacitance for it to have any measurable effect in practice, but apparently some do: see varicap. —Ilmari Karonen (talk) 21:48, 29 May 2008 (UTC)[reply]
I am guessing that the questioner is thinking that shorting the ends of a diode together will have some effect of connecting the P-type side of the diode to the N-type side. However, they are already connected in the P-N junction inside the diode. Connecting them again will have no effect. By bringing up the depletion region, the questioner is implying that the diode is in a circuit and has current passing through it. Shorting out the terminals in that case will mean that you have a parallel circuit. The short has a resistance of nearly 0 ohms. The diode is commonly treated as having 0 ohm resistance, but that is not true. The good ones are 300 ohms (if my memory from circuit design 15 years ago is correct). So, until the current surpasses the capability of the short, there will be basically no current in the diode. The depletion region will quickly dissipate. If you take a good electronics course, this is the type of problem you'll see on a final: At 0.02 seconds after shorting out the diode, what is the potential difference between the terminals? -- kainaw 21:49, 29 May 2008 (UTC)[reply]

Electronics home project

Well I would like to build a switch which is switched on and off by clapping your hands twice. What do I need, and how do I do it? Bastard Soap (talk) 20:29, 29 May 2008 (UTC)[reply]

You can order a kit to build a sound-activated switch (which seems to cost exactly the same as The Clapper). This page also has a link to a PDF version of the assembly manual which includes a theory of operation section. --LarryMac | Talk 20:41, 29 May 2008 (UTC)[reply]
Or order everything separately [16] (This doesn't have to be cheaper in the end. I've had a couple of DIY projects that added up to more in parts than the thing off the shelf. Less fun buying it, though.) Be sure to follow local code in all projects involving home installations! Have a licensed electrician check any circuit before connecting to your house electricity. (No advice, but if you burn down the house your insurance won't pay and your spouse will never shut up about it.)--71.236.23.111 (talk) 22:55, 29 May 2008 (UTC)[reply]

Nuclear Power Plant on "automatic mode"

In the event of a hypothetical doomsday scenario how long would a modern nuclear power plant be able to supply electricity before it shuts down? I know an that nuclear power plant's have some kind of dead man's handle mechanism as a failsafe but let's assume that it was turned off. How long could a modern nuclear power plant work on it's own? For hours, days, weeks or years? Mieciu K (talk) 21:57, 29 May 2008 (UTC)[reply]

Hours would be easy, and days would be likely. Weeks would be a stretch. This assumes that the system has some sort of automatic load following, or that the load on the generator remains constant. so it just has to do steady-state operation. If "doomsday" meant that the load dropped abruptly or that it dropped gradually and there was no load following, the generator would trip followed by the shutdown of the reactor. Edison (talk) 23:34, 29 May 2008 (UTC)[reply]
(Caveat: I'm only an expert on US Navy nuclear propulsion systems) Think of your car's engine. Things happen inside the engine too fast for a human to contol, so we automate things like lubrication, fuel and air intake, and valve timing. Whether controlled by a carburetor or an electronic fuel injection system, there is an automated control system that looks at system outputs like speed, exhaust temperature, and exhaust chemistry, and adjusts system inputs like fuel and air to maintain whatever it was preset to. Completely separate from that, there is a system that looks at lube oil pressure, and adjusts oil flow to keep that constant (that system is pretty simple; it's just a dump valve attached to a spring, so if oil pressure rises the valve opens more, dumping more oil back to the sump, etc). If you walk away from your car after you start the engine, what happens? It stays running until something goes wrong beyond the capacity of the control system. The engine will shut down when the fuel tank runs empty and fuel pressure drops to the point where the injectors or carburetors can't compensate. The engine will shut down if the car is in an enclosed space like a garage, and the oxygen concentration goes down to the point that it won't support combustion any more. The engine will shut down if a part breaks. The engine will shut down badly if it runs out of oil. Etc, etc, etc.
Now, with that in mind, let's look at a reactor plant. Again, nuclear reactions happen too fast for a human to control, so there are completely automated systems that make minor adjustments to keep the plant running at whatever settings the operators decide. Of course, there are very stringent limits on what the operators are allowed to decide. Some limits are set by the physicists who designed the reactor, some are set by the engineers who designed the steam plant the reactor is there to drive, some limits are set by government safety analysts who want to prevent accidents, and some limits are set by the Navy's operational needs.
However, if started up within those limits, a reactor plant will run on it's own until something happens that the control system can't handle. To the best of my knowledge, there is no "Dead man switch" that would make the plant shut down - and cause a regional blackout - just because there's a flu bug going around and all the operators had to run to the bathroom at the same time. That's the whole point of the automated systems - the humans look for trends, while the machinery runs itself.
Running out of fuel isn't a problem for several years, perhaps decades, depending upon fuel level. Fuel poison buildup shouldn't be a problem for a plant designed to run at full capacity for years at a time, as an electric generator plant would be; that would be factored into the design and the control system would compensate. Leakage shouldn't be a problem, either, as the nuclear side is designed to not leak, and minor leaks would be compensated for by an automated makeup system.
Any reason for the plant to shut down if the operators "just walked away" would come from the steam plant and electric generator side. Again, an electric generator plant would be designed to automatically compensate for changes in load, because they happen too fast for humans to follow. And, if we assume that nothing breaks, and no turbines run out of lube oil, and the river keeps providing cooling water, the reactor plant will keep providing electricity until the electric load on the grid drops to below minimum power. Another point. These plants generally have more than one of everything. More than one core, more than one turbine, more than one oil pump for each turbine, etc, simply because we are aware that things break, and we don't want to have a blackout just because an oil pump broke.
We can figure that the coal-fired and oil-fired plants will fail first, because no one refilled the coal hoppers and fuel tanks. Still, the hydro-electric plants and nuclear plants will have no problem with the remaining load, because there's no people left anymore, so there's not much using electricity. Major system loads will be automated lights for cities and highways and other things like that that turn themselves on. And, if a plant that is fueled for, say, five years at full power finds itself only running at ten percent power, well, then, it has enough fuel to run for 50 years at THAT power level.
Eventually, though, some random storm will down some power lines, and the transformer farm will isolate itself from the grid to protect itself, and _then_ the power plant will find itself running with no load. The steam plant can run at idle, just waiting for someone to shut the breakers again. but the reactor can't. Those fission by-product poisons we dismissed earlier DO have an effect. It's minor compared to the rest of the reactor running under full load, but when running at idle they have an effect that adds up. They will continue to contribute heat, and the control system will throttle back the fission process, until it is forced to shut the plant down. Safely, automatically, with no people involved. At least, that's the way US Navy plants are designed. When will that happen? Who knows? A day, a month, 5 years? Can't tell. But the nuclear and hydro plants will stay running until some outside factor makes them shutdown. -SandyJax (talk) 14:53, 30 May 2008 (UTC)[reply]
There was a straight dope "staff report" on this subject as well. [17] APL (talk) 16:27, 30 May 2008 (UTC)[reply]
Ok, thanks for all the answers. Mieciu K (talk) 07:22, 2 June 2008 (UTC)[reply]

List of Physics equations

We have a page which lists lots of trigonmetric equations. Is there a similar page for physics equations. In particular "v = u + at", "s = ut + 0.5at^2", "F = ma", etc. Maybe also for electrical equations "V = IR" "Q = VC" etc. If it does exist you can't find it by searching for "v=u+at". -- SGBailey (talk) 22:24, 29 May 2008 (UTC)[reply]

We have list of equations (not all Physics), list of relativistic equations and list of equations in classical mechanics (this one has the constant acceleration equations, but I think the TeX stops search working). Algebraist 22:42, 29 May 2008 (UTC)[reply]

Logic gates

I was mulling over how to make demonstration logic gates using many different technologies. Obviously a mechanical AND gate can be made with two input levers which can be up or down pushing on opposite ends of a floating pivoted beam where the central pivot holds a flagpole. If both are down, the pole is down, if only one is up the the beam pivots and the pole is down, if both are up then the beam itself is raised and hence the pole is up. What other technologies are there and how might they be demonstrated? Hydraulic (liquid pressure?); Pneumatic (flow (or not)); electromechanical (relays); electronic (transistors and voltages); Pure optical (no idea how); Magnetic (how?), etc etc. Suggestions and advice on a postcard please... -- SGBailey (talk) 22:35, 29 May 2008 (UTC)[reply]

Our school used to have a cool board with chutes, marbles and switches that switched LEDs on/off. --71.236.23.111 (talk) 23:02, 29 May 2008 (UTC)[reply]
Here's an idea for an optical OR gate. Light shines down both tubes, but each one may be blocked by a piece of cardboard. Where the tubes combine is some sort of prism/two-way mirror set-up that combines the two beams together and sends it down the bottom tube. If one tube or the other is open, or if both are, then there will be light coming out the bottom.
     |    |         |    |
     |    |         |    |
    -+----+-       -+----+-
     |    |         |    |
     |    |_________|    |
      \     *     *     /
       \    *     *    /
        \   *     *   /
         \  *     *  /
          \ *     * /
           \*     */
            |     |
            |     |
I think that perhaps to make a XOR gate you could consider how to make a NOT XNOR gate first - pick a medium that involves something flowing (like water, or sand), and let each switch divert the flow in opposite directions somehow. So having neither, or both on, lets the flow continue as it was, while having just one diverts it (into a secondary channel, say). You can then turn that all around, and make that secondary channel the primary one, and you've got a XOR. Confusing Manifestation(Say hi!) 23:38, 29 May 2008 (UTC)[reply]
Hmm. Yes. Remember that all logic operations can be built from NAND or NOR if you are really desperate. --Prestidigitator (talk) 01:04, 30 May 2008 (UTC)[reply]
Please be sure to see fluidics. Also, Tinkertoy computer [18] [19].
Atlant (talk) 15:27, 30 May 2008 (UTC)[reply]

How to calculate minimum wind speed to cause white caps on ocean

Wiklearn (talk) 23:42, 29 May 2008 (UTC)How is wind speed related to the height of ocean waves? At what wind speed would white caps appear? Wiklearn (talk) 23:42, 29 May 2008 (UTC)[reply]

You might try looking at Beaufort scale—in particular, the table that relates Beaufort numbers to wind speeds and to surface conditions on the open ocean. Deor (talk) 13:08, 30 May 2008 (UTC)[reply]

Definition for trompe on wikipedia please

Trompes are simple devices that use the power of falling water to compress air. Trompes were used widely in the 19th century. They were used to provide the air for mines, blast furnaces pneumatic tools for alpine tunnel making, etc. The town of Cobalt in Ontario, Canada claims to have the only hydraulic air compressor (Trompe) left in the world http://www.cobalt.ca/ragged_chutes.htm Researchers in England have tried to use low pressure trompes (attached to an enclosed wind turbine and electric generator) to extract energy from the water running over weirs in a cost effective way. (Pdf is available if required). I believe that a dedfinition of trompe would help show people in poor countrys that energy is available from low grade (low head) hydro sources without the huge investment in machinery that is currently a barrier to using these (often high volume) sources of energy. —Preceding unsigned comment added by Gaiatechnician (talkcontribs) 23:46, 29 May 2008 (UTC) Gaiatechnician (talk) 23:47, 29 May 2008 (UTC)[reply]

Not sure what you're driving at, but have you looked at "Trompe"? --Milkbreath (talk) 01:25, 30 May 2008 (UTC) Thank you but at the top it says "This article does not cite any references or sources. (November 2006)[reply]

Please help improve this article by adding citations to reliable sources. Unverifiable material may be challenged and removed". I actually wrote it and my poor effort has been sitting there with that proviso (and doubt) over it for a long time. Just hoping to get it fixed. My sources are long ago photocopys from engineering manuals. I have made some so I know they exist. Gaiatechnician (talk) 15:04, 30 May 2008 (UTC)[reply]

Ah. Now I see. That message about sources is just Wikipedia houskeeping. It serves two purposes, basically, the way I understand it: to warn the reader that there is no proof of the accuracy of the information, and to flag the article for possible deletion if it sits like that too long. Your concern is jusatified, I think. This is the Reference Desk, I'm sure you know, and it is not the right place for announcements of this type (though I think you will have gotten the attention of people who have the knowledge to fix the article). You should try the talk pages of whatever WikiProjects are concerned with things like trompes, for instance, Wikipedia_talk:WikiProject_Engineering and Wikipedia_talk:WikiProject_Technology. --Milkbreath (talk) 15:53, 30 May 2008 (UTC)[reply]

Such devices have existed. Within the last year there was an article in New Scientist showing methods of using low-head flow to compress air. The water flowed in the shape of a U taking air bubbles down to the submerged air reservoir in the middle. The water leaves at a lower level.Polypipe Wrangler (talk) 10:23, 30 May 2008 (UTC)[reply]

May 30

Adipocere smell

Hello, My question is about Adipocere aka, grave wax.

http://en.wikipedia.org/wiki/Adipocere

A friend of mine sent me a hunk of it, part of a woodchuck that had been trapped in an old well. It was in a zip-lock bag, which sat on a of table of hats and gloves waiting to be put away after a long Alaskan winter.

The smell of the Adipocers seeped out of the zip-lock bag, and stuck to the hat and gloves. It washed out, but how/why can it get though plastic. And stick to anything, including my hands, and some how I could not get the smell of it out of my nose for a long time. Why? Thank you for your time.

Art4u (talk) 02:24, 30 May 2008 (UTC)[reply]


The larger question is why a "friend" would send someone a hunk of dead woodchuck. It seems of little use, other than for pranks I described then thought better of per WP:BEANS and did not post. Serial killers have had similar problems when they leave their victims in the crawlspace of their home. Perhaps you should have covered the plastic bag of dead woodchuck with six feet of earth or a layer of concrete. If you had left it outside during the Alaskan winter, would it have more likely attracted or repelled Grizzly bears when they emerged from hibernation? Crime novels and some forensicsources[20] describe police and pathologists placing a bit of Vicks#VapoRub under theis nostrils when dealing with stinky corpses, and some sources also say it helps with adipocere [21] [22] but we do not offer medical advice. Edison (talk) 03:28, 30 May 2008 (UTC)[reply]
Erm, where's is the medical problem that requires medical advice? I would postulate that the apodocere has a fat based intense odour that produces some sort of particularly persistent and/or adherent molecules that have lodged in the nose and continue to cause the smell. Yes, I realise that individual molecules are required to stimulate the olfactory plate but show me another possibility. With regard to the seeping of odour through a plastic bag this is a well known phenomenon in certain plastics, the actual mechanism of the migration of the smell at molecular level is not known to me. Richard Avery (talk) 07:27, 30 May 2008 (UTC)[reply]

COOLING TOWER EVAPORATION

WHAT IS THE BASIS OF EVAPORATION CALCULATION IN COUNTER FLOW COOLING TOWERS ? WHAT ARE THE UNITS USED FOR LATENT HEAT OF EVAPORATION? —Preceding unsigned comment added by 59.182.38.14 (talk) 03:23, 30 May 2008 (UTC)[reply]

We don't do your homework here. Chris M. (talk) 04:54, 30 May 2008 (UTC)[reply]
Hi. True, we don't do others' homework here, but we can at least help the asker by posting some links to help. Thanks. ~AH1(TCU) 17:22, 31 May 2008 (UTC)[reply]

CHEMISTRY

PLEASE EXPLAIN THE INDUSTRIAL PREPARATION OF THE FOLLOWING ORGANIC COMPOUND (a). Ethene (b). Propene —Preceding unsigned comment added by Frankdinero (talkcontribs) 03:30, 30 May 2008 (UTC)[reply]

You might want to check out Ethene and Propene. Paragon12321 (talk) 03:54, 30 May 2008 (UTC)[reply]

Burning stomach fat with an ab machine

I recently saw a commercial for one of those ab machines, the kind that facilitates crunches. Naturally, it included before-and-after pictures of people who lost lots of fat around their stomachs in "just 30 days!"

Of course doing crunches would strengthen and define one's abdominal muscles. But in regard to burning fat in specifically the area around the stomach—does the body burn fat stored in a certain area faster if the muscles around that area are exercised? In other words, say that two identical people each expended a certain amount of energy during workouts, but one did crunches and one did, say, cardio. Would the first person's body burn more fat around the stomach than the second person's, or does the body have its own set order as to which fat deposits it burns? --zenohockey (talk) 04:15, 30 May 2008 (UTC)[reply]

Theres no such thing as spot fat reduction unfortunately. So the only way to get abs is have a really low body fat % (for most people). 61.69.132.119 (talk) 04:55, 30 May 2008 (UTC)[reply]
Well, there is liposuction for "spot fat reduction"...
Atlant (talk) 15:23, 30 May 2008 (UTC)[reply]
Look at this [[23]], particularly at the "Battling the Bulge" section where they describe that walking a mile burns 120 calories. What you can lose in 30 days is water. You also might find this article interesting [24] Crash exercising isn't that great, because you tear your muscles if you subject them to a lot more work all of a sudden. --76.111.32.200 (talk) 00:06, 31 May 2008 (UTC)[reply]
Very interesting...Thanks all. --zenohockey (talk) 01:49, 2 June 2008 (UTC)[reply]

Battery question, part II

Thanks to everyone who answered my question earlier, your comments were very helpful. So (in summary) the main consideration with batteries, in terms of the electronics that use them, is the voltage they produce. And now, operating on this premise, I am going to raise a few eyebrows. What would be the feasibility of using a series of common batteries (e.g., C or AA) to power my laptop? Bare question there, again hypothetical. For reference, the power adapter for my laptop says it gives the laptop 20v at 3.5A and 70W. 63.224.79.202 (talk) 04:24, 30 May 2008 (UTC)[reply]

Possible, but not really feasible. Let's take AAs. It'll take 13 in series to make 20v. The AAs I have sitting around list at just under 3000 mAh, which means those batteries will run that laptop (at peak power) for about an hour. Supposing that the laptop's standard battery lasts 4 hours, that means you need 52 AAs to duplicate its functionality. Those are the single-use ones. Go to rechargeables and the mAh rating drops to about 2000, which means you're looking at about 75 AAs to replace the laptop battery. Larger batteries (say, D cells) will have a higher mAh rating -- about 10000 mAh for rechargeables, or 16000 for single-use. That means you'd only need 13 single-use D-cells instead of the 52 AAs, though Ds cost about 4 times as much (if not more) per battery. — Lomn 05:02, 30 May 2008 (UTC)[reply]
Wow, so it would actually work. How would one hook that up? (Now I'm asking for trouble) 63.224.79.202 (talk) 05:09, 30 May 2008 (UTC)[reply]
Connect them in series, you will also need the same type plug (which plugs into laptop) as for laptops power supply (in worst case it could be connected directly, but it would require disassembly of device). -Yyy (talk) 10:00, 30 May 2008 (UTC)[reply]

NB: Doing this could easily void your warranty - especially if you have to take the laptop apart to get it connected up. While it's a nice theoretical idea, I advise against actually attempting it. --Tango (talk) 12:43, 30 May 2008 (UTC)[reply]

If device is not taken apart, then only problem with this would be undervoltage, when batteries will run out (i am not sure if computers is able to accept reduced voltages from power supply). If power plug is taken from pawer supply, it might void warranty for power supply. This method probably will not be efficient, because laptop computers usually consumes considerable amount of electricity, and batteries are expensive (if you have unlimited access to free batteries, then this is not an issue). -Yyy (talk) 14:53, 30 May 2008 (UTC)[reply]
Also, if batteries are connected in place of original laptop battery, these would need nome sort of smart battery chip emulation (it might be complicated). Laptop batteries usually have lower voltages (11-14V) and this power interface should be able to deal with reduced voltages better (when batteries are running out). -Yyy (talk) 15:01, 30 May 2008 (UTC)[reply]
I'm not sure if I've seen such a device for laptops, but for mobile phones, iPods, and the like, you can definitely buy commercial devices that let you use one-time-use (primary) batteries to power the gadget. And years and years ago, I had a flat-pack external lead-acid battery that connected to the power-inlet jack of my PowerBook 170 laptop computer. That battery was sized to match the outline of the laptop and added (maybe) a 1/4" to the thickness of the laptop, but doubled (+/-) the run-time of the laptop. Another gadget I had for the laptop allowed you to connect an external 9V rectangular-cell battery to the laptop's power-inlet jack. This wasn't enough to operate the laptop, but it allowed the laptop to keep sleeping even while you exchanged the laptop's discharged main battery for a fresh one. (Later, Apple built a small NiCd cell into their laptops to perform the same function: sleeping through a battery change.)
Atlant (talk) 15:14, 30 May 2008 (UTC)[reply]
You can simply take a 4AA battery pack and an USB extension cord, connect the two terminals to the two power cables in the USB cord and charge your iPod off that. The iPod has quite a big tolerance on input voltage so you can charge it with car batteries, lantern batteries etc. as well. --antilivedT | C | G 00:00, 31 May 2008 (UTC)[reply]

hens eggs

is it possible to freeze eggsand for how long —Preceding unsigned comment added by 84.48.90.44 (talk) 04:49, 30 May 2008 (UTC)[reply]

if we can freeze human eggs we can freeze chicken eggs i'm sure. now are you talking about fertilized eggs or unfertilized eggs or are you talking about chicken ova for the purpose of later insemination. if the former two, for what purpose, eating or breeding? if its for food yes you can freeze eggs, they just don't taste very well when you thaw them out. i would suggest you only boil them afterwards.` —Preceding unsigned comment added by Myheartinchile (talkcontribs) 06:19, 30 May 2008 (UTC)[reply]

It's probably not possible to freeze them in their shells, but for commercial use, restaurants commonly buy plastic bags containing the frozen, scrambled contents of a hundred eggs. That's the quickest way to crank out omelettes by the dozens. I'd imagine the same process (on a smaller scale) could be done at home. (This has come up on the Reference Desk before, BTW.)
Atlant (talk) 15:06, 30 May 2008 (UTC)[reply]

abreva for genital hsv2 treatment

can you somebody use a product such as abreva to alleviate or quicken the healing of genital (HSV1 not HSV2) fever blisters/cold sores? If not abreva then what? —Preceding unsigned comment added by Myheartinchile (talkcontribs) 06:16, 30 May 2008 (UTC)[reply]

I'm sorry but we can't give medical advice.--Lenticel (talk) 09:37, 30 May 2008 (UTC)[reply]

This isn't a medical inquiry, its a research one for my human sexuality class. and also out of curiousity. the articles on abreva and genital herpes didn't answer my question neither did web searches or a call to a health center. —Preceding unsigned comment added by Myheartinchile (talkcontribs) 01:15, 31 May 2008 (UTC)[reply]

Huh? I'm no expert, but "abreva" redirects to Docosanol quote used mainly as an antiviral agent, specifically for treatment of "cold sores" caused by the herpes simplex virus. unquote An here's the wikipedia magic: click on herpes simplex virus and it will get you to an article that starts out with Herpes simplex virus 1 and 2 (HSV-1 and HSV-2) I have no clue what you were looking for, but this seems to cover what you asked about. --76.111.32.200 (talk) 04:04, 31 May 2008 (UTC)[reply]

again does anyone know if abreve is used on the genitals?Myheartinchile (talk) 21:41, 1 June 2008 (UTC)[reply]

chemistry - material science

1.How pH affects the particle size in a chemical reaction ?

2. Explain the properties of Polyethylene glycol (PEG) ?

3.Give the actual definitions of Acid and Base ( Not basing on theories ) ? —Preceding unsigned comment added by Auap (talkcontribs) 06:52, 30 May 2008 (UTC)[reply]

3 media of amplitude can be expressed in keyboards, per Keyboard_expression. Are there any others, and if so, could you list them all?

Also, for all possible, probable and theoretical instruments, can you list and explain all forms of amplitude expression?68.148.164.166 (talk) 08:21, 30 May 2008 (UTC)[reply]

We do not do your homework, we can only point you in the right directions on Wikipedia and, assuming you make an attempt at solving them yourself, help you out where you may be wrong or stuck. Scaller (talk) 09:28, 30 May 2008 (UTC)[reply]
Some weired homework that would be! I think this is genuine.

1) As far as I know it doesn't, How could PH change the size of a particle?

2) See our article Polyethylene glycol

3) I don't know what you want here. What do you mean by "not based on theories"? Theresa Knott | The otter sank 15:56, 30 May 2008 (UTC)[reply]

"An acid (often represented by the generic formula HA [H+A-]) is traditionally considered any chemical compound that, when dissolved in water, gives a solution with a hydrogen ion activity greater than in pure water, i.e. a pH less than 7.0."
"In chemistry, a base is most commonly thought of as a substance that can accept protons. This refers to the Brønsted-Lowry theory of acids and bases. Alternate definitions of bases include electron pair donors (Lewis), as sources of hydroxide anions (Arrhenius) and can be (commonly) thought of as any chemical compound that, when dissolved in water, gives a solution with a pH higher than 7.0. Examples of simple bases are sodium hydroxide and ammonia."
Hope that helps. Regards, CycloneNimrodTalk? 16:42, 30 May 2008 (UTC)[reply]
An acid can coagulate a colloid, due to the charge changing on the colloidal particles. For example, add acid to milk and it will go thick and lumpy, onthe road to cheese. Graeme Bartlett (talk) 22:35, 30 May 2008 (UTC)[reply]

SPACE

WHY DO OBJECTS GET SMALL WHEN TRAVELING NEAR THE SPEED OF LIGHT IN SPACE 59.88.65.80 (talk) 09:16, 30 May 2008 (UTC)[reply]

Those objects that get smaller are most likely moving away from you. They occupy less and less of your field of vision. Was this a clever troll, meant to inspire the questions of many a confused theoretician not aware of the most logical and sound explanation? I will assume good intent. Scaller (talk) 09:32, 30 May 2008 (UTC)[reply]
I think the question is regarding Length contraction also known as Lorentz contraction. It is for the same reason as time slows down relative to an outside observer in order to maintain the constancy of the speed of light. Jdrewitt (talk) 10:04, 30 May 2008 (UTC)[reply]
(Scaller) Oh, then the impression of decreasing size (versus lengthening, that seems an increase) mislead me. Thank you. :) 81.93.102.185 (talk) 09:44, 31 May 2008 (UTC)[reply]
The core tenet of Special Relativity is that the speed of light will always been measured as constant by any observers (it is invariant), but all other things will be measured relatively based on the reference frame of the observer (they are relative). As a result, when you start doing talking about things which move at or near light speed, the only way for the very large speed of light to be measured as constant by other observers is you start seeing both time and space as being relative, as actually changing. You don't have to see things as getting small at the speed of light; you can see them as occurring at very different times than what the person inside that fast-moving frame would say. The reason why time and space seem relative is due to how we measure both time and space—how we define what length is, what an interval of time is, both are dependent on measurements of the speed of light. Make any sense? --98.217.8.46 (talk) 15:00, 30 May 2008 (UTC)[reply]
There once was a fencer named Fisk
Whose stroke was exceedingly brisk.
So fast was his action
the Fitzgerald contraction
reduced his rapier to a disk.
Atlant (talk) 15:02, 30 May 2008 (UTC)[reply]

Canthaxanthin concentration in Cantharellus cinnabarinus

What is the concentration of canthaxanthin in Cantharellus cinnabarinus (Wikipedia currently doesn't have an article on it, but on the genus Cantharellus)? The answer can probably found in the original paper of the discovery of canthaxanthin, but I don't have fulltext access - maybe some of you have? Thanks in advance! Icek (talk) 11:14, 30 May 2008 (UTC)[reply]

They don't provide a concentration figure. Also, they don't provide a good figure for the amount of canthaxanthin recovered, saying only that they got 0.9 mg, plus "a lesser amount" by recrystallization. They also mention "small amounts" of what they think was an isomer of canthaxanthin. I guess the total amount was in the neighborhood of 1 mg. They started with 75 g of mushrooms, so a rough figure for the concentration would be 13 μg/g. I had no idea that a pigment could color something at such a low concentration, but that shows what I know, I guess. --Milkbreath (talk) 12:13, 30 May 2008 (UTC)[reply]
You can ask for fulltext versions of pretty much anything at the Resource Exchange so you can read it yourself. Hint to Milkbreath, are you signed up there? ;) Franamax (talk) 17:25, 30 May 2008 (UTC)[reply]

Thank you very much, Milkbreath, and thanks for the hint, Franamax! Icek (talk) 14:03, 2 June 2008 (UTC)[reply]

You're welcome, and upon reflection I guess the stuff is mostly in one tissue or another. --Milkbreath (talk) 14:44, 2 June 2008 (UTC)[reply]

animal colouration

why do so many mammals(dog,horses,cats for instance)have a flash of a different colour on their chests?---Jo. Russell —Preceding unsigned comment added by 86.13.210.32 (talk) 12:34, 30 May 2008 (UTC)[reply]

Why questions are always hairy (if you'll excuse the pun :-) Animal colouration gives 3 main uses for color: concealment, signaling and mimicry. Although it may seem odd, a different color spot can actually help hide an animal. On top of your carpet a tabby cat like the one in Camouflage would stand out, but look how well the stripes hide it. So the spot could help "break up the pattern" of the animal. It might also signal that "this is the end with the head". To prey it would be useful to know what end of an predator it's facing and a predator would find it useful to know where an animals neck to bite is. (That would however not serve your own survival, so maybe not a likely scenario) It may also just be attractive to mates. (After all an animal can't just throw on a cool T or sassy skirt, but ^o.o^ have you SEEN the one with that spot?) I can't think of a model for mimicry. Having all those very sensible possible reasons, it's unfortunately just as likely that it's just a bit of gene left over from an ancient ancestor or pure coincidence and doesn't serve any purpose at all. --76.111.32.200 (talk) 21:03, 30 May 2008 (UTC)[reply]
Countershading? --Carnildo (talk) 21:19, 30 May 2008 (UTC)[reply]

Can anyone identify this bird?

http://pets.webshots.com/photo/1101444074041514092KxzdCz Thanks. :) 99.245.92.47 (talk) 12:58, 30 May 2008 (UTC)[reply]

I think this is the Crested Coua (Coua cristata), native to Madagascar. Most of the few images that I can find are greyer than shown in your photograph, but the patch of bare blue skin around the eye and the untidy crest seem to match. Here are a few links: [25] [26] [27] [28]. There is a Blue Coua (Coua caerulea), but it doesn't seem to have much of a crest [29] [30]. Also, the patch of blue skin around the eye doesn't seem quite right in the Blue Coua--Eriastrum (talk) 20:14, 31 May 2008 (UTC)[reply]

Uncontacted Tribes in Brazil

[[31]]

From what I think I understand, no one from the outside contacted these remote tribes but does that also mean that they don't know that another world exists from their jungle homes? --Vincebosma (talk) 13:08, 30 May 2008 (UTC)[reply]

It depends on the context you mean. Take, for example, the ancient Greeks. Eratosthenes estimated the size of the Earth some 2200 years ago. In that sense, the Greeks were quite aware that far more world existed than they had explored. On the other hand, they would not have been able to imagine the details of, for instance, the Maya civilization. Similarly, uncontacted peoples may vary in the degree of knowledge of the rest of the world. — Lomn 13:20, 30 May 2008 (UTC)[reply]
I'd say they would all have some inkling something else is out there, by seeing airplanes passing overhead. They may not figure out that people are on the planes, however. Also, they may have heard by word-of-mouth (from another nearby tribe that has had contact with the outside world). StuRat (talk) 00:03, 1 June 2008 (UTC)[reply]
Well, first off, we dind't know about their world. It's just that 'our' world is bigger. :) But if they have contact with neighbouring peoples (which may be very limited), then, as StuRat says, they will probably have heard of peoples still further away. Including us. But that info is likely to be very distorted. Think of the pictures mediaeval Europeans made of distant peoples. Absolutely weird, some of them, and mostly scary. We 'knew' about them, but not really. And it's likely to be the same with them now. Imagine how their possible perception of us as something weird and scary may have been reinforced when they saw the helicopter hover over their vilage. They must have been scared shitless, and if they could make out the (white?) people inside, they any attempts at overland contact is likely to have become much more risky. DirkvdM (talk) 08:01, 2 June 2008 (UTC)[reply]

Hologram monitors?

Are there any emerging technologies that replace monitors or TV's with holograms? 65.41.95.198 (talk) 13:17, 30 May 2008 (UTC)[reply]

Nothing very promising, no. --98.217.8.46 (talk) 14:50, 30 May 2008 (UTC)[reply]
The bandwidth required for a true "holographic" (volume-filling) display is much, much larger than the bandwidth required for a 2D display. If you need a million pixels for a given 2D display, you'd need a billion voxels for the equivalent space-filling display. We also don't yet have a good, cheap mechanism for creating visible voxels in free space. On the other hand, stereoscopic displays are quite practical today and are often used in scientific applications. Because they only represent the stereoscopic view seen from one point in space, they only require about twice the data as a 2D display.
Atlant (talk) 14:58, 30 May 2008 (UTC)[reply]
The Feb. 7 issue of Nature has a letter on "An updatable holographic three-dimensional display" described in that issue as "a breakthrough". It is a 102-mm square photorefractive polymer that can be rewritten and has a 45-degree horizontal viewing angle, meaning you could walk one-quarter of the way "around" the object. This is the first one ever, and it takes 3 minutes to update the screen, but it's there. I'm not sure if the bandwidth comparison is exactly right, since a holograph encodes the image in a completely different way, but the overview mentions military, medical and video-gamer markets, who are presumably willing to pay for bandwidth. It certainly won't be on the shelves for Christmas though. Franamax (talk) 17:20, 30 May 2008 (UTC)[reply]

Have you read the article on volumetric display? it's not exactly holography but fairly close -russ (talk) 22:33, 30 May 2008 (UTC)[reply]

And it's not terribly promising, at least for commercial usage. I don't think any of the methods mentioned on that page have much promise. --98.217.8.46 (talk) 23:44, 30 May 2008 (UTC)[reply]

Orgasms

I am 17 years old and I have not orgasmed yet... I just had sex for the second time yesterday... And I still can not bring myself to orgasm I masturbate regulary it feels good but I cant bring myself to climax. Even when I cum I don't get that enormous satifactation I hear about from others. What can I do? JenJenAndAway (talk) 13:44, 30 May 2008 (UTC)[reply]

How do you qualify "enormous satisfaction"? It's impossible to know what feeling another person gets from something beyond their expression/descriptions of it. Are your peers over-exaggerating? Are you expecting too much? Generally 'cumming' is a way of describing the act of orgasm (certainly for males), so in the second part it suggests you have already orgasmed before. The obvious options open to you are to experiment and try different ways to get yourself in that state. It sounds more like a mental, rather than physical issue from what you've said. If in doubt then see a doctor/medical professional who may be able to help you further. 194.221.133.226 (talk) 14:06, 30 May 2008 (UTC)[reply]

I suggest you stop worrying about it. If it feels good, your doing it right! Theresa Knott | The otter sank 15:46, 30 May 2008 (UTC)[reply]

We're bordering on medical advice here, really. I'd advise you to visit appropriate professionals for advice. Regards, CycloneNimrodTalk? 16:38, 30 May 2008 (UTC)[reply]
Get a copy of "Our Bodies, Ourselves" and read it. --BenBurch (talk) 01:12, 1 June 2008 (UTC)[reply]
"... and read it"? Aren't you being a little too demanding? :) DirkvdM (talk) 07:51, 2 June 2008 (UTC)[reply]
This is the Avril troll [32] Nil Einne (talk) 12:21, 2 June 2008 (UTC)[reply]
Seems to be a legitimate question, though. I think it's best to respond to questions here on their own merits, regardless of who asks them. It's certainly easier that way. --Tango (talk) 12:36, 2 June 2008 (UTC)[reply]
Perhaps, but this question doesn't make much sense anyway. While there's a possibility a 17 year old will not realise that particularly for males, 'cumming' is an orgasm/climax, it's rather unlikely IMHO. (There's a big difference between saying I don't seem to get much satisfaction from my orgasm and saying I cummed but didn't orgasm.) This combined with the fact the question seems to be coming from a male, but the name sounds female made the question somewhat suspicious. Now that there's evidence this person is the Avril troll, I personally don't see any point responding to this question since this users repeated bad faith questions make me suspect he or she does not really have a genuine question and so there's a good chance no benefit will come from my answer so I don't see any point wasting my time further on someone who's already wasted enough of my time. Of course, others are free to respond how they see fit, but I think it's worth letting them know who that this comes from a known troll. Nil Einne (talk) 18:49, 2 June 2008 (UTC)[reply]

Jack

Re definition of 'jack' as used in electronics:

Editors: please view the page

http://en.wikipedia.org/wiki/TRS_connector

Suggest that the author/s was/were confused.

A 'jack' is a socket, as in the slang 'up your jack'. Therefore a 'plug' cannot possibly be a jack.

Cheers, bwegz —Preceding unsigned comment added by Bwegz (talkcontribs) 13:53, 30 May 2008 (UTC)[reply]

Can be either plug or socket - see jack (connector). I would assume "jack" by default meant a plug, but that may be a Britishism. Reminds me of the time when we pretended innocent puzzlement and made our physics teacher explain the origin of the terms "male connector" and "female connector" ... "Gosh, sir, isn't that a bit rude ?". Gandalf61 (talk) 14:18, 30 May 2008 (UTC)[reply]
Strangely enough, I've always thought using the word "jack" to refer to to connectors, is an americanism, while the british would prefer the words "plug" or "socket". Astronaut (talk) 15:55, 30 May 2008 (UTC)[reply]


A jack is the socket and a plug is the plug. "Jack" alone has never, to my knowledge, referred to a plug in a reliable source such as a standards guide, a handbook for professionals or a textbook. See [33] from 2006 for phone plug and phone jack. A manufacturer's site [34] shows an adapter with two RCA jacks and one RCA plug. The terms are not interchangeable. Somehow the possible British usage is used in TRS connector with the redundant term "jack plug" to refer to what the references from the article call simply a "plug." It would be interesting to add a historical paragraph showning the development of this redundant terminology in popular British usage. Edison (talk) 18:54, 30 May 2008 (UTC)[reply]
See [35] & [36] & [37] & [38]. From what I can tell, jack means connector or more specifically TRS connector in the UK. It does not generally mean socket so jack jack makes no sense Nil Einne (talk) 05:41, 31 May 2008 (UTC)[reply]
There are some books [39] which use "jack plug" to refer to a plug. Perhaps this was to make it clear they were not referring to a "mains plug." Logically they should call the female part a "jack jack." Older books such as this 1917 one [40] just call them a plug and a jack (referring to 1/4 inch telephone plugs and jacks). Edison (talk) 19:18, 30 May 2008 (UTC)[reply]
Anyone else ever encountered "Jill"? —Preceding unsigned comment added by 76.111.32.200 (talk) 00:08, 31 May 2008 (UTC) Boticide would be sweet--76.111.32.200 (talk) 00:09, 31 May 2008 (UTC)[reply]
According to the OED (2nd ed. 1989), a jack is specifically a socket with one or more pairs of normally closed contacts that open when the plug is inserted, so that the plug becomes part of the circuit (earliest def. 1891). Thus (OR alert) the term 'jack plug' makes sense if you define it as the type of plug that is designed to be inserted into a jack. I also looked on answers.com, which consists mainly if not entirely of US sources, but it also says that a jack is a socket. So I agree with the OP. --Heron (talk) 10:51, 31 May 2008 (UTC)[reply]
It is news to me that 'jack' refers to a socket. I am used to 'jack' and 'jack plug' used interchangeably to mean the male connector described here as TRS connector (a term not only unknown to me but positively misleading, because 'TRS' to me is heat-resistant mains cable). If I needed to refer to the female connector I would say 'jack socket'. I am familiar with the phrase 'phone jack', and now I think of it I recognise that it often means the socket; but I would have taken this to be a transference from what I thought of as the primary meaning of a particular kind of plug.
Since words mean what people use them to mean, I assert that it is simply wrong to say that 'jack' does not mean 'plug', at least in the UK. --ColinFine (talk) 00:05, 1 June 2008 (UTC)[reply]

Ants

I have read somewhere that if ants where scaled up to human size they would collapse under their own weight. Is this true? If so, what forces change over thw dimensions which are so relatively close to each other? Bastard Soap (talk) 14:10, 30 May 2008 (UTC)[reply]

Yes. I think we have an article on this somewhere, but it's simple to explain: if you scale up an ant by a factor r, then it will become r3 times heavier, and its legs will become r2 times thicker (in terms of cross-sectional area, which is proportional to load-bearing ability). Thus the stress on the legs will go up by a factor of r, so for r large enough (up to human scale is plenty), they'll break. Algebraist 14:19, 30 May 2008 (UTC)[reply]
The classic text here is On Being the Right Size. Algebraist 14:24, 30 May 2008 (UTC)[reply]
See also Square-cube law. --Milkbreath (talk) 17:49, 30 May 2008 (UTC)[reply]

EPISTEMOLOGICAL UNITY:

DOES AN EPISTEMOLOGICAL UNITY AMIDST GRAVITY AND ELECTROMAGNETISM EXIST, OR NOT? —Preceding unsigned comment added by 134.2.3.103 (talk) 15:16, 30 May 2008 (UTC)[reply]

Which answer would be more satisfying to you, "YES" or "NO"? You need to more carefully define epistemology and unity to really enable a meaningful answer here. Nimur (talk) 15:35, 30 May 2008 (UTC)[reply]
Considering epistemology refers to philosophy, perhaps the philosophy desk would be a better place for your question? Had you wanted to know something on a scientific basis, we may have been able to help.
Umm... there isn't one. the Humanities desk, perhaps (as cryptically linked to by Nimur above)? --ColinFine (talk) 00:08, 1 June 2008 (UTC)[reply]
I didn't mean to be cryptic. Sometimes I forget that users do not all have WP:POPUPS installed to allow for quick mouse-over skimming of wiki-links. I'll be more explicit! Nimur (talk) 16:23, 2 June 2008 (UTC)[reply]

Sky color with thicker atmosphere

If the Earth's atmosphere was thicker but of the same composition, what would the sky look like? How much thicker would the atmosphere have to be to make a difference that we could see with the naked eye? 69.111.189.55 (talk) 18:11, 30 May 2008 (UTC)[reply]

Well, I'd say a thinner atmosphere would scatter less light, so would look darker - possibly dark enough to see stars through it. Then, a thicker atmosphere would scatter more light and likely appear brighter and whiter. The extraterrestrial skies article may shed some light too. ~Amatulić (talk) 18:20, 30 May 2008 (UTC)[reply]
It's been a long time since I did any work on simulating atmospheric effects, but I don't believe there would be any major changes. Rayleigh scattering is already strong enough that outer space doesn't contribute much to the color of the sky. The most noticeable change would be that atmospheric bluing of distant objects would get stronger, and be visible on closer objects. Eventually you'd reach a point where the Moon is no longer visible (quick estimate: a 1000-fold increase in atmospheric pressure), but I don't think you'd lose sight of the Sun before the atmosphere got so thick it wouldn't be a gas anymore. --Carnildo (talk) 21:34, 30 May 2008 (UTC)[reply]
I don't agree with the Sun part, as 1000 times more atmosphere would presumably mean 1000 times more and/or thicker clouds. Thick storm clouds can block out the Sun as is, but at 1000 times the normal density even light fluffy clouds would blot out the Sun. StuRat (talk) 23:55, 31 May 2008 (UTC)[reply]
That would depend on whether or not you increase the amount of water when you increase the pressure. It's a common problem with this kind of question - when you change one thing, you have to think carefully about what else you change and what you keep fixed. --Tango (talk) 00:30, 1 June 2008 (UTC)[reply]

ladybug diseases and breeds

I live in Topeka, Kansas at 39° N 95° W. We have had three dry summers in a row. Last summer I noticed none of the lady bugs had very good spots. Our local ladybugs have always been a bittersweet color, not bright red, and that has not changed. Very suddenly it became difficult to find a ladybug with spots. The ones that do have spots are "faded". It's like the spot disintegrated. The black is still a true black but the spots have become sort of granular, like the way a newspaper makes gray with black ink on white paper. In addition the ladybugs seem slow, they don't fly as often and they don't even discharge their poison very readily. My question is, could this be a disease like the colony collapse virus in bees, or is it an invasion of a slow and spotless species?

99.10.73.248 (talk) 20:11, 30 May 2008 (UTC)[reply]

To me, this sounds more like natural selection due to environmental pressures than diseases. According to the ladybug article, there are hundreds of species, presumably all with different combinations of colorations and spots. Because of the short breeding times, it is quite possible that environmental changes have, over successive generations, caused a different population of species to become more common than what you were observing a few years ago. Or the dominant species in your area have adapted to the new environment, and a consequence of this adaptation may include a different coloration. ~Amatulić (talk) 20:26, 30 May 2008 (UTC)[reply]

worm farms

I'm thinking of getting a worm farm, but I'm concerned about issues like smelliness. I've looked at the article, and indeed odours are a potential hazard, so can anyone tell me from theory or experience if this is a major problem, or if it is easy to avoid? I live in a block of flats, and my gardening ventures have turned out poorly, like if I try to preserve a plant, it soon turns into compost, so I'm not sure how I'll go at maintaining compost itself. The indicators are that I have no skill whatsoever at this kind of thing, so I need it to be easy. thanks, 203.221.127.63 (talk) 20:52, 30 May 2008 (UTC)[reply]

You didn't say if you are restricted to gardening indoors. Assuming you must do it indoors, you would have similar problems with odors as a pet owner. If it were me, I'd put my worm farm in a terrarium with a tight lid, with sufficient water, sunlight, and plant matter growing to provide oxygen to the worms. The odor should then be fairly well contained. I can't imagine why you'd want a worm farm indoors, though.
Also, if you have no skill whatsoever at gardening and related activities, perhaps you should try another hobby? Raising goldfish or something? ~Amatulić (talk) 21:36, 30 May 2008 (UTC)[reply]
Do you wish to have a worm-farm for the sake of breeding worms, or as a means of producing compost or in order to compost your kitchen waste (rather than disposing of it in the trash.) All these choices have different answers. In general if compost gets smelly, something is going wrong. You either added something that killed "the right kind" of bacteria or your compost isn't getting enough air. You might consider using a tumbler. But read the reviews. (Don't trust anyone who says something about 2-4 weeks!) Some types aren't as convenient as they look. I remember seeing DIY instructions somewhere, but couldn't find them just now. If you don't trust your abilities, start small. Good luck.76.111.32.200 (talk) 22:31, 30 May 2008 (UTC)[reply]
What's the point? ----Seans Potato Business 22:56, 30 May 2008 (UTC)[reply]
If you have chickens or like going line fishing you might want to have the worms. (There are even people who maintain a small business by selling them.) If you have a garden you want the compost and if you want to pamper your "green conscience" you can recycle your biodegradable waste that way. (Warning: The average person produces more of that than is good for an ordinary sized vermicompost!)--76.111.32.200 (talk) 00:18, 31 May 2008 (UTC)[reply]

Sciences

How are the 3 branches of sciences, Chemistry, Biology and physics linked. Many physicists say that chemistry and biology are just subsets of physics. However physics seems to be the most theoretical science as it revolves around theories while Biology and chemistry seem to be less theoretical with clearer evidence. Chemistry and biology also seem less quantitative. Are chemistry and Biology just looking at the same thing as physics from a different perspective? Clover345 (talk) 21:16, 30 May 2008 (UTC)[reply]

Well, I've heard it said "without physics, there would be no chemistry, and without chemistry, there would be no biology." Yes, they are linked. There are even entire disciplines that consist of overlaps between physics, chemistry, and biology: biophysics, biochemistry, physical chemistry, chemical physics, and solid state physics, for example. Physics isn't necessarily more theoretical; we have applied physics as well as theoretical chemistry as disciplines of study. ~Amatulić (talk) 21:31, 30 May 2008 (UTC)[reply]
Technically all sciences should be linked as describing different levels and aspects of the same reality. In practice, though, the linkages are not easy to make. Part of it is a problem of scale—there's a limit to how many different elements you can describe in a physical system before the calculations produce chaotic and unstable outcomes, much less the computational difficulties. But beyond that, it's often the case that concepts developed within one context don't easily translate into another context—they're incommensurable, in the terminology of philosophy of science Thomas Kuhn. In any case, we wouldn't necessarily expect all current theories to mesh together perfectly, since a good number of them are probably wrong, incomplete, or just rough abstractions of how nature actually works anyway. You'd have to assume they were all correct to expect they all fit together perfectly, and at the moment nobody even thinks that holy-of-holies subject, physics, is entirely complete or correct (cf. the commonly cited problem that General Relativity and the Standard Model are incompatible).
And then, on top of everything else, there are also the problems you mentioned: the different branches of science, the different disciplines, have evolved very differently over time, some being more quantitative, some being more qualitative, some focusing on a rather strict empiricism, some putting emphasis on mathematical theorizing, etc. Theoretically, again, if they were all describing the same reality accurately and with the same degree of precision, they'd all find ways to translate, but they aren't necessarily doing that (accurately, anyway).
There's no historical or philosophical reason to think that many of the currently mature sub-fields of science should easily reduce to one another. Some do; some don't. In any case, reductionism has its limits. Technically quantum physics should correspond in some way to molecular biology and from there to cellular biology and organismic biology. But quantum physics doesn't really tell us anything useful about organisms. It's not a useful way to think about them and it's not going to give you an insights into them. Can everything be reduced to it? Maybe, maybe not. Would it help us understand very much if we knew how to reduce things to that? Not necessarily. At some level we can consider all of the different levels of nature to be somewhat autonomous, somewhat unconnected, because it simplifies things even more than reductionism does—even though reductionism appears to simplify things, it really complicates them, because you start having to talk in terms of emergent effects and other things that are pretty hard to express either mathematically or conceptually, and are far beyond our computational abilities.
One last, last comment. One of the most intense areas of research in the United States over the last 50 years has been nuclear weapons. The dream has always been to be able to have total computational understanding of all of the processes that go on inside the exploding weapons, from the quantum level up through the macroscopic turbulence effects. The largest supercomputers have been developed again and again to work on specifically this purpose. If any area should demonstrate a mature understanding of how various sciences reduce to one another, this should be it. But it doesn't quite work. They've never been able to design reliable nuclear weapons from purely first principles—they've always needed testing, test data, modeling on smaller systems. If they can't do it there, I doubt they can do it anywhere, just because this has been a major area in which they've tried to do it and have strong incentive to do it. Just my two (dozen) cents. --98.217.8.46 (talk) 23:06, 30 May 2008 (UTC)[reply]
World-class answer, .46. --Sean 01:15, 31 May 2008 (UTC)[reply]
I think *.46 was dancing around this issue, but let me say it anyway: even with a theory of everything and unlimited computational power, you can't derive biology from physics. People have, sometimes jokingly and sometimes seriously, proposed that the final theory of physics will be "all logically possible worlds exist". Imagine this is true for a second. Does physics reduce to it? Only if our world is the only logically possible world, which I find pretty implausible. Otherwise we still need general relativity and the standard model, not as a matter of practicality but as a matter of principle. And we still need physicists, doing the same things they do now, because "all logically possible worlds exist" doesn't tell us which extension of the Standard Model is the correct one in the world we find ourselves in. The same thing happens at every level of science. Physics can't predict penguins. -- BenRG (talk) 13:31, 31 May 2008 (UTC)[reply]
Put simply: To predict an outcome, you need both the theory and the initial conditions. While biology could be perfectly predicted from a theory of everything, it would require an impractical amount of knowledge about the initial conditions. With those initial conditions, you could predict penguins (give or take quantum effects, anyway...). --Tango (talk) 00:27, 1 June 2008 (UTC)[reply]
According to one former physics department chair I know: The 19th century was the century of chemistry, the 20th century was the century of physics, and the 21st century will be the century of biology... This is meant to the reflect that person's opinion about when the most important discoveries in each discipline occured/will occur. Dragons flight (talk) 23:11, 30 May 2008 (UTC)[reply]
Of course, historically it doesn't quite work out that way. Biology started to eclipse physics as "the" hot thing as early as the mid-1950s, and by the 1970s it was clear that physics wasn't going to come up with anything as earth-shattering as it had in the past, not compared to biology. --98.217.8.46 (talk) 23:42, 30 May 2008 (UTC)[reply]
I dunno, perhaps it could in a few months, though biology does need some more of a push to assert it's validity in the face of some silly doubts. Chris M. (talk) 17:16, 31 May 2008 (UTC)[reply]

Amino acid substitutions in engineering; phosphorylation

There are certain amino acid substitutions that can be made in genetic engineering to maintain a protein's structural properties but prevent phosphorylation of residues or mimic constitutive phosphorylation. What are these substitutions? ----Seans Potato Business 22:51, 30 May 2008 (UTC)[reply]

Typically single amino acid mutations to glutamic or aspartic acid will mimic the phosphorlyated state due to the net negative charge. Additionally a serine, threoine or tyrosine mutation to alanine is a good starting point for preventing phosphorylation at known sites. Wisdom89 (T / C) 01:45, 31 May 2008 (UTC)[reply]

Liquid

Do all liquids contain water? —Preceding unsigned comment added by 75.169.24.1 (talk) 23:41, 30 May 2008 (UTC)[reply]

No. To be "liquid" is just a state of matter. If you heat anything up to its melting point it'll become a liquid. No water necessary. Some pure elements are even liquids at just room temperature, like mercury. Water is a specific molecule that happens to be a liquid at room temperature (and even it is not always a liquid—it doesn't need to be too hot to become a gas, or too cold to become a solid). --98.217.8.46 (talk) 23:46, 30 May 2008 (UTC)[reply]
Hi. Also, some substances, such as Carbon dioxide, will go directly from a solid to a gas (or vice versa) at its evaporation/freezing point, in a process called sublimation, and the visible gas released does not have to contain water, but whether it usually does or not, I'm not sure, as the cold temperatures can condense water to become visible. Thanks. ~AH1(TCU) 17:31, 31 May 2008 (UTC)[reply]

May 31

Fermentation in eukaryotic cells

Where does fermentation or anaerobic respiration occur in eukaryotic cells. Is it the cytoplasm, the mitochondria, or what? I've been searching online, and I can't find any clear answer. Any help would be greatly appreciated. FlamingSilmaril (talk) 00:25, 31 May 2008 (UTC)[reply]

In the cytoplasm. Wisdom89 (T / C) 00:30, 31 May 2008 (UTC)[reply]

Ok, thanks very much. FlamingSilmaril (talk) 00:32, 31 May 2008 (UTC)[reply]

EEG and epilepsy

Hi,

Does anyone knows if EEG data that are typical of epileptic seizures can be the sign of other illnesses than epilepsy?

Thanks! —Preceding unsigned comment added by 85.0.41.187 (talk) 02:37, 31 May 2008 (UTC)[reply]

Electroencephalogram#Comparison_table may be of some use to you. If you are seeking medical advise, however, it's advisable to see your physician. Regards, CycloneNimrod talk?contribs? 17:12, 31 May 2008 (UTC)[reply]

Acceleration Due to Gravity

Hello. How is the following equation for acceleration due to gravity proved: ? Thanks in advance. --Mayfare (talk) 02:49, 31 May 2008 (UTC)[reply]

In science there is no such thing as a proof, see Scientific method and Epistemology as well as the Science article. For example, for centuries Newton's law of gravitation was the widely accepted theory, but know we know that General relativity explains many things that Newton's theory did not. Jkasd 05:43, 31 May 2008 (UTC)[reply]
Although Jkasd is right, this doesn't mean there's anything wrong with your equation just because it's newtonian. On that note, newton did derive it from somewhere, he didn't just pull it out of hat. It's just straight up calculus. One of the major ideas underlying newtonian physics is that when a force is applied to an object, it ACCELERATES in the direction of that force. for a constant force (such as gravity). Accerleration is given in units of m/s^2, so . If you've done calculus, you may have noticed that as speed is the first derivative of accerlation, similarly is the first derivative of . The equation you gave is just the second derivative:
 


Ta Da! That's how it's derived. Your equation is just a fancy way of saying "Displacement is the second derivative of acceleration." As Jkasd pointed out, the most you can do is verify it to a high degree of certainty via experimentation. PS: this was totally the first time I played with making equations in wikipedia, and I found it to be a huge pain in the ass. For the record. --Shaggorama (talk) 08:09, 31 May 2008 (UTC)[reply]
Actually, I think you meant to say that the acceleration is the second derivative of the displacement function. What you did above was not differentiation, but integration. If you take the original definition of acceleration as the change in velocity over the change in time (valid only if acceleration is constant), and rearrange the equation you get the velocity equation above. If you integrate that with respect to time, you get the equation you originally asked about. PhySusie (talk) 11:32, 31 May 2008 (UTC)[reply]
Except (on closer examination) that the second equation in the box should have final velocity on the left side of the equation (not change in velocity) and the third equation should have final position on the left side (not change in position). PhySusie (talk) 11:35, 31 May 2008 (UTC)[reply]

Why is there ? --Mayfare (talk) 13:15, 31 May 2008 (UTC)[reply]

This is the standard result for integrating a polynomial of degree one; that is to say:
See integral. Very briefly, 1/2 is the result of calculating this integral from the definition of integration. Evidently you haven't had your first calculus class yet! All in good time; you will learn these tools if you continue your study of physics. For now, suffice to say that calculus is the mathematical tool developed by Isaac Newton while he was working on mechanics (physics). Calculus is essential to the study of physics, because it allows consideration of continuous quantities (smooth motion, for example). Integration and differentiation are not very difficult to learn if you already understand algebra; the twist is conceptual - applying limits, continuity, and functional analysis; and then practicing on a lot of sample problems. Nimur (talk) 15:12, 31 May 2008 (UTC)[reply]
File:D=at²+vt.svg
It's easier to see if you draw it.
Note that the 1/2 here has a very simple geometric interpretation: it comes from the fact that the area of a right triangle is half that of a rectangle with the same sides. To see the connection, plot speed (v) as a function of time (t): by the definition of acceleration, it's a straight line with slope a. Assuming you start at v = 0 when t = 0, the distance (d) travelled at time t > 0 will be equal to the area under the line and to the left of t (i.e. the integral of v with respect to t), which forms a right triangle with base length t and height at. This has 1/2 times the area of a rectangle with sides t and at, which of course has the area t·at = at2. If you start at some non-zero initial velocity v = v0 > 0, this moves the line upwards so that the area under it (and left of t) now also includes a rectangle with sides t and v0. Putting these together, you get d(t) = 1/2 at2 + v0t. —Ilmari Karonen (talk) 04:15, 1 June 2008 (UTC)[reply]
Another way of looking at it that may offer some insight is that the speed of the object at time t is at + v0. Thus, d1 = at2 + v0t = (at + v0)t (without the 1/2) is the distance the object would've travelled if it had been moving at that speed all along. But since the object only gradually accelerated up to its full speed, d1 is obviously an overestimate of the real distance. On the other hand, the distance the object would've travelled if it had been moving at its original speed of v0 is simply d0 = v0t, which is clearly an underestimate. To better estimate the real distance travelled, we can try taking the average of these two distances, which is davg = avg(d0, d1) = 1/2 (d0 + d1) = 1/2 (at2 + v0t + v0t) = 1/2 at2 + v0t, which we might expect to be closer to the real distance than either d1 or d0. As it happens, since, in this case, the change in speed is linear, this averaging "trick" actually does yield the exact distance the object actually travelled — but even if the acceleration had varied a bit over time, it still would've been a better approximation.
Also, in both of my comments above, I've implicitly assumed a and v0 to be positive scalars, since that's the simplest case to visualize. Once you have a good handle on that, it's a good exercise to look at the case where either a and/or v0 might be negative, or even non-collinear vectors, and show that the same rule still works in those cases as well. —Ilmari Karonen (talk) 20:36, 1 June 2008 (UTC)[reply]

Ice in a Hot Room?

How effective, if at all, would a jar of ice be at cooling down a hot room? Digger3000 (talk) 03:22, 31 May 2008 (UTC)[reply]

Evaporative cooler has a section on performance.--76.111.32.200 (talk) 04:12, 31 May 2008 (UTC)[reply]
Evaporative coolers start with liquid water. Ice can absorb more heat. Just a moment, let me do some arithmetic... --Anonymous, 05:00 UTC, May 31, 2008.
Okay, say you start with 1 kg of ice at -15 C from a freezer, and you want to finish with water vapor at a room temperature of 25°C. The specific heat of ice is 2.05 J/gnbsp;K (or equivalently kJ/kgnbsp;°C), so you'll need 15x2.05 kJ to reach the freezing point. Then the latent heat of fusion of water is 333.55 kJ/kg, so you'll need that many more kilojoules to get a liquid. Now liquid water has a higher specific heat, 4.18 kJ/kgnbsp;°C, so it's 25x4.18 kJ to reach room temperature. And finally the latent heat of vaporization is a further 2,270 kJ/kg. So the total heat energy needed adds up to over 2,700 kJ.
Now say that the room is 5 m square by 3 m high; that's 75 m³. The density of air is about 1.2 kg/m³, so that's 90 kg of air. It could give up 2,700/90 = 30 kJ/kg. The specific heat of air is 1 kJ/kg °C, so a 1 kg chunk of ice could lower the temperature of the air by a full 30°C.
But there's a big flaw in the above calculation, which is that I assumed that the same 90 kg of air would be present the whole time! In practice, air in a normal room is circulating in and out all the time, even when the doors and windows are closed. That's why you don't suffocate when inside a building. The melting and evaporation of ice is a slow process and there's not much you can do to speed it up.
In other words, it's not the heat capacity of the ice that's the issue, it's the transfer rate of heat from the air to the ice. You not only need to cool the air, you need to cool it before it's replaced with fresh warm air from outside. It is possible to cool air with ice -- it used to be done routinely on trains, before head-end power was available to run air-conditioners - but in practice it takes a lot of ice, way more than my calculation above would suggest. Forcing the air to blow over the surface of the ice would only help a little, because it doesn't have much surface area.
--Anonymous, 05:36 UTC, May 31, 2008.
On trains in India during the 19th century, this type of air-conditioner worked by misting ice water into the train car. The device was called a thermantidote. I had a heck of a time looking this word up when I read this novel (a broken thermantidote ended up being some kind of ... hiding place for a criminal, or something like that). This terminology is decisively not in current use in America (we've still got a redlink article)! I figured it sounded like some kind of medication. Nimur (talk) 15:23, 31 May 2008 (UTC)[reply]
Oh, mist coolers are used all the time in tropical places! Every open-air bar in Key West, FL has them! --BenBurch (talk) 01:08, 1 June 2008 (UTC)[reply]
Rather than cooling the whole room, try just cooling yourself by blowing air over the block of ice onto you. That should be noticeably cooler than just using a fan alone. StuRat (talk) 23:49, 31 May 2008 (UTC)[reply]
Or as a short cut, misting yourself near a fan for evaporation effects. Julia Rossi (talk) 00:41, 1 June 2008 (UTC)[reply]
That's an effective cooling method, but doing so inside will result in damp (and possibly moldy) furniture, drapes, carpet, etc. StuRat (talk) 15:58, 2 June 2008 (UTC)[reply]

Human Biology

The subject is Sweat Glands. The question is: The body has many sweat glands, but what parts of the body or where on the body sweat glands CANNOT be found?? 75.89.250.154 (talk) 04:09, 31 May 2008 (UTC)[reply]

Next question, "When's your homework due?" Here's a hot tip on how to find answers for such questions. Your teacher has probably assigned a textbook for this class. I's a great idea to actually get that. (Check for used books if your budget's strapped or check your local library.) Then go to the table of contents in the back. Look for "sweat" or related subjects. That should either get you a page, or at least the section. skim that section for things that say "sweat" or "glands", and start reading around it. Pay attention to things saying "except", "no" or "not". That should get you the answer and the skills you're building that way can let you survive not just to the next test, but throughout all your studies.76.111.32.200 (talk) 04:58, 31 May 2008 (UTC)[reply]
WP:SARCASM. Careful with the tone. This doesn't appear to be a homework question. Regardless, to the user who asked the question, simply scour the internet. Seriously, there's loads of info out there about this topic. Wisdom89 (T / C) 05:03, 31 May 2008 (UTC)[reply]
To me it does sounds either like a homework or quiz question Nil Einne (talk) 05:27, 31 May 2008 (UTC)[reply]
You need to qualify the answer you're looking for, or the question is trivial: I don't think you can find sweat glands in any of the internal organs. If you're restricting the answer to places on the surface of the body, you need to decide whether, say, finger nails count as an acceptable answer. --72.78.237.206 (talk) 12:56, 31 May 2008 (UTC)[reply]
Eyeballs don't have sweat glands either, do they? And have about scar tissue, does that grow back sweat glands? Graeme Bartlett (talk) 21:44, 1 June 2008 (UTC)[reply]

Night Luminous Pearl (Ye Ming Zhu in Chinese PinYin)

How is the Night Luminous Pearl (frequently given as gifts to Emperors in Ancient China) formed in nature and how are they harvested ? What is the difference between this Luminous Pearl and normal Pearl from Oysters e.g its mineral contents ? Is it found only in China, as no other countries seem to refer to it in history. Currently, where can one view / purchase a real Night Luminous Pearl ?

218.186.72.68 (talk) 05:20, 31 May 2008 (UTC) KKC[reply]

logic programming

(1)why is logic programming regarded as the corner stone of knowledge based programming41.209.23.34 (talk) 06:01, 31 May 2008 (UTC) (2)can an informal model theoretic argument be established for statements like this,(has niece(x)if(3x)daughter(x,y)sibling(x,y)) {x/brother(y)}.[reply]

>::cough::HOMEWORK::cough::< --Shaggorama (talk) 07:26, 31 May 2008 (UTC)[reply]

Got Lost In One Of The Questions (I'll Post This On Entertainment, As I'm Not Sure Which Desk Is Most Appropriate)

3 media of amplitude can be expressed in keyboards, per Keyboard_expression. I am assuming different instruments, such as Brass instruments, or instruments that use water, have different ways of changing dynamics, or the amplitude of the sound that can be normally produced by them, or not, say banging a saxophone against a wall. Are there any others, and if so, could you list them all? Also, please list all theoretical, possible and/or probable ways of changing dynamics (or, in other words, the amplitude of a sound).

So what I'm saying is that for different Keyboard instruments, as per the article Keyboard_expression, there are 3 ways to express dynamics.68.148.164.166 (talk) 07:30, 31 May 2008 (UTC)[reply]

I think you're asking, what are the different ways of varying the volume on different musical instruments? I suggest looking at Musical instrument classification, and following the links from there. I would point out that asking people on reference desks to list all of something is not usually a very productive request: the people who reply on these desks do so voluntarily out of interest or a desire to help, but unless somebody happens to be a completist about a topic, they may not want to spend the time compiling lists. --ColinFine (talk) 00:23, 1 June 2008 (UTC)[reply]

Humidity and electronic equipment

Why do specifications for electronic equipment (TV, computer, etc.) specify a range for humidity, eg. 10-70%? If the air is too humid or not humid enough, how does that affect the equipment? I've heard from friends that it would catch on fire, but I doubt it. Rilak (talk) 07:42, 31 May 2008 (UTC)[reply]

I'm not entirely sure, but if it's too humid, I'd say condensation might occur in the device which might short circuit the whole thing. A lack of humidity doesn't sound like it would do much, although dry air does increase the chance of static electricity building up and whatnot.84.198.96.249 (talk) 08:37, 31 May 2008 (UTC)[reply]
I agree with the above for high humidity, now let me try to explain the low humidity issue, as well. It might seem odd, but quite a few electronic devices include paper (say a circuit diagram glued to the inside of the case) or paper-based tape. When either loses all of it's humidity it can fall apart. Those pieces can then fall onto some hot component and catch fire. StuRat (talk) 23:40, 31 May 2008 (UTC)[reply]

Birds and hot chips

moved from misc desk
When I eat a very hot chip (or fries, depending on your preference) I can burn my mouth. But when I drop a hot chip on the floor a seagull will swallow it down whole. Do they not feel the heat of the chip in their mouth?Iiidonkeyiii (talk) 08:48, 30 May 2008 (UTC)[reply]

Good to see another seagull question here at last. First, I would think that for a bird to swallow it whole, it would be a relatively small chip so it is already cooler than the fat ones. Secondly, birds don't chew their food and gulls usually swallow their food whole, so they probably don't notice its temperature until it's too late.--Shantavira|feed me 09:24, 30 May 2008 (UTC)[reply]
Given that they do not chew, can we conclude that they have additional mucous or other sturdy membrane linings in their esophagus? I imagine they would need to be made of tougher stuff if they regularly swallow food whole. Nimur (talk) 15:30, 31 May 2008 (UTC)[reply]
Gulls also seem to be capable of quickly horking up anything that doesn't feel right in their crops. Sometimes they'll make several attempts at eating that large/hard/spiky thing to see if they can get it to sit comfortably inside. The other week, I saw a gull making three attempts at keeping a whole tomato down before finally deciding just to bite into it. --Kurt Shaped Box (talk) 22:23, 31 May 2008 (UTC)[reply]

Power Quality Managment

Is it possible to improve power by improving power factor? If yes upto what percent we can improve —Preceding unsigned comment added by Syedshahid (talkcontribs) 08:31, 31 May 2008 (UTC)[reply]

See power factor and power factor correction. Both of your questions are answered there. --Heron (talk) 10:11, 31 May 2008 (UTC)[reply]

Can you shoot yourself in the head twice?

Assume the following: a person has two pistols, one in each hand. Would it be possible for that person to fire both guns at their own head? Or would one lose muscle control faster than you can squeeze the second trigger? Pardon my morbid curiousity :) 84.198.96.249 (talk) 08:35, 31 May 2008 (UTC)[reply]

Rigor mortis—contraction of skeletal muscles due to depletion of ATP or cadaveric spasm might trigger the contraction of fingers of the other hand making them press the trigger. So, this might result in the situation you are speculating. I personally, have no problems with your curiosity. Regards. —KetanPanchaltaLK 09:05, 31 May 2008 (UTC)[reply]
You would be lucky to still be aiming right by the time rigor mortis sets in. It would be easier to fire both guns at the same time - that ought to work fine (even if you're off by a fraction of a second there is enough delay in the firing of the gun and the loss of muscle control to get away with it - or at least, I expect there would be, I've never actually tried it!). --Tango (talk) 12:04, 31 May 2008 (UTC)[reply]
If you're very, very lucky, you could get the 'classic' sort of suicide story where both your bullets hit each other and so don't kill you (since it's likely both your bullets will be at an angle, I presume they will still go forward and may still hurt you but I guess could potentially lose enough momentum so that they don't kill you although I can't be bothered doing the maths) Nil Einne (talk) 16:10, 31 May 2008 (UTC)[reply]
I was imagining one gun on each side of your head, so if the bullets hit each other they would do so in the middle of your brain. Where are you imagining the guns? --Tango (talk) 16:46, 31 May 2008 (UTC)[reply]
I'm also curious to know what's going on inside Nil Einne's head. I imagine Nil misunderstood the question since I can't imagine any configuration that conciliates two shots aimed at the head with both bullets hitting each other to save the experimenter. ----Seans Potato Business 13:03, 1 June 2008 (UTC)[reply]
He (she?) was considering the scenario where each gun's bullet is aimed towards the side of the head farthest from the gun. The bullets would then have a possibility of colliding in mid-air prior to entering the head. here's a diagram.
Suppose both guns are pointed towards the head so that their barrels, if extended, touch the head. The bullets cannot be travelling directly towards the head's centre since their collision point would then be inside the brain. Each bullet can be considered to be concurrently travelling towards the centre of the head and in the perpendicular direction. Upon collision, the bullets' velocities perpendicular to the path to the head's centre partially cancel out, while their velocities toward the head add. Whether the resultant amalgamation of the projectiles would have a greater speed than its components would have had without a collision depends on the angle between the head and the guns.
The probability of two bullets intercepting each other in mid-air, though, is likely less than the probability of winning the lottery. --Bowlhover (talk) 15:26, 1 June 2008 (UTC)[reply]


A simple fail-deadly system would be to put a strong rubber band around each trigger finger that requires you to actively resist pulling the trigger. Point the guns at your head such that one bullet's exit won't knock the other gun out of position, and then just ... relax. --Sean 17:12, 31 May 2008 (UTC)[reply]
Fail-safe, you mean? Since the goal is suicide and contingencies are planned to meet it, an initial failure is "safe" because it would not render the goal impossible. --Bowlhover (talk) 15:26, 1 June 2008 (UTC)[reply]
I'm disturbed at how practical these answers are getting. Doesn't this fall under the aegis of no medical advice? ;-)--Fangz (talk) 18:59, 31 May 2008 (UTC)[reply]
It's no medical advice, not no immoral advice. Discussion of war and other violent means is allowed here. SpinningSpark 19:55, 31 May 2008 (UTC)[reply]
Is it true that rigour mortis doesn't set in if you're shot in the eye? Bastard Soap (talk) 20:26, 31 May 2008 (UTC)[reply]
While being shot in the head is usually fatal, a surprisingly large number of people survive. Here are some examples [41][42][43][44]. However, my favourite is Michael Moylin who woke up with such a severe headache he thought he had suffered an anuerism. The hospital soon put him straight though, and pointed out that his wife had shot him in the head while he had been sleeping [45][46]. So yes, it would be quite possible to shoot yourself in the head multiple times because you might survive the first bullet. This is not a reliable method however - don't try this at home. SpinningSpark 20:47, 31 May 2008 (UTC)[reply]
I was trying to find a newspaper story on a woman who shot herself in the head in (possibly) the 1980s and survived. She had shot herself in the right temple but the surgeon removed the bullet from her left temple. The bullet had not, as one might imagine, passed through the brain. It had, in fact, skidded across the top of her skull underneath the skin. There was a great x-ray pic in the papers at the time - if anyone has a link please post it, I couldn't find it. Some women are just lousy shots I guess[47]. SpinningSpark 20:47, 31 May 2008 (UTC)[reply]
Well, Bastard Soap (am not calling you names! That's your user name), considering the mechanism by which rigor mortis develops (depletion of ATP) in the individual skeletal muscles, I don't think what happens to the brain (shooting in the eye) should have much influence upon the setting in of rigor mortis. Though there are factors like temperature, emotional state, poisoning by arsenic, etc. that could alter the rapidity and depth of rigor mortis, I don't know if the factor you are talking of could also be one of these. But, remember all these factors I enumerated simply determine the intensity and timing of rigor mortis not whether it develops or not, per se.
With all this talk going about medical advice and some one stating "Discussion of war and other violent means is allowed here.", this point from WP:Common sense could be instructive here:

"Similarly, just because something disruptive is not forbidden in a written rule doesn't mean it's a good idea (e.g., don't disrupt Wikipedia to illustrate a point). The spirit of the rules is more important than the letter."

But since, I believe the user who originally started the topic must be well aware that it can be fairly assumed that firing of both the pistols doesn't really greatly alter the probability of death, he'she is not asking for medical advice. Regards. [[User:KC Panchal|

With rigour mortis I mean the sudden contraction at the moment of death (don't think it's the right term but don't know the right one). I remember reading somewhere that snipers aim for the eyes in hostage situations so even if the terrosit has a gun pointed he won't fire in death. Bastard Soap (talk) 15:49, 1 June 2008 (UTC)[reply]


—KetanPanchal]]taLK 20:52, 31 May 2008 (UTC)[reply]

Yes, I'm indeed quite aware that one shot to the head is usually quite fatal enough without the need for a second one ;). I just get the occasional bout of morbid curiousity. I think the rubber band guy might just be on to something though, very clever :D84.198.96.249 (talk) 22:22, 31 May 2008 (UTC)[reply]
Going back to the original scenario, let's say that the guns involved are Walther P99 pistols loaded with american .40 calibre ammunition with a muzzle velocity of 320m/s. Let's further assume that the bullet is decelerated to a complete stop just before exiting the head (this assumption gives you the maximum possible reaction time, a "through and through" shot will give you less time as the bullet travels faster). With those assumptions, the travel time through my 16cm head would be exactly 1ms. So you have, at most, a 1ms reaction time to fire the second gun before your brains are scrambled and unable to respond. According to the article reaction time for an audio stimulus is about 150ms. So the answer is no unless you contrive to fire both guns simultaneoulsy. Simultaneous in this context being . This is humanly next to impossible, but probably unverified by experiment. Don't try this at home. SpinningSpark 22:15, 31 May 2008 (UTC)[reply]
There's also going to be some time between squeezing the trigger and it actually firing. Also, you don't need to respond to a stimulus to fire the second gun, you can have already made the decision to fire it before firing the first. If you fire them simultaneously your margin of error is the 500 microseconds you mention, plus the time the gun takes to fire and the time it takes for the signal to get from your brain to your finger (you'll still pull the trigger even after your brain is gone if the signal has already been sent). I don't know what that works out to, but I expect it's significantly more than 500 microseconds. --Tango (talk) 23:18, 31 May 2008 (UTC)[reply]
One could argue philosophically that if it is a reflex or rigor mortis that causes you to squeeze the trigger rather than a conscious choice, then you, personally, in fact did not shoot yourself twice. Wrad (talk) 23:24, 31 May 2008 (UTC)[reply]
You could, but I don't think anyone mentioned reflexes... --Tango (talk) 00:22, 1 June 2008 (UTC)[reply]
They did further up. Wrad (talk) 01:29, 1 June 2008 (UTC)[reply]
The breakdown of reaction time is (very approximately) 10ms ear to brain transmission, 100ms decision time, 40ms brain to hand transmission. SpinningSpark 13:13, 1 June 2008 (UTC)[reply]

Trapezius muscle—A doubt!

If all the fibers in the trapezius muscle are supplied by the same nerve, will the simultaneous contraction of the superior and inferior fibers not antagonize each others' actions? Hope someone answers it.

PS: I have posted the same question on the talk page of the article trapezius muscle.


Regards.

—KetanPanchaltaLK 09:05, 31 May 2008 (UTC)[reply]

Not all neurons that form the part of the accessory nerve that supplies the trapezius need fire at the same time. Indeed, upper and lower fibres of trapezius have opposite actions on the scapula. -- Flyguy649 talk 14:29, 31 May 2008 (UTC)[reply]
Hi flyguy! You mean these neurons come from different spinal segments? Yes, that is a distinct possibility considering that the spinal accessory nucleus has the root values of C1-C5. Thanks. Regards. —KetanPanchaltaLK 17:43, 31 May 2008 (UTC)[reply]

Pauli Equation

I I wanted to use the Pauli equation to describe a spin-4 particle, would I just have to replace the Pauli matrices with the spin-4 equivalents, or would I have to change something else, such as changing the part at the beginning fron 1/2m to 4m? Thanks, *Max* (talk) 16:01, 31 May 2008 (UTC).[reply]

The 1/(2m) part doesn't change. You just increase the dimension of the spinor part of the wavefunction (for spin 4 it has dimension 9), and replace the Pauli matrices. —Keenan Pepper 02:50, 1 June 2008 (UTC)[reply]
Good, that's what I'm doing. Looking at the equation again, it's obvious that the 1/2 doesn't chamge. Will changing the matrices get rid of the Pauli exclusion principle? *Max* (talk) 22:16, 1 June 2008 (UTC) It was never there. *Max* (talk) 19:52, 2 June 2008 (UTC)[reply]

NUCLEOPHILICITY VS BASICITY

Hello,can some1 please help me to undstand what is the difference between NUCLEOPHILICITY and BASICITY.After going through books,i have learnt that they are somehow directly proportional in terms of strenght.But I'm bit confused as I find sometimes stronger nucleophiles are weaker bases.also i found that type of solvent affects nucleophilic nature.My text books dont explain clearly why.Can Some1 please help? Abhiroopron (talk) 17:06, 31 May 2008 (UTC) Abhiroop.[reply]

The strongest bases typically bear negative charge and lone pairs on the same atom. This property makes them both excellent nucleophiles and lewis bases (electron pair doners). This actually creates a problem in protic solvents, as they are typically very good at accepting hydrogen bonds if not outright reacting with the solvent. As a result, stronger bases (especially when comparing halide ions) can often be very poor nucleophiles in such a solvent. This can be thought of in either of two ways; in the kinetic sense, having the nucleophile lone pairs bound up by the solvent reduces their availability to attack the electrophile, lowering the rate of reaction; in the thermodynamic sense, the interaction with the solvent lowers a nucleophile's (and thus, the total reactant's) energy, reducing the favorability of a reaction. As an example of such a thing, iodide ion (an extremely weak base) will complete its reaction with methyl iodide (that's an interesting reaction, isn't it? But you can measure it via iodide isotopes) in 17 minutes in methanol, and 8.7 seconds in Dimethylformamide (a polar aprotic solvent). Faster in the aprotic sovlent, as expected from what I just told you. Cyanide, a much stronger base than iodide, reacts in 0.011s in DMF (the stronger base reacts faster than the weaker base, as you yourself probably wanted to say). In methanol, however, cyanide ion takes 1.5 hours to completion, about six times slower than iodide. Thus, as you can see, stronger bases are often worse nucleophiles in a polar protic solvent. Someguy1221 (talk) 23:36, 31 May 2008 (UTC)[reply]

Thanks a lot,that made some things much clearer to me.But smthng still confuses me.I now undstand about this nucleophilic strenght being reduced in protic solvents due to H-bonding(i know now after i looked up the terms here after your reference about the action of such solvents),but why is the basic strenght not reduced too.i mean isn't the strenghth of a base also measured by its ability to co-ordinate with a proton.if its lone pair is hindered(thereby making it a weaker nucleophile evidently),then why doesn't its(or does it)proton abstraction ability decrease hence making it a weaker base according to the definition of a lewis base if i'm not wrong.I'll be really grateful if you can clear my doubts.Thanks mr...umm Someguy —Preceding unsigned comment added by Abhiroopron (talkcontribs) 11:40, 1 June 2008 (UTC)[reply]

Well, keep in mind that nuclephilic/electrophilic attacks are Lewis acid-base reactions. The coordination with a proton is Brønsted basicity. Now, something that is usually not discussed until advanced chemistry courses, the strength of an acid/base is dependent on the solvent. But for Brønsted reactions, I doubt you are going to see the complete reversal of order of rates that you do for Lewis reactions; and the reason would be that a horrible base (like iodide) still won't be able to hold on to a proton. Someguy1221 (talk) 22:58, 1 June 2008 (UTC)[reply]

Ok,i see.Thanks.Though i doubt i'll do chem hons. i wish they'd make our plus II text bks better.Its an interesting subject really!Abhiroopron (talk) 15:50, 2 June 2008 (UTC)Abhiroop[reply]

Properties of silk dope

Hi - our articles are a bit vague on the actual properties of the protein solution that passes through spinnerets to produce silk. Is it called 'silk dope' or is there a more technical general term? What does it look like en masse? What is its colour, viscosity etc?

Thanks Adambrowne666 (talk) 17:58, 31 May 2008 (UTC)[reply]

Which articles? The spider silk article has some good information. The properties of the silk ingredients are rather complex (the protein sequences are different depending on the species, and only a few have been decoded), and describes various attempts to synthesize it, with references for further reading. ~Amatulić (talk) 19:45, 31 May 2008 (UTC)[reply]
Yes, it's a good article, and thanks for the answer, but it doesn't address my questions above - what are the properties en masse of silk dope - if you had a cupful, what would it look/smell like? - and is there a more technical generic term for the stuff? Adambrowne666 (talk) 00:18, 1 June 2008 (UTC)[reply]
Otherwise known as gossamer. Seriously, where did you find the idea of "silk dope"? Julia Rossi (talk) 00:29, 1 June 2008 (UTC)[reply]
I get it, here at Softpedia[48] seems to refer to the fibre "stock" before being spun into thread. Julia Rossi (talk) 00:33, 1 June 2008 (UTC)[reply]
It's mentioned too in the Synthesis section of the Spider Silk article, but dope seems to often be contained in inverted commas - just thought there might be a more official generic term - and a description - I realise it's a difficult question... Adambrowne666 (talk) 04:48, 1 June 2008 (UTC)[reply]
Not difficult if the scientists were more pictorial, but here[49] and searching "synthetic spider silk stock", gives terminology of "stock" and "stock solution". Only found chemical descriptions (polypeptide etc), but no sensate accounts (smell, texture, colour) among these. Synthetic spider silk "stock" etc gets more hits than "dope" so I'd go with the stock/solution term which as you know, is the bulk before drawing out fibres as such. Interestingly there are some mixtures of both natural and synthetic stock. It's freezable, too. Cheers, Julia Rossi (talk) 06:37, 1 June 2008 (UTC)[reply]

Thanks for that, Julia - and thanks, all, for your efforts - I'm starting to realise it's a ridiculous question, like asking what a gallon of pure DNA looks like. Adambrowne666 (talk) 00:52, 2 June 2008 (UTC)[reply]

Asthma + Other disease

When I was three, I had an athsma attack with another disease at the same time. I do not remember the name of the disease, but I could not breath in or out. The last I remember of that day was falling asleep after a mask was put on me, leaving me to believe I had surgery. Ever since then, I have only had acute bronchitis, not general asthma. Does anyone in the medical field know what treatment could possibly lead to this(As well as what the disease could be)? I have a highly curious mind, so semi-complex awnsers are okay, just try not to get all E=MC2 over me. 67.171.165.4 (talk) 20:34, 31 May 2008 (UTC)[reply]

Do you mean chronic bronchitis? --Tango (talk) 21:22, 31 May 2008 (UTC)[reply]
We're not really supposed to give medical advice here but i'll give you some background, I suppose. Have you been confirmed to have bronchitis, or was it a disease similar to it? I ask this because both acute bronchitis and chronic bronchitis are pretty much always caused by an infection, be it by viruses or bacteria. The proportion of cases that haven't been caused by an organism are almost always due to breathing in industrial fumes, or even more commonly by smoking. I'm not sure it bronchitis can be caused by any physical treatment but i'll leave that to the more medically trained.
As for the insight into what the disease is, it's essentially an inflammation of the bronchi. It's normally treated by either antibiotics (in cases where a pathological aetiology exists) and/or by bronchodilators. Regards, CycloneNimrod talk?contribs? 12:41, 1 June 2008 (UTC)[reply]
With no medical background, but just based on "grandma says": child + trouble breathing + surgery brings tonsillitis to mind. That would however be an upper respiratory infection. Respiratory diseases has long lists of what it could be/have been. But that might just confuse matters further rather than clear things up. (No medical advice whatsoever.) Have you asked your physician? --76.111.32.200 (talk) 14:04, 1 June 2008 (UTC)[reply]
The user mentioned that they've had it ever since they were three, I highly doubt it was tonsillitis. Tonsillitis doesn't really lead to bronchitis. Regards, CycloneNimrod talk?contribs? 14:13, 1 June 2008 (UTC)[reply]

Help with my daughter

My daughter is going through puberty and is starting to develop acne. I'm trying to find research papers and information regarding the effect of physical exercise on acne. Thank you.--Goon Noot (talk) 20:36, 31 May 2008 (UTC)[reply]

Our article acne mentions many things that may cause or help counter acne, but physical exercise isn't one of them. I suspect therefore that exercise has little impact on it. -- SGBailey (talk) 21:08, 31 May 2008 (UTC)[reply]
We can't give medical advice here, so there's not a lot of help we can offer. You would be best off consulting your GP. --Tango (talk) 21:21, 31 May 2008 (UTC)[reply]

I did not ask for medical advice. I specifically asked for research papers and information. Not advice.--Goon Noot (talk) 22:10, 31 May 2008 (UTC)[reply]

Try this search string [50] which seems to get a lot of relevant hits. however, most of them require a subscription to view the full article. SpinningSpark 22:46, 31 May 2008 (UTC)[reply]
Here's one you can get for free: [51] - if you come up against a journal requesting a subscription or fee, you can always check to see if the research institution has archived the paper for free on the internet. Just yesterday, Nature tried to fleece me for $32 but I found that the research institution in Germany puts their papers online for free. You also ought to lobby your political establishment and research charities to mandate open access since its the people's donations and taxes that support so much research. ----Seans Potato Business 12:59, 1 June 2008 (UTC)[reply]

Muscle Growth

I know this isn't probably the best place to ask for exercise advice, but I had some questions about the notion of between-lifting rest. I know you're not supposed to lift weights every day, because you need to give your muscles time to rest and regrow. But I've also heard that there are some muscles that it's OK to work every day (like, it's OK to run daily, I think), and I've also heard that abs work like this. I've also heard something, which doesn't make much sense to me, about how body-weight exercises (push ups, pull ups, etc) can be done daily, for some reason possibly having to do with lightness of weight. Do any of you know, from personal or scientific knowledge, how much of this is accurate, or what the real story is?

And, regardless of what particular muscles and exercise types call for what types of rest, what are the deleterious effects of not getting enough rest time? Will my muscles grow less if I do pushups daily than if I did them every other day? Will the muscle be more dense? Bigger? More wirey? I don't know.

And which is it that causes denser muscles, heavy weightlifting or many reps of lighter (bodyweight etc) weights? Or is the different-muscle-type-by-different-exercise stuff I've heard about pretty much made up?

Thanks,

70.108.222.173 (talk) 21:51, 31 May 2008 (UTC)[reply]

Running every day is mostly to build endurance and reduce fat. It may actually reduce muscle growth though if done excessively. I don't know about muscle density, but more reps means more tone, while more weight means bulkier muscles. Hope that helps. 24.46.50.159 (talk) 23:11, 31 May 2008 (UTC)[reply]

Genome and DNA

So a Gene is part of a DNA molecule. And a Genome is a complete set of Genes in a given organism. So does a single DNA molecule contain the entire genome of an animal? 24.46.50.159 (talk) 23:08, 31 May 2008 (UTC)[reply]

The pairs of chromosomes (which are the same in all cells of the body) contain the entire genome of an animal. Wisdom89 (T / C) 23:11, 31 May 2008 (UTC)[reply]
A chromosome just contains one DNA molecule right? So just one cell will have all the chromosomes and DNA? How many chromosomes does a human have? 24.46.50.159 (talk) 23:28, 31 May 2008 (UTC)[reply]
A normal human has 23 pairs of chromosomes. Algebraist 23:32, 31 May 2008 (UTC)[reply]
I see. When you say pairs of chromosomes, do you mean 46 DNA molecules? 24.46.50.159 (talk) 23:55, 31 May 2008 (UTC)[reply]
Chromosomes are not the only DNA molecules in cells. According to chromosome, a normal human cell also contains hundreds of copies of the mitochondrial genome. Algebraist 23:59, 31 May 2008 (UTC)[reply]
And the genome isn't just the genes, it's all genetic information, coding and uncoding. - Nunh-huh 00:01, 1 June 2008 (UTC)[reply]
Nunh and Alge are absolutely correct. To answer more directly the last question, however, what we mean by "23 pairs" is 46 total chromosomes (see diploid), one copy from each parent. There are 22 pairs of autosomes and two sex chromosomes. Essentially, yes, 46 DNA molecules I suppose. But, that's misleading since "DNA molecules" can mean different things depending on the species we're talking about, the cell type (gametes are haploid for instance), or whether we're talking about in vivo or in vitro analysis. Wisdom89 (T / C) 00:35, 1 June 2008 (UTC)[reply]

June 1

Are scientists not telling us everything?

I've always had a suspicion that there are some scientists that know... THE TRUTH. The very unpleasant truth,- and aren't telling. For good reason. Dont wanna cause mass panic or anything. What I mean is-I'll bet there are some scientists that know, just know what's really going on.

Take the age old mystery of what happens after you die. Maybe it's very unpleasant. For EVERYONE. Maybe there is no Heaven or Hell. Just Hell. Or something like it. Doesn't matter if you've been a good person. Maybe that's just the way it is. For reasons we'll never understand. Bad things happen to undeserving people all the time, so why couldn't this "injustice" continue in the next realm?

How about where our thoughts come from. They appear at random all the time. Same for dreams. Would you really want to know that our minds are being programmed by aliens that don't necessarily have the best intentions? I doubt researchers would happily share this information with us.

I could go on but you probably don't want to get me started on mutating viruses, genetic slavery, and the insignificance of our existence. On second thought, maybe I dont want to know the answer.--Dr. Carefree (talk) 00:30, 1 June 2008 (UTC)[reply]

Well, we tell people about global warming and the end of oil and the death of the oceans and the consequences of overpopulation, and nobody listens who matters... --BenBurch (talk) 00:59, 1 June 2008 (UTC)[reply]
Science is one of those things that only rewards those who tell things—you get no points for keeping things secret, for good or ill. If I (a researcher, of a sort) knew aliens were programming our minds I'd definitely tell. If I thought I had evidence of anything massive and important I'd publish it, become famous, get a great job, write a book, be on Oprah, etc. Would I worry that people couldn't handle the truth? Not at all. Am I unique? No. There are scientists working right now, right now!, who, if they had new answers (based in science, not just speculation) to the questions of the afterlife, extraterrestrials, dreams, etc., they would rush, RUSH to publish it before somebody else did. That's not just Nobel Prize quality work, that's #1 Scientist of the Century sorts of stuff (what would Einstein's redefinition of spacetime have on something like that?). So no. I don't see it as likely. I'm sure many scientists have uninformed (from the point of view of scientific method) opinions on these issues, and they surely keep them to themselves. But actual access to the truth? That they're hiding away from us all? Systematic conspiracy? No way. Too much at stake for the individual. --98.217.8.46 (talk) 01:15, 1 June 2008 (UTC)[reply]
Also, please note that those questions you are asking are outside of realm of science, because they are not verifiable. If you want answers to these things, you should look into religion, because no real scientist will give you authoritative answers on those, as it is outside of his/her specialty (and interest, in most cases). Samohyl Jan (talk) 01:21, 1 June 2008 (UTC)[reply]
I read an article in a journal recently about how popular culture has made scientists look more and more evil through time. See the "Popular culture" section of Frankenstein (this section cites the article I'm talking about.) It's natural in our day and age not to trust scientists, even if there isn't any logical reason for it. Wrad (talk) 01:28, 1 June 2008 (UTC)[reply]
Hmm... minor quibble: understandable? Perhaps. Natural? No. – ClockworkSoul 05:07, 1 June 2008 (UTC)[reply]
To argue that religion answers what science does not is to argue for a god of the gaps. I think that science HAS provided definitive answers to the purpose of life and the state of death, and I don't believe there to be any realm of knowledge but what can be attained through the scientific method. Imagine Reason (talk) 21:47, 1 June 2008 (UTC)[reply]

Thing is,- if a scientist ever tried to talk about anything I just mentioned (or worse) I think he'd be labeled a crackpot. I also believe that if he did have absolute proof of something that terrifying and tried to go public with it,he'd probably "disappear" long before being labeled a crackpot anyways, IMHO. I was just kinda hoping that some anonymously editing scientist (using an untraceable proxy, perhaps?) would have the bravery to say what's really happening!Dr. Carefree (talk) 01:32, 1 June 2008 (UTC)[reply]

To prove anything through scientific means including what you mentioned earlier, scientist would have give strong empirical proof that something like that happens or is happening. Without verifiable proof through repeatable experiments, there is no science and hence, no scientist (that's why they are called "crakpots"). Oh, and nobody has "absolute proof" of anything, only "pretty good proof" at best. Sjschen (talk) 02:17, 1 June 2008 (UTC)[reply]
Can we imagine similar things that would "rock our world" that have been discovered, publicized, etc.? Sure. How about the fact that time and space are relative? That was pretty radical at one point. How about the fact that our brains have bits for religion hardwired into them (and they are the same parts of the brain that flash when epileptics have fits)? Big book money in that research. How about the fact that humans are descended from apes? Sure, it caused one scientist enough fear that he decided not to publish on it for a few decades—but then when it became clear that another scientist was going to make his own name on it, he rushed it into publication for posterity's sake. How about the fact that the splitting of atoms can create fantastic explosions? Yes, one scientist tried to keep others from publishing on the subject lest their common war enemy found out, but he also was more than willing to use that information where he thought it would have done good (and again, even if all of those scientists had stopped publishing completely, it would have gotten out eventually). All I'm saying is, we've had "world changing" research, examples of scientists being afraid or unwilling to publish, etc., and in the end, it all came out. It's also the case that you could remove any of the "top scientists" from history and someone else would eventually fall into place as the "discoverer" of this or that. Science is both dependent on the strivings of individual as well as independent of the work of any given individual. I don't see much possibility for real long-term coverup. Truth will out, as they say. --98.217.8.46 (talk) 04:00, 1 June 2008 (UTC)[reply]
The problem with such conspiracy theories is that they depend on governments, commercial organisations or other large groups of people keeping big secrets secret. Experience shows that people are just not that good at keeping secrets. On the other hand, if your conspiracy theory involves telepathic mind control by omnipotent aliens, it becomes much more credible ... Gandalf61 (talk) 09:55, 1 June 2008 (UTC)[reply]
The only way to keep a big secret is to limit the number of people who know about it. The CIA was caught completely off guard by the first Indian nuclear test in 1974 in part because they kept the number of staff super small—their top scientists were also their technicians, there were no more than a few dozen people involved directly. By contrast, the massive Manhattan Project, supposedly a well-kept secret, had numerous, numerous leaks, inadvertent releases, etc. not to mention the whole espionage thing; entirely predictable, given that there were thousands upon thousands of people involved. --98.217.8.46 (talk) 16:36, 1 June 2008 (UTC)[reply]
I disagree about death without consciousness being unpleasant (the idea of a bodiless floating soul is very discomforting for me), but I think people do realize on some level that science HAS provided the answers to questions about life and death, and that is why religionists seize upon Einstein's comment about science without religion. I also think theists are not wrong when some of them say that evolution is godless. Imagine Reason (talk) 21:43, 1 June 2008 (UTC)[reply]
Dr. Carefree, you seem to be quite paranoid, perhaps you should read up on dopamine hypothesis of schizophrenia (also related to paranoia and psychosis). Of course this theory is proposed by scientists.. the same ones the government are controlling ;-). --Mark PEA (talk) 23:43, 1 June 2008 (UTC)[reply]
The original question implicitly suggests that all of scientists are in collusion to prevent access to certain information. While there is a great deal of consensus regarding many facts in science, it is unlikely that anyone could make all scientists everywhere agree about this sort of censorship. Wikipedia's stance is pretty much the same - there is no cabal!. Nimur (talk) 16:31, 2 June 2008 (UTC)[reply]

I dont mean to imply that all scientists are in on hiding unpleasant information. What I mean is something along the lines of- Let's say a small group of , or even an individual scientist confirms that nature is indeed out to get us and will eventually get us in the long run. All good is an illusion, we're completely insignificant, we're all the the playtoy of sadistic extraterrestrials, and death is no escape. I highly doubt that any scientist would be allowed to go public with this information. First, he'd be discredited, and if that didn't work, he'd probably be found dead from a "suicide".

As a more practical example, do you think if it was discovered that a nearby star will explode tomorrow, killing us all (Eta Carinae?), there would be a big announcement about it? Supposedly the radiation from that star wont affect us. How about the front page news of a meteor hitting Earth in 30 years or so? Then, all of the sudden, the orbit was recalculated and, don't worry folks, it's gonna miss us. Somewhere, I see those scientists running around thier lab in a zigzag pattern, saying over and over again "Oh my God, oh my God what are we gonna do!!? It's 30 years away, Calm down!"

I'm not sure I'd want to to know the truth about any of this stuff anyways, but an interesting question, -no?Dr. Carefree (talk) 17:06, 2 June 2008 (UTC)[reply]

There was a brief scare that 99942 Apophis was (relatively) likely to impact Earth. And they published that almost instantly.
The primary motivation of research science is to publish. It's how you move up in your field and pay the bills. To suggest that a scientist is sitting on the biggest discovery in the world and hiding it, is like saying McDonalds has invented the tastiest hamburger in the world but is refusing to sell it. It doesn't make sense. Calories be damned, McDonalds would rush that burger to market so they could beat Burger King to the punch. Same with science, If researcher X hides information he loses out to researcher Y who publishes. APL (talk) 18:05, 2 June 2008 (UTC)[reply]

Need to see parts of a pig

Trying to figure out the different parts of pork. Is the Shoulder Roast the same as a Butt Roast? —Preceding unsigned comment added by 71.139.29.177 (talk) 02:05, 1 June 2008 (UTC)[reply]

Well shoulder mean the pig's front leg and butt means the back, so, no. Franamax (talk) 02:09, 1 June 2008 (UTC)[reply]
[Here's a link to a diagram showing the cuts of pork. -- Flyguy649 talk 04:05, 1 June 2008 (UTC)[reply]
Is the Shoulder Roast the same as a Butt Roast? Yes, check out the text and diagrams here . Pork#Cuts GameKeeper (talk) 05:20, 1 June 2008 (UTC)[reply]
Shoulder roast the same as butt roast? Seems like butchers don't know their arse from their elbow ;-) Fribbler (talk) 23:21, 1 June 2008 (UTC)[reply]

Non-quite-black snake in central Florida?

Today I fished a little "Black Racer" snake out of our pool; the poor thing was only about a foot long and couldn't lift himself far enough out of the water to climb the 6" tile lip, being all wet and slippery and all. It took me awhile, since he kept trying to get away from me, but I finally won that fight and gently deposited him in the grass where he could go back to eating flies, mosquitoes, fleas, and baby cockroaches like he's sposta. Only, as I was putting him down I realized he had a bright red ring around his neck. WTF? Grass snakes, in my experience, come in various flavors of green and black, but each individual snake is always the same color from nose to tail. Thus, I wonder if this might have been something I shouldn't have released again. I'm familiar with all the snakes up in Appalachia, but I ain't home no more, and I won't claim to be an expert on Florida snakes. I couldn't find anything on Google Images that looked right. The closest I've found is a color photo of a "Ringsnake" in the nonpoisonous section of Maynard Cox's "Protocol" handbook, but that shows a black body, yellow stripe, and red tail, so I don't think that's right. Any ideas? -SandyJax (talk) 02:41, 1 June 2008 (UTC)[reply]

I think it is indeed a ringneck snake. There are several subspecies, and the one you saw in the picture doesn't sound like the one you found. However, the top one on this page does look like it: [52]. StuRat (talk) 14:08, 1 June 2008 (UTC)[reply]
The top picture on your link looks exactly right, except that if it had a colored belly I didn't notice it, and it was wiggling around quite a bit, both in the pool and on the skimmer I finally used to pick it up. I probably would have seen any change in body color if the belly was different from the black head, body, and tail. Naturally, I had no camera with me when I went out to clean leaves and bugs out of the pool.... -SandyJax (talk) 22:22, 1 June 2008 (UTC)[reply]
This link [53] seems to reinforce your suggestion, and says that mine was full-grown. Huh. I guess I just missed whatever bright color was on the belly. I like the quote: "Ringnecks are the snakes that are most frequently found in Florida swimming pools — they crawl in to get a drink and then cannot climb out because they are too small to reach the lip of the pool. If you find one in your pool, lift it out with the leaf skimmer or a dipnet and turn it loose in the shrubs where it can get back to eating things you do not want in your garden." -SandyJax (talk) 22:37, 1 June 2008 (UTC)[reply]
Not the brightest of animals to climb into a pool before checking to see if there's a way out, are they ? StuRat (talk) 08:27, 2 June 2008 (UTC)[reply]
Another comment: Unless you are an absolute expert on snakes I'd stay away from them, as some, especially those with bright colors on them, are poisonous. Or, at the very least, make sure there is somebody with you who can get you medical attention if you're bitten. StuRat (talk) 14:15, 1 June 2008 (UTC)[reply]
I respectfully disagree. No, you can't stay away from strange things in your own yard. That just leads to someone less capable of dealing with it getting surprised. I can ask the wife & kid to stay away, and try to keep the dogs inside until I take care of it, and put the cats out in hopes that they will get eaten, but I'm the guy who has to deal with the problem. I can't even ask for help from the neighbors. The 4 nearest houses I see from my front door all have old retired people in them; if there's a problem like this _they_ will come to _me_ for help. As above, I will claim to be an expert on Appalachian snakes, but certainly not everything around here that may have escaped from someone's pet cage. Why d'you think I have a copy of Maynard's "Protocol" handy, and then ask for help when I can't ID a snake? -SandyJax (talk) 22:22, 1 June 2008 (UTC)[reply]
I got the impression, though, that you handled an unidentified snake with your bare hands prior to looking it up in the book and posting your question here. That's the dangerous part. You should treat any unidentified snake as if it were poisonous. Using the pool skimmer is a good idea, so it can't bite you. I believe there is also a device sold for larger snakes that has a lasso on the end of a long pole; you can tighten the loop around the snake by pulling the other end (does anyone know the name of this device ?). If you need to transport the snake you then need to deposit it into a thick bag designed for this purpose. I believe it's also far more effective to capture snakes with an assistant, as one can distract it with a long stick while the other maneuvers behind it and drops the loop around it's neck. I certainly hope that if you get a 20 foot alligator in your pool some day you won't try to handle it yourself, as you could give him indigestion that way. :-) StuRat (talk) 08:27, 2 June 2008 (UTC)[reply]
Not a problem. If someone asks for help, you are certainly right to include all the safety tips you can think of; any that don't apply will be happily ignored. The surface of a pool is commonly skimmed using a device that looks like a tennis racket; it attaches to a variable-length pole for the central parts that you can't reach from the edge of the pool. Unfortunately, Mr Wiggle-worm wouldn't stay on the skimmer. I ended up using a handy 5' inflatable raft because I don't know what it is yet. It's about a foot long; I grew up with rattlesnakes, cottonmouths, water moccasins, and gar, so I'm not letting it within 2 feet of my dainty little toesies without ensuring I'm in charge. If I'd used my bare hands, I could not have used the adjective 'gently' originally, as holding it behind the neck firmly enough to ensure it could not bite me (hadn't ID'ed it yet, right?) probably would have killed it. 20' alligators are best skimmed with a tool from Smith and Wesson. -SandyJax (talk) 10:14, 2 June 2008 (UTC)[reply]
Agreed, although I wouldn't handle a snake with bare hands even if I was certain it's wasn't venomous, as a snake bite could still lead to a nasty infection. StuRat (talk) 10:31, 2 June 2008 (UTC)[reply]
Well since you are an expert, it's not an issue but I disagree that you not doing it leaves it to someone less capable to do it. The responsible thing to do when you find a snake in your backyard, particularly when you think it may be venemous is not to leave your kids/wife/neighbour/cats to deal with it, but call up people who are experts are dealing with it. A Google for this [54] came up with [55] which has a link to [56]. I presume in most other parts of the US as well, there there are pest control or wild life removal experts who can similarly remove unwanted snakes (or other creatures) for you, obviously for a fee. Nil Einne (talk) 14:34, 2 June 2008 (UTC)[reply]
One other comment: You might want to keep the pool covered when not in use. Aside from the obvious advantages of keeping leaves and snakes out, the more important reason is to keep small children out. A surprisingly large number of children manage to drown themselves each year by falling into an unattended pool, so much so that covering vacant pools is the law in many areas. I don't know how old your kid is, but all those elderly folks nearby might have grandchildren over occasionally who might wander off and fall into your pool. StuRat (talk) 15:50, 2 June 2008 (UTC)[reply]

Need Bird Expert

There are a pair of birds nesting in my yard that look like Eastern Phoebes. Except that the Eastern Phoebe supposedly separate when their babies are still eggs, and though the chicks of the pair in question have hatched long ago and already have feathers, they continue to guard the nest together. Is there some other species that looks like the Eastern Phoebe and has similar range that I could be mixing these birds up with?

(More detailed description of the birds: about the size of sparrows. Grey-brown color all over except for white throat and belly. Black beak. No markings around the eyes, and no distinct wing bars although there are some very very faint markings on wings. Frequently flicks tail. Call is short high chirping sounds. Exactly five chicks in nest. The parents always appear to be within view of the nest, frequently together, and both have been seen flying to nest and back often in quick succession of each other.)

In case my question isn't clear: I am looking either for an explanation as to why this pair didn't separate or help identifying them as a different species than the one I've been suspecting. 96.233.8.220 (talk) 05:48, 1 June 2008 (UTC)Phoebe[reply]

Sorry I'm no bird expert. But the black and white pattern on the wings, white chest and dark head is also found in some tits The bird kind. There are just an awful lot of varieties of those and I may be off entirely.--76.111.32.200 (talk) 09:21, 1 June 2008 (UTC)[reply]
I too am no bird expert, but I recall from the Boy Scout days of my youth that the eastern wood-peewee can be difficult to distinguish from the phoebe. I was taught this: If the bird continually bobs its tail up and down while perching, it's a phoebe; if it doesn't, it's a wood-peewee. Deor (talk) 11:33, 1 June 2008 (UTC)[reply]
Here's the WP article on the wood-peewee. Note that it says that both parents raise the chicks. Your description of an all-black beak, tail flicking, and indistinct wing bars would seem to rule it out; but are you sure of your observations? Deor (talk) 11:42, 1 June 2008 (UTC)[reply]
Assuming that your description of the behavior and appearance of the bird is correct (especially no wing bars and tail flipping), it must be an Eastern Phoebe. While it is true that phoebes tend to be loners, both parents do tend the nest according to this site [57] (and our article Eastern Phoebe). Continue your observations and let us know what happens. Remember that animal behavior can vary quite a bit, and what you read in books or on the Internet is just a summary of the behavior of many individuals.--Eriastrum (talk) 15:46, 1 June 2008 (UTC)[reply]

Refererencing

"The CKTFS1B1 protein was shown to interact with YWHAH (or 14-3-3 eta) (4)⁠, a protein which blah blah blah..." - notice where it says (4) after '14-3-3 eta'. 14-3-3 eta is the alias of YWHAH whilst '4' is the reference number. Is it appropriate to leave '4' in brackets or should it be '(or 14-3-3 eta; 4)⁠' as one usually does when one has two sets of brackets adjacent to each other? I'd say that the purpose of the '4' in the second case is not clear. ----Seans Potato Business 11:41, 1 June 2008 (UTC)[reply]

Generally having consecutive parentheses is awkward. If it were full referencing, your second solution works (14-3-3 eta; Widget et al., 2008). However, I've never seen that with parenthetic numerical referencing. Why not recast is as "... YWHAH, also called 14-3-3 eta (4), a protein..." or move the reference to the end of the sentence? -- Flyguy649 talk 14:13, 1 June 2008 (UTC)[reply]
I decided to recast the sentence (I couldn't put the reference at the end of the sentence because there lies another reference and I wanted to make it clear as to which morsel came from where). Thanks. ----Seans Potato Business 19:01, 1 June 2008 (UTC)[reply]

Why does wintergreen oil dissolve plastic ?

It doesn't seem like nasty stuff, I can even put my hands in it with no ill effect (although it apparently can be lethal in large doses). So why did it dissolve the plastic case on my walkie-talkie ? StuRat (talk) 13:41, 1 June 2008 (UTC)[reply]

"mild solvent" isn't particular meaningful. Different things dissolve in different things. Water is quite a powerful solvent for things like sodium chloride, it doesn't do much at all to dissolve plastic, though. --Tango (talk) 19:57, 1 June 2008 (UTC)[reply]
True. So what makes things like methyl salicylate work as solvents? I couldn't find much about it in that regard other than the mention that oils like this dissolve some of the plastic in water bottles and such. --jpgordon∇∆∇∆ 23:01, 1 June 2008 (UTC)[reply]
Doesn't it have something to do with the miscibility of similar objects (ie. two non-polar like the oil and plastic or two polar like water and sodium chloride)? --71.117.35.118 (talk) 04:10, 2 June 2008 (UTC)[reply]
That may be part of it, but certainly not all polar substances dissolve each other and all non-polar substances don't dissolve each other, either, so something else is clearly going on here. StuRat (talk) 17:11, 2 June 2008 (UTC)[reply]
Wintergreen (aka salicylic acid methyl ester, oil of wintergreen, betula oil, methyl-2-hydroxybenzoate) is more than just oil. I accidentally "etched" a clear plastic clock face, wiping it with lavender oil. Using olive oil would have been harmless to the plastic though it dissolves some label glues. Is the resident chemist able to tell what the property is that dissolves stuff? I mean, etches the surface? Julia Rossi (talk) 08:45, 2 June 2008 (UTC)[reply]

Do Horses and Zebras have more in common than Humans and Chimps?

Genetically speaking? 24.46.50.159 (talk) 14:06, 1 June 2008 (UTC)[reply]

Probably, as they can interbreed, I think. Donkeys seem even closer to zebras than horses, though, in the stout build and braying sound they make. StuRat (talk) 14:18, 1 June 2008 (UTC)[reply]
Interesting question. I'm not sure about genetics, but in terms of time, horses and zebras are closer. According to zebra, horses and zebras split off about 4 million years ago, according to hominini, "the [human/chimp] divergence was completed between 5.4 to 6.3 million years ago, after an unusual process of speciation that ranged over four million years." It's not a big difference and time is far from the only thing which affects evolution, but it gives you an idea. StuRat's point that horses and zebras can interbreed is a good one, although I don't know of any attempts to interbreed humans and chimps, so we can't be sure it's impossible! --Tango (talk) 14:27, 1 June 2008 (UTC)[reply]
I'm no biologist, but isn't it worth comparing number of generations, instead of number of years? If zebras reproduce much more rapidly than primates, they would have had more generations, even in a shorter time, allowing for greater biological divergence. Our article says zebras may have a foal by age three. This suggests they've easily had far more generations in 4 million years than we primates have in the 6.3 million years since common ancestry. Nimur (talk) 16:38, 2 June 2008 (UTC)[reply]
That's a good point, but biological divergence isn't strictly proportional to number of generations, either. The evolutionary pressure on the two populations to change is important. With no pressure to change, two isolated populations may have nothing but minor changes even after millions of generations. And this assumes 100% isolation. If there is any opportunity, however limited, for interbreeding, this could easily counter random mutations in two populations with no evolutionary pressure to differentiate. StuRat (talk) 17:01, 2 June 2008 (UTC)[reply]
There have been attempts, none successful. See humanzee. --98.217.8.46 (talk) 16:42, 1 June 2008 (UTC)[reply]
Although none of these refer to attempts to inseminate a human ovum/female with chimp sperm (at least from a brief search, there were plans but none were carried out). Since these AFAIK sometimes show apparent differences because of the gametes (i.e. unrelated to the problems the differences in the organisms may cause), these would provide a more complete picture, although I think we can say it's rather unlikely Nil Einne (talk) 18:23, 1 June 2008 (UTC)[reply]

Shuttle SRB exhaust composition

What is the approximate composition of the exhaust from the APCP Space Shuttle Solid Rocket Boosters? It looks like HCl was a large component of the output in 1991, although I've found a US government patent from around that time giving a method for reducing this. Angus Lepper(T, C, D) 14:15, 1 June 2008 (UTC)[reply]

Ammonium perchlorate: NH4ClO4 is the both the fuel and the oxidiser here so ammonium is oxidised, and perchlorate is reduced. The products could be NO2, H2O and Cl2, these are probably the main components, but there might be, as you said HCl, maybe ClO, H2(this would be oxidised to water anyway) and probably other nitrogen oxides. (I haven't found a reference for this, I'm just assuming, it is only rocket science :D)--Shniken1 (talk) 04:17, 2 June 2008 (UTC)[reply]

Word jumble

Are dyslexia suferers actually better at solving word jumbles, being used to having to decipher jumbled letters ? Since we seem to lack an article on word jumbles, it's when words are presented with the letters mixed up, and you need to figure out the word, like CUKIQ for QUICK. StuRat (talk) 14:30, 1 June 2008 (UTC)[reply]

  • An anagram typically has meaning in the unscrambled form, unlike a word jumble. The Wikipedia article is apparently just called Jumble, which is confusing, as many things other than words can be jumbled. I will add a redirect from Word Jumble to Jumble. StuRat (talk) 15:05, 1 June 2008 (UTC)[reply]
I remember reading two different studies, one a couple years ago and another at least 15 years ago, that found a strong correlation between children who were taught phonics as a primary tool for spelling and children's ability to solve jumbles. Both assumed that the children don't attempt to rearrange random letters. They group the letters into phonics and rearrange those. -- kainaw 19:12, 1 June 2008 (UTC)[reply]

what is water resistance

give some examples —Preceding unsigned comment added by 92.0.151.133 (talk) 19:22, 1 June 2008 (UTC)[reply]

Well, it's resistance caused by water. Anything moving through water will experience it, so I'm sure you can think of some examples. --Tango (talk) 19:52, 1 June 2008 (UTC)[reply]
Wouldn't be a homework question would it? SpinningSpark 20:38, 1 June 2008 (UTC)[reply]
Viscosity or Waterproofing? Julia Rossi (talk) 08:48, 2 June 2008 (UTC)[reply]

Pholidota

What is the ethymological origin of the name of this taxon? Thanks. Leptictidium (mt) 20:43, 1 June 2008 (UTC)[reply]

According to the OED, it's from the Greek pholidotos, 'covered with scales'. Algebraist 20:51, 1 June 2008 (UTC)[reply]

Question as topic. I've read stories of cat/rabbit hybrids but (AFAIK) they were all either hoaxes or deformed kittens. --Kurt Shaped Box (talk) 22:30, 1 June 2008 (UTC)[reply]

African elephant and Indian Elephant have hybridised, which is very surprising because the have different genera. See [58] & Motty. GameKeeper (talk) 22:59, 1 June 2008 (UTC)[reply]

There has been at least one hybrid between the Galah and the Cockatiel bred in aviary conditions. This is also an intergeneric hybrid - though as I understand it, the genetic relationship between cockatoo species is still very poorly understood and many of the placings within certain genera are hotly disputed. This may just be one of those "they were more closely related than we realized" cases. --Kurt Shaped Box (talk) 23:54, 1 June 2008 (UTC)[reply]

In a discussion not long ago on this desk, it was said that different genera of macaw can interbreed ([59]). Anyone know of animals from different families that can interbreed? --Tango (talk) 23:40, 1 June 2008 (UTC)[reply]
To answer my own question, according to Hybrid (biology): "Extremely rare interfamilial hybrids have been known to occur (such as the guineafowl hybrids)". I haven't found any more information about them, but it seems they do exist. --Tango (talk) 00:04, 2 June 2008 (UTC)[reply]
Ok, Wikipedia really does have an article on everything: Gamebird hybrids. It doesn't contain much more information, though. --Tango (talk) 00:11, 2 June 2008 (UTC)[reply]
I'm a little surprised no one joked about crossing beavers with ducks! --Wirbelwindヴィルヴェルヴィント (talk) 02:25, 2 June 2008 (UTC)[reply]

snakes in mt holly nc

we just found a black and white snake in our yard and we are not sure what kind it is. it has the look of a king snake but i'm not positive. what snakes are found in the area mt holly nc —Preceding unsigned comment added by 67.33.243.17 (talk) 23:14, 1 June 2008 (UTC)[reply]

Here's a nice web site with photos of snakes of North Carolina [60]. And here's a website on how to identify snakes in North Carolina [61]--Eriastrum (talk) 23:29, 1 June 2008 (UTC)[reply]

Feline Behaviour

Whenever my cat rubs the side of his face along my palm or around my leg, what is he telling me? Is he saying I'm marking you as my territory, or is he saying I trust you, or is he just saying that he's content being around me? He doesn't seem afraid when he does it but I don't know what it means. Althought it usually happens when he's hungry :P --Hadseys 23:35, 1 June 2008 (UTC)[reply]

When cats rub against you, they're marking you with their scent (secreted by glands, some of them on the face), so yes they are marking you as their territory. Other times, he is just showing affection, or wants attention. --DrVornado (talk) 00:13, 2 June 2008 (UTC)[reply]

SO its the kitty equivalent of ass-licking, sometimes? --Hadseys 01:02, 2 June 2008 (UTC)[reply]

Or the equivalent of a dog urinating on your hand... SGGH speak! 14:45, 2 June 2008 (UTC)[reply]
Cats have no trouble communicating 2 different things at the same time. Just ask our tom who after we got back from a trip purred his head off while hissing intermittently, telling us he was miffed we left him alone (with a sitter) and was happy we were back. Young wild cats rub their cheeks against their mother's head to make her drop prey she brings home. So to answer your original question, probably all of the above. She's marking you as a member of her social group (sharing scent marks), acknowledges that you are the one with the can opener providing food and is making sure no one is going to lay claim to the food you'll drop in her bowl. There are no clear distinctions between social, sexual or territorial behavior as some would like to draw. (That doesn't even work for humans.) There's some debate as to whether saying an animal is happy is anthropomorphism. Cats that are secure in their social position and territory certainly seem to just enjoy hanging out, although they do get off on being petted at times. (Don't tell auntie she'd be scandalized!) We are not that far apart, though, because most people will answer a cat's advances when it rubs against their leg with petting it, asserting they are part of kitty's social group. (And isn't it nice we have other terms like "happiness" and "enjoyment" for human feelings when petting the cat.) ^ω^76.111.32.200 (talk) 15:15, 2 June 2008 (UTC)[reply]

June 2

Battery Life in Hybrid Vehicles

Just how long can and do these batteries last? I know that all rechargeable batteries loose the ability to hold a charge over time, so this make me wonder how long the batteries that are used in hybrid vehicles can last. I've read some where that NiMH batteries have a lifespan of 400-600 charge cycles with Li-ion being around that number also, but I have no idea how true those numbers are. Any help would be appreciated. Deltacom1515 (talk) 00:19, 2 June 2008 (UTC)[reply]

Anything good here? Fribbler (talk) 00:44, 2 June 2008 (UTC)[reply]

what is the three intrinsic property of a pure substance?

--24.78.51.208 (talk) 00:33, 2 June 2008 (UTC)[reply]

I'm not sure that you could say that there are exactly three intrinsic properties of a "pure substance", so this sounds like it's probably a homework question that expects you to look at your textbook, or your previous notes, to where you've been told three properties. However, a look at chemical substance may offer a little assistance. Confusing Manifestation(Say hi!) 00:41, 2 June 2008 (UTC)[reply]

infact that's in review booklet and the question is which isn't a intrinsic property..a)densiy.b)color.c)Melting point.d)boilling point

i cant find answer for it--24.78.51.208 (talk) 02:17, 2 June 2008 (UTC)[reply]

Ask yourself this; which of those properties are you able to change? That's the one that isn't intrinsic. Also, don't think about it too deep - it's not that kind of question. SpinningSpark 02:29, 2 June 2008 (UTC)[reply]
See Intensive_and_extensive_properties). --Bowlhover (talk) 03:34, 2 June 2008 (UTC)[reply]
I think you may have confused him more Spinningspark.. (you confused me more). This appears to be a very poor question as all four of these can be changed by simple modifications of the system, but maybe, as you said, I am thinking about it too hard. So...I think it is Density as you just need to change the temperature to change this...that being said the other three properties can be easily modified.--Shniken1 (talk) 04:09, 2 June 2008 (UTC)[reply]
No, it's not density. You are right though, it is a rubbish question and my answer was possibly not much better, my excuse is I was trying not to directly answer a homework question. The article found by Bowlover explains it better. An intrinsic property does not depend on the amount of the substance so it is not density - that remains the same however much there is of it. SpinningSpark 04:39, 2 June 2008 (UTC)[reply]
I don't think an "intensive property" is the same as an "intrinsic property". If you think about it, even the boiling and freezing points require a certain quantity to measure, as those terms are rather undefined for a single molecule. I'd say the answer they are fishing for is indeed density, as that can be changed easily for materials in a gaseous form, simply by adding or subtracting some from the container, or by changing the temperature for a liquid or a solid. Changing the pressure will also change the boiling and melting temps, but those can't be the answer as the same logic would apply to both, and we're only looking for one answer. StuRat (talk) 16:36, 2 June 2008 (UTC)[reply]
Yes, density is easily changed with temperature. However, color can also change, e.g. sulfur, which turns red when heated above 200 C; there are probably special cases, but density is easily changed in any element that undergoes thermal expansion. Nimur (talk) 16:44, 2 June 2008 (UTC)[reply]
I agree that the question is total crap (can we give the teacher a failing grade ?), but I still think they are going for density. StuRat (talk) 20:46, 2 June 2008 (UTC)[reply]

If stars can't fuse past iron...

How did we get heavier elements than iron in our universe? 24.46.50.159 (talk) 01:39, 2 June 2008 (UTC)[reply]

They get generated in supernovae, if I remember correctly. -mattbuck (Talk) 01:51, 2 June 2008 (UTC)[reply]
(ec) See Supernova#Source_of_heavy_elements. --Milkbreath (talk) 01:52, 2 June 2008 (UTC)[reply]
So it means that the solar system was created out of an earlier supernova due to the number of heavy metals found here?--Lenticel (talk) 01:57, 2 June 2008 (UTC)[reply]
At least one. The common opinion is that at least some of the material on earth must have gone through more than one supernova to account for the relative abundances of elements. SpinningSpark 02:23, 2 June 2008 (UTC)[reply]
Note, however, that planets, and terrestrial planets in particular, manage to filter out most of the lighter elements and leave a highly disproportional amount of heavy elements. So, while our solar system is 99% hydrogen and helium, most of the hydrogen and helium on Earth and the other terrestrial planets has bled off into space, by rising to the upper atmosphere and being blown away by the solar wind. StuRat (talk) 16:26, 2 June 2008 (UTC)[reply]

liquid water versus water

I was watching a history channel documentary on the universe and a scientist kept talking about "liquid water". My family was mocking him for not just saying "water", but as the documentary went on we discovered all the scientists in it used the term. Why? What is different between water and liquid water? thanks experts. 64.231.202.9 (talk) 01:42, 2 June 2008 (UTC)[reply]

Well, it's about being precise. In scientific terms, water refers to the molecule H2O. In common terms, water refers to the stuff that comes out of your tap. To a scientist, this is liquid water. This distinguishes it from ice (solid water), as ice can be the solid form of many different compounds (dry ice for example); and also from steam (water vapour) as steam can again refer to the gaseous state of many different compounds. -mattbuck (Talk) 01:50, 2 June 2008 (UTC)[reply]
Also, since the documentary was on the Universe, the scientists may have been referring to environments in which water won't necessarily be liquid. Zain Ebrahim (talk) 01:56, 2 June 2008 (UTC)[reply]
Yes, we scientists (I rather like saying that) use the term "liquid water" to distinguish between liquid water, solid water, and gaseous water. – ClockworkSoul 02:51, 2 June 2008 (UTC)[reply]
I use the scientific terms as well, but often get laughed at by those who simply call them "water", "ice", and "steam". StuRat (talk) 16:21, 2 June 2008 (UTC)[reply]

Very strange pigeon picture....

WTF?

Just discovered this odd pigeon image whilst browsing. Does it look like a photoshop job to anyone else here? The position of the head looks completely anatomically impossible, to the extent that I was unable to even parse the content at first... --Kurt Shaped Box (talk) 01:48, 2 June 2008 (UTC)[reply]

Unless these two pictures and others from google images are all shopped, I think they might actually be real. The necks of these pigeons do seem pretty long, even though it still looks anatomically wrong to me with that knowledge. Guess their necks are just really flexible. (addendum) Found this and this image too, the latter makes me think that it's all the more likely it's possible, but maybe the patterns on the grey ones make it look like it's in a weirder position than it really is. --Wirbelwindヴィルヴェルヴィント (talk) 02:15, 2 June 2008 (UTC)[reply]

I suppose it's the pigeon equivalent of a human being able to touch his buttocks and the back of his head together. That said, some humans *can* do that. Maybe these birds were selectively bred - specifically so they could pose like that? Can't see the point myself, but those selective breeders do seem to like the weird and wonderful... --Kurt Shaped Box (talk) 02:32, 2 June 2008 (UTC)[reply]

Well, from what I can gather, "modern" fantail pigeons are the ones featured in these pictures, so I would guess also that it's selective breeding. --Wirbelwindヴィルヴェルヴィント (talk) 02:37, 2 June 2008 (UTC)[reply]
Yep. A lot of people just don't seem satisfied with normal pigeons. [62] --Allen (talk) 04:07, 2 June 2008 (UTC)[reply]
Given the taste of Bride of Wildenstein it's not surprising, since people often aren't satisfied with normal people either. Julia Rossi (talk)
It was quite popular in England in the 19th century to breed all sorts of bizarre and strange looking pigeons (it still is today, to a much lesser extent). Pigeon fancy was one of the many inspirations for Charles Darwin when thinking about the plasticity of heredity. That particular one just has a huge tail, and a ridiculously huge, puffed-up chest. It is "not natural" in the sense that the appearance did not evolve that way in the wild, it is a human intervention to make funny looking birds. --98.217.8.46 (talk) 15:52, 2 June 2008 (UTC)[reply]

redox

Okay, so I'm having a bit of trouble understanding the whole redox thing. Here's the question I'm supposed to be answering:

If copper wire is placed in AgNO3(aq).

a. Which element is going to try to lose electrons?
Copper
b. Write the redox equation between copper and silver
Cu + AgNO3 → Ag+ + CuNO3
c. What is the net voltage for this reaction?
.28V
d. Is copper wire the anode or cathode?
anode

So I'm wondering how well I did, thanks for the help! --71.117.35.118 (talk) 04:39, 2 June 2008 (UTC)[reply]

Now, I'll have to say that this definitely smacks of a homework question. Wisdom89 (T / C) 04:57, 2 June 2008 (UTC)[reply]
Well at least he made an effort in answering the questions.--Lenticel (talk) 05:07, 2 June 2008 (UTC)[reply]
It could be just a typo, but in part b., your equation violates charge conservation. —Keenan Pepper 05:18, 2 June 2008 (UTC)[reply]
Who cares if this is a homework question? They've made an effort and they're struggling so they want some help, that's absolutely fine. At least they made the effort. As Keenan said, your second part violates charge conservation but I think you just missed the + off the initial Cu. Regards, CycloneNimrod talk?contribs? 10:45, 2 June 2008 (UTC)[reply]
Nope, changing Cu to Cu+ does not make the equation correct. That's solid Cu in that wire, with no other elements involved, so it must be neutral. Something else must be involved to balance the charge... —Keenan Pepper 18:41, 2 June 2008 (UTC)[reply]

measuring fecundity

This line appears in Niall of the Nine Hostages:

In January 2006, scientists suggested that Niall may have been the most fecund male in Irish history, and second only to Genghis Khan worldwide.

They get this distinction not so much by fathering numerous children themselves (Niall is said to have had at least eight sons, which is remarkable but hardly record-setting) as through their later descendants. But then what about their fathers? Is there a way of weighting generations such that one can meaningfully say "This guy, this is the one who gets most of the credit for founding a proliferative lineage"? —Tamfang (talk) 05:29, 2 June 2008 (UTC)[reply]

I believe he's called Y-chromosomal Adam. 67.68.34.26 (talk) 09:41, 2 June 2008 (UTC)[reply]
...who lived long before Niall and Genghis. Thank you for playing. —Tamfang (talk) 19:30, 2 June 2008 (UTC)[reply]
Could it be that Niall and his father have the same amount of living descendants but Niall is a generation more recent? The only alternative that makes any logical sense to me is that Niall's father isn't a "male in Irish History". Stanstaple (talk) 12:50, 2 June 2008 (UTC)[reply]

Compressed hydrogen car

A disadvantage of hydrogen is that transportation requires compression/cooling, resulting in low efficiency. But can't this energy be won back? For example, like in a compressed air car, the pressure could be used to drive the pistons or a quasiturbine or whatever (or the wheels directly, as done by K'airmobiles). I don't know how risky it would be to use hydrogen in such a mechanical manner. (Such as sparks. Maybe use plastic parts for the mechanism?) Alternatively, instead of the pressure-difference with the surroundings, the temperature-difference could be used to generate energy, although I believe that can't be done very efficiently. What is the name for that technology again? DirkvdM (talk) 07:21, 2 June 2008 (UTC)[reply]

You could possibly get something back from those methods, but the question is whether hauling around the weight of the equipment to do so incurs more of a cost than the benefit. I would guess that it would. The one exception that comes to mind is using cooled liquid hydrogen to air condition the car. The advantage here would be that this method would replace the mechanical compressor and Freon system used currently, and quite possibly reduce the car's weight rather than increase it. Compressed hydrogen could also be used, as it provides quite a bit of cooling when decompressed. StuRat (talk) 16:15, 2 June 2008 (UTC)[reply]
It is true that there is mechanical energy which could be extracted from compressed Hydrogen. However, I think it would be a bad idea to use hydrogen for any sort of pneumatic system. It is highly flammable, and the probability of disaster would be significant. It's not a matter of "how risky" in absolute terms, it's "how risky relative to the alternative" (which would be, use any other thing except hydrogen in these systems). Putting flammable materials into a pneumatic system is tantamount to confining an explosive charge - should anything go wrong, the contained gases would pressurize the chamber, and firey metal fragments would fly everywhere - it would not be a good idea. Imagine using gasoline as brake fluid! It can be done, but it's far more dangerous than any alternative. Nimur (talk) 16:52, 2 June 2008 (UTC)[reply]
Yeah, I already mentioned the risk, but any risk can be dealt with (hell, nuclear energy is even being used all around the world and millions of people die in car accidents, and that doesn't stop us from driving cars). If the advantage is big enough, that is, and that is what I am asking this for. One of the main (principal) advantages of hydrogen is transportation, which requires compression/cooling and thus causes energy loss. So solving that could have big enough advantages that someone else will come up with a safer system. Let's take this one step at a time. :) DirkvdM (talk) 17:12, 2 June 2008 (UTC)[reply]
I agree with Dirk here. Sure "confining an explosive charge" is potentially dangerous, but that's the basic concept behind the internal combustion engine, after all, so it certainly can be done safely. Also note that hydrogen is only explosive once mixed with oxygen, before then it could absorb a lightning strike and not combust. StuRat (talk) 20:34, 2 June 2008 (UTC)[reply]
StuRat, I'm not talking about an add-on to existing cars, which would increase the weight. The original idea is to have a hydrogen powered car and then adapt that to use the cold/pressure, but you could also look at it the other way around. A compressed air car is so light that that is actually named as one of the disadvantages in the article. So the question then becomes if adding the hydrogen sytem would not negatively affect the car's performance through weight. But of course if either system works, then the combination should too.
But the main thing that is bugging me is what method might be used to make use of the cold. Or rather the heat difference between the tank and its surroundings. That represents energy, but I have no idea how to make use of that (other than for airco). Something that uses a heat pump would probably be way to heavy for a car. DirkvdM (talk) 17:12, 2 June 2008 (UTC)[reply]
I don't quite understand. If a car has weight X and only has equipment to utilize the burning of hydrogen, and you then add components to allow it to also take advantage of the pressure and/or cold of the hydrogen gas, then you're adding weight to the vehicle (unless you can remove something as heavy or heavier, like the A/C compressor and refrigerant storage system). This same logic applies whether the additional equipment is added after-market or at the factory. StuRat (talk) 20:30, 2 June 2008 (UTC)[reply]

Number one exporter of potassium

Now, we all know that according to Borat, Kazakhstan was the "number one potassium exporter in the world". But that isn't, I believe, true. Nor is it true that "all other countries have inferior potassium". So here is my question, as I cannot find the answer in the potassium article. Which country is the number one potassium exporter in the world? Based on potash (potassium carbonate), the biggest company is the Potash Corporation of Saskatchewan, so that would be Canada. But is potash the primary source of potassium? What about potassium chloride, potassium oxide or potassium hydroxide, all of which are used in fertilizers? A bit random, but I even thought of bananas - they have a very high potassium content. I did the math, and India (#1 producer of bananas - 16.8 million tons of bananas produced in 2005, at 358mg of potassium per 100g) produces 60,200 tons of potassium per year via bananas. I'm not sure if this is even close to the weight of potassium in fertilizers. Anyone know the answer? Who is number one potassium exporter in the world? Neıl 15:38, 2 June 2008 (UTC)[reply]

I would be quite surprised if bananas are worth more for their extracted potassium than as food. Are you asking who produces the most potassium sold as such, or are you including potassium found in other products ? StuRat (talk) 16:02, 2 June 2008 (UTC)[reply]
(after edcon)I'd say if you want to include potassium inside plants and animals it gets rather extensive. Potassium is an electrolyte essential for both plants and animals. So if you count bananas you'd have to look at all the other foods, too. e.g. [63] Just an example: although soybean only contain 149 mg potassium /100g, the US reported exports of 4.36 million tonnes [64] for just one week. Add to that exports in meat, rice and other foods. You'd then have to do that for all major exporters. Next you'd have to add compounds [Category:Potassium compounds] and again products that contain those, like e.g. toothpaste and softdrinks. Good luck.Lisa4edit (talk) 16:32, 2 June 2008 (UTC)[reply]
As a matter of interest, how exactly would you go about extracting potassium in elemental form from bananas? --Kurt Shaped Box (talk) 17:52, 2 June 2008 (UTC)[reply]
The traditional way, I think, would be to burn them to make potash (potassium carbonate), boil that with slaked lime (calcium hydroxide) to make potash lye (potassium hydroxide), then electrolyse that to get potassium metal. But there probably isn't a huge demand for potassium metal, so you might be better off leaving it at the potash stage to make fertiliser. --Heron (talk) 18:49, 2 June 2008 (UTC)[reply]

DREAMS

why do we dream? I have read that dreams occur due to the memories that had been stored in our brain during sleep. but i need a clear cut idea —Preceding unsigned comment added by Pinni smart (talkcontribs) 16:00, 2 June 2008 (UTC)[reply]

I don't think we know for sure. There have been many theories:
1) They are "random firings of nerve cells" while we're asleep.
2) They are a method for us to review what we've done and come up with better strategies for dealing with similar situations in the future. (I like this idea.)
3) They are attempts by our subconcious to communicate with our concious minds. (Sigmund Fraud loved this idea.)
4) They are attempts by God/the gods to communicate with us. StuRat (talk) 16:07, 2 June 2008 (UTC)[reply]
Check out Dream#Dream_theories. --98.217.8.46 (talk) 16:15, 2 June 2008 (UTC)[reply]
Maybe because 'directed thinking' (as described by Carl Jung) is tiring. Somehow dreaming recharges our batteries, so to speak. Em3ryguy (talk) 17:53, 2 June 2008 (UTC)[reply]

Burr oak

I posted a question concerning burr oaks last week and it is no longer posted. What happened???129.112.109.252 (talk) 16:33, 2 June 2008 (UTC)[reply]

What's your question? If it asked for medical or other advice it might have been deleted.Lisa4edit (talk) 16:37, 2 June 2008 (UTC)[reply]
After a week, questions are moved to the Archives. -- Coneslayer (talk) 16:44, 2 June 2008 (UTC)[reply]
Your edit history shows no such question, although you might have a varying ISP number. By all means ask again.--Shantavira|feed me 16:49, 2 June 2008 (UTC)[reply]
Might want to look at burr wood. Ziggy Sawdust 18:55, 2 June 2008 (UTC)[reply]
Bur oak?--Lisa4edit (talk) 19:03, 2 June 2008 (UTC)[reply]

thin film book

i need book about thin film, how can i find thin film book? —Preceding unsigned comment added by More55 (talkcontribs) 17:26, 2 June 2008 (UTC)[reply]

Nutritional information Aquafina Alive Satisfy water - specifically, what substance is the fiber?

This product states it has 3 grams of dietary fiber per 8 ounce serving. Reading the ingredients on the Pepsi website states exactly what the label on the bottle states....nothing that I can identify as being the substance that is the "fiber" in this water. I know that there are some types of seed husks that are used in Metamucil and the like.

What is in this water that is considered to be fiber?

--Dbacksfanbrian (talk) 17:53, 2 June 2008 (UTC)[reply]

Looking at the ingredients list from this site, the only ingredient listed that might contain dietary fiber is the NATURAL FLAVOR. Everything else is water, sugar, and assorted small-molecule chemicals. I'd say it's likely that those 'natural flavors' include some sort of pureed fiber source. I couldn't tell you what that was, however. TenOfAllTrades(talk) 18:56, 2 June 2008 (UTC)[reply]
Maltodextrin is considered a "dietary fiber" apparently. Lisa4edit (talk) 18:59, 2 June 2008 (UTC)[reply]

Propeller aeroplane under water

If a propeller aeroplane is made waterproof, will it work under water? What are the issues involved? --Masatran (talk) 18:20, 2 June 2008 (UTC)[reply]

Submarine what would be the difference? What changes do you propose? Lisa4edit (talk) 18:48, 2 June 2008 (UTC)[reply]
Do the gear ratio of the motor of the propeller, etc. have to be adjusted? If yes, how? --Masatran (talk) 18:55, 2 June 2008 (UTC)[reply]
Water is a heck of a lot thicker (more viscous) than air. If you start trying to spin an airplane prop at its normal speed, it will cavitate for a little while – if you're very lucky – and then snap off. TenOfAllTrades(talk) 18:58, 2 June 2008 (UTC)[reply]
Yea, it wouldn't work well. For water you need a shorter, thicker, slower prop than for air. I suppose you could run an airplane prop extremely slowly (less than one rev per second), and get some propulsion under water, but you would do better to have a separate prop for underwater use. Another issue is cooling, as airplane engines will commonly be air-cooled and won't tolerate being submerged in water. The wings will also provide a lot of drag under water. Finally, an airplane is way too light to submerge in water, it would float until it fills with water, then it would sink. So, this idea really won't work. However, an airplane that would operate on (or just above) the surface of the water could work well, as a ground effects aircraft. StuRat (talk) 20:17, 2 June 2008 (UTC)[reply]
Normal airplane engines work by burning fuel and therefore require an oxygen supply, which is in the form of air. See Submarine#Propulsion for discussion of underwater power supplies. --Anon, 16:50 UTC, June 2, 2008.

Mercury poisoning

If there were a liquid metal other than Mercury, would it also be poisonous? --Masatran (talk) 18:31, 2 June 2008 (UTC)[reply]

No, that's not a logical conclusion. For example, Gallium melts in the hand and it is non-toxic. Mercury has a low melting point, and it also happens to be toxic, but those two facts are independent and not logically connected. —Keenan Pepper 18:44, 2 June 2008 (UTC)[reply]
Does Gallium conduct electricity? If not: Are all liquids that conduct electricity poisonous? --Masatran (talk) 18:50, 2 June 2008 (UTC)[reply]
Salt water conducts reasonably well. It depends on your application. How conductive do you need it to be? APL (talk) 19:34, 2 June 2008 (UTC)[reply]
I'm not sure where you got this idea that liquid metals relate to toxicity but it's not the case. Toxicity depends on what your body does with it (or often doesn't do with it). For example, water is a liquid and it conducts electricity, but it is not poisonous. As for metals, there are several metallic ions there which are not poisonous (at least in the correct quantities). Regards, CycloneNimrod talk?contribs? 19:36, 2 June 2008 (UTC)[reply]
Ahem. Water is an excellent insulator. --Milkbreath (talk) 21:46, 2 June 2008 (UTC)[reply]
The OP seems to be committing the illicit minor fallacy. All A is B, all A is C, therefore all C is B - where A = mercury, B = toxic and C = liquid metal (or conductive liquids in his/hers second post). --Mark PEA (talk) 19:47, 2 June 2008 (UTC)[reply]
(ec) I don't know about gallium specifically, but most metals do conduct electricity. (Because they are held together with metallic bonds, where electrons are only loosely-held.) To get to the real meat of your question, though, you want to know "Why is mercury toxic?" I'm sorry I don't have a reference, but as I understand it, mercury poisoning is due mainly to the fact that mercury is not readily excreted (it bioaccumulates). This causes problems because mercury likes to bind to sulfur atoms, and sulfur atoms are a vital component to the activity of a number of proteins (e.g cysteine proteases). -- 128.104.112.147 (talk) 19:47, 2 June 2008 (UTC)[reply]
OK, I'll be the bad guy this time. Both "poisonous" and "conductive" are relative terms. Everything is poisonous at a certain dosage, and everything is conductive given enough voltage. I know that sounds like I'm just trying to be a jerk, but if you want good answers on the Science Desk, you've got to ask good questions. To try to answer, no, not all liquids that are good conductors of electricity are dangerous poisons, and neither are all metals that are liquid at temeratures that the human body can tolerate well. --Milkbreath (talk) 19:51, 2 June 2008 (UTC)[reply]

Toy helicopter

I want to buy a toy helicopter to get an understanding of aircraft. Is there any inexpensive model that has a control to fly forward? I am looking at Apache Havoc but I don't think it can fly forward. --Masatran (talk) 18:46, 2 June 2008 (UTC)[reply]

If you're looking for an understanding of aircraft broadly, then a helicopter isn't really a good choice -- that's a much more specific form of aircraft based on different principles. I'm afraid I can't advise regarding selection of an r/c copter if that's what you want to continue with. I will note that weighting the nose of a very cheap rc copter will tend to cause it to fly forward, though I doubt this constitutes the control you want. — Lomn 19:09, 2 June 2008 (UTC)[reply]
Here, for example, is a r/c copter with forward flight control -- but said forward flight is accomplished by pusher propellors, not the standard means of helicopter propulsion (again falling afoul of your "understanding of aircraft" criteria). — Lomn 19:16, 2 June 2008 (UTC) —Preceding unsigned comment added by Lomn (talkcontribs) [reply]

journal article without subscription

I am trying to access an article from a journal to which my university (u of oregon) does not subscribe. Our library also does not stock the print edition of this journal for any newer dates than 1988, so my only option is an inter-library loan and this usually takes over a week. I need the article sooner than that for a presentation, so I was hoping that someone here might be able to access it and provide it somehow.

The article is titled "The Redfield equation in condensed-phase quantum dynamics" by Pollard and Friesner, Adv. Chem. Phys. 1996

The article is cited by every other article I've been reading so I'd like to go directly to the source. My apologies if this request is a breach of wikipedia guidelines. Man It's So Loud In Here (talk) 19:17, 2 June 2008 (UTC)[reply]

I think it would probably breach copyright in some way to comply with your request, so I'd advise you to buy the article instead. Regards, CycloneNimrod talk?contribs? 19:33, 2 June 2008 (UTC)[reply]
As helpful as the above suggestion is, I'd like to think there is some site on the web where people do fulfill requests such as this one. Does anyone know of such a site? Man It's So Loud In Here (talk) 19:57, 2 June 2008 (UTC)[reply]
A helpful hint would not be posting such requests on very public boards such as this one. I'd consider asking elsewhere. Sorry I could not be more illegalhelpful. Regards, CycloneNimrod talk?contribs? 20:51, 2 June 2008 (UTC)[reply]
Perhaps if you were to write the author(s) directly, they could get a slave post-doc to send you a copy? Here's one of them. --Sean 22:01, 2 June 2008 (UTC)[reply]
Thank you very much for your help, I think that that will work very well. I just didn't see paying $30 as a viable option. If someone would like to delete this section, that would be fine with me. Man It's So Loud In Here (talk) 22:14, 2 June 2008 (UTC)[reply]

You can also try Wikipedia:WikiProject Resource Exchange, either posting to the requests page or asking one of the members. You can count me out though, as I don't have a subscription to that series. Someguy1221 (talk) 22:15, 2 June 2008 (UTC)[reply]

It is not illegal to distribute single copies of articles for academic purposes—it easily falls under fair use (even if might be a breach of user policies for the journals, but even then, most journals are pretty lenient when you are talking about very small-scale, not-for-profit, one-off stuff). My university doesn't carry e-copies of that one either (which surprises me), otherwise I'd be happy to send you a copy. Your best bet might be e-mailing one of the authors, if you want it quickly. It seems like a pretty rare journal. --Captain Ref Desk (talk) 22:15, 2 June 2008 (UTC)[reply]

Liquid state of water

Why is water a liquid and hydrogen sulphide a gas, when hydrogen sulphide's Mr is nearly twice as much as that of water's? —Preceding unsigned comment added by 195.158.94.171 (talk) 19:28, 2 June 2008 (UTC)[reply]

The conventional answer is hydrogen bonding. The hydrogens from one water molecule really like to bind to the lone pair electrons of an oxygen from a different water molecule. They can't do this in the gas phase, so the boiling point is much higher that it would be otherwise, as you have to add in enough energy to break all of those hydrogen bonds before the water will boil. Sulfur, for various reasons, doesn't participate all that well in hydrogen bonding, so hydrogen sulfide doesn't get the boiling point boost. -- 128.104.112.147 (talk) 19:33, 2 June 2008 (UTC)[reply]
The reason for H2S not having hydrogen bonds is due to the small difference in electronegativity of S and H atoms, where as O and H atoms have a much larger difference in electronegativity. --Mark PEA (talk) 19:52, 2 June 2008 (UTC)[reply]
Interesting. The density of water is almost the same as the density of liquid oxygen but the density of hydrogen sulfide is much less than the density of sulfer (which is approximately 2 g/cc. At least I think it is. Something else I read said it was 5.4 g/cc). Em3ryguy (talk) 20:43, 2 June 2008 (UTC)[reply]
In liquid sulfur the atoms are held together tightly with short covalent bonds, which is not the case in hydrogen sulfide liquid. Graeme Bartlett (talk) 21:58, 2 June 2008 (UTC)[reply]